Superpage
Page views in 2024 to date: 19

Abscess
Definition / general
  • Localized, space occupying lesion within the brain composed of purulent (neutrophil rich) exudate and other inflammatory cells reacting to the presence of bacteria within the CNS parenchyma
  • Can also be caused by nonbacterial pathogens, including fungi, protozoa and helminths
Essential features
  • Localized, pyogenic infections of brain parenchyma that occur via direct spread of contiguous infections, hematogenous spread or CNS trauma
  • Streptococcus spp. are the most common pathogens, although a wide range of organisms can cause disease; culture negative cases are not uncommon
  • Pathogenesis involves binding of microorganism to Toll-like receptors (TLRs), with subsequent release of proinflammatory molecules, blood brain barrier disruption, activation of astrocytes and resident microglia and influx of inflammatory cells
  • Radiographic imaging, particularly CT and MRI, is critical to accurate diagnosis; classical appearance is that of ring enhancing lesions with edema and mass effect
  • Biopsy with culture of the abscess provides the most specific diagnostic information; no other specific laboratory tests
Terminology
  • Synonyms include cerebral abscess and intracranial abscess
  • Localized collections of purulent exudate in adjacent extra-axial regions (i.e., subdural or epidural spaces) are more accurately referred to as empyemas
ICD coding
  • ICD-10: G06.0 - intracranial abscess and granuloma
Epidemiology
  • Prevalence 0.4 - 0.9 per 100,000 population in developed countries
  • Male predominance (2 - 3:1)
  • Most common between third and fifth decades of life
  • Risk factors include bacteremia, intravenous drug abuse, bacterial endocarditis, congenital cardiac defects with right to left shunts, hereditary hemorrhagic telangiectasia with pulmonary arteriovenous malformation, bronchiectasis, intra-abdominal infections, lung abscesses, paranasal sinus and ear infections, dental infections and immunosuppression (Neurohospitalist 2014;4:196)
  • Source of infection never identified in some cases (cryptogenic abscesses) (Clin Microbiol Infect 2017;23:614)
Sites
  • Can occur anywhere but most common in frontal and temporal lobes; cases associated with sinusitis most commonly involve frontal lobes and cases associated with otitis typically involve temporal lobes and much less commonly cerebellum (Neurology 2014;82:806, Neurohospitalist 2014;4:196)
  • Hematogenous abscesses favor middle cerebral artery territory and are more likely to be multiple (Int J Surg 2011;9:136)
  • Hematogenous abscesses typically located at gray-white junctions or in deeper cerebral white matter
Pathophysiology
  • Microorganisms reach brain parenchyma through 1 of 3 routes: hematogenous dissemination, extension from contiguous sites of infection and implantation via CNS trauma (including neurosurgical procedures)
  • Initial reaction to organisms involves binding of Toll-like receptors (especially TLR2 and TL4) to components of bacterial cell wall
  • TLRs interact MyD88, a cytoplasmic protein that plays a central role in the activation of the innate immune response
  • Activation of MyD88 dependent signaling pathways results in release of a host of proinflammatory cytokines and chemokines (including IL1 and TNF), blood brain barrier disruption, upregulation of ICAM1 and other adhesion molecules
  • Proinflammatory molecular pathways activate astrocytes and resident microglia and trigger an influx of neutrophils, followed by recruitment macrophages and T lymphocytes
  • References: J Neuroinflammation 2004;1:16, J Immunol 2007;178:4528, J Neuropathol Exp Neurol 2005;64:27, Am J Pathol 2008;172:132
Etiology
Clinical features
  • Classical triad includes headache (49 - 93%), fever (14 - 88%) and focal neurological deficits (29 - 71%) but all elements of triad are only seen in a minority of patients; other manifestations include altered mental status, nausea and vomiting, progressive behavioral changes and seizures (Clin Microbiol Infect 2017;23:614, Neurology 2014;82:806)
Diagnosis
  • Diagnosis is based on a combination of a thorough history and physical examination coupled with appropriate brain imaging studies, particularly MRI (Curr Opin Infect Dis 2017;30:129)
  • Biopsy may be helpful in distinguishing abscesses from other infectious and noninfectious processes (e.g., neoplasms)
  • Delayed diagnosis associated with worse prognosis
Laboratory
  • Preoperative laboratory tests are of limited value in the diagnosis of brain abscess; may see leukocytosis and elevated C reactive protein and erythrocyte sedimentation rate but these may be absent; blood cultures positive in a minority of patients (Clin Microbiol Infect 2017;23:614, Neurohospitalist 2014;4:196)
  • Cerebrospinal fluid (CSF) findings usually nonspecific and risks of lumbar puncture need to be carefully considered in patients with space occupying CNS lesions and mass effect (S Afr Med J 2000;90:609, Neurology 2014;82:806)
  • Intraoperative cultures should always be obtained in cases of suspected brain abscess
    • Material obtained from the suspected abscess should be submitted directly from the operating room to the microbiology laboratory for both aerobic and anaerobic bacterial cultures
    • Cultures for mycobacteria should be obtained if risk factors for tuberculosis are present, along with cultures for fungi in immunocompromised hosts (Clin Microbiol Infect 2017;23:614)
Radiology description
  • Brain imaging critical in the diagnosis of suspected brain abscess
  • Early lesions may appear as only an area of edema, with or without contrast enhancement
  • More fully developed abscesses are mass lesion(s) with a central necrotic core (hypodense on CT, hypointense on T1 weighted MRI, hyperintense on T2 weighted MRI and diffusion weighted MRI), surrounded by contrast enhancing smooth rim and edema in adjacent brain; fluid attenuated inversion recovery (FLAIR) sequences highlight edema surrounding abscess(es)
  • MRI more sensitive than CT in visualizing lesions
  • Additional imaging techniques (e.g., 1H nuclear MR spectroscopy) may be helpful in distinguishing bacterial abscesses from neoplastic lesions
  • References: J Popul Ther Clin Pharmacol 2020;27:e11, Radiographics 2015;35:1555
Radiology images

Contributed by Toral Patel, M.D.
Missing Image Missing Image Missing Image

Left parietal white matter

Prognostic factors
  • Mortality: ~10% in developed countries
  • Old age, delayed diagnosis and treatment, ventricular rupture, rapid progression of symptoms and coma associated with poorer prognosis
  • Chronic sequelae include seizures, permanent focal neurological deficits and, in younger children, cognitive impairment
  • Reference: BMC Infect Dis 2012;12:332
Case reports
Treatment
  • Optimum management involves multidisciplinary approach (neurosurgery, infectious disease, neuroradiology, pathology and neurology)
  • Empirical broad spectrum antimicrobial therapy covering both Gram positive and Gram negative bacteria is critical due to the wide range of potential pathogens; clinical features (e.g., age of the patient, potential source of infection, immunosuppression) can guide selection of antimicrobials
  • Guidelines for neurosurgical intervention vary from center to center; surgical intervention with drainage or excision of abscess is necessary in some patients owing to poor penetration of antimicrobials into abscess cavity and can also be of great value in the identification of the microorganisms causing the abscess
    • Other potential indications for surgical intervention include impending herniation, abscesses > 2.5 cm in diameter, location of abscess in paraventricular areas or posterior fossa
  • References: Curr Opin Infect Dis 2017;30:129, Neurohospitalist 2014;4:196
Gross description
  • Localized mass, typically well demarcated from adjacent brain, containing a variable quantity of opaque, purulent exudate; older lesions may be cystic, with an irregular shaggy lining surrounded by a fibrous capsule
Gross images

Contributed by Dennis K. Burns, M.D.
Missing Image

Brain, coronal section

Frozen section description
  • Tissue samples submitted for intraoperative frozen section are usually quite small and the appearance depends on the part of the abscess that was sampled
  • Central areas of the abscess present as purulent debris
  • More peripheral areas may contain only reactive astrocytes
Frozen section images

Contributed by Dennis K. Burns, M.D.
Missing Image Missing Image

Stereotactic brain biopsy

Microscopic (histologic) description
  • Early lesions appear microscopically as foci of cerebritis, characterized by edema and neutrophilic infiltration
  • Later lesions contain a central core of necrotic debris, organisms and neutrophils surrounded by a variably developed rim of granulation tissue, a mixed inflammatory infiltrates and activated macrophages and reactive astrocytes in the adjacent brain; over time, granulation tissue matures into a denser fibrous capsule
  • Histological evolution of abscesses has traditionally been divided into 4 stages (Clin Microbiol Infect 2017;23:614, Int J Surg 2011;9:136):
    • Stage I: early cerebritis (1 - 4 days)
    • Stage II: late cerebritis (4 - 10 days)
    • Stage III: early capsule stage (11 - 14 days)
    • Stage IV: late capsule stage (> 14 days)
Microscopic (histologic) images

Contributed by Dennis K. Burns, M.D.
Missing Image Missing Image

Paraffin section

Cytology description
  • As in the case of intraoperative frozen sections, the appearance of intraoperative touch or smear preparations is variable, depending on the area of the abscess that was biopsied; tissue from central areas is dominated by neutrophils in varying stages of degeneration, while tissue from more peripheral areas usually contains only nonspecific reactive elements, including reactive astrocytes
Positive stains
  • Standard stains for infectious agents (gram, acid fast, GMS) may highlight organisms
    • Histochemical staining is not sensitive; culture or other microbiologic testing usually necessary to identify organisms
Sample pathology report
  • Brain, left basal ganglia mass, stereotactic biopsy:
    • Acute inflammation consistent with abscess; Gram positive cocci identified on Gram stain
  • Brain, autopsy:
    • Organizing abscess, left frontoparietal region; no microorganisms identified
Differential diagnosis
  • Neoplasms (particularly metastatic carcinoma, high grade glial neoplasms, lymphoma)
    • Atypical, pleomorphic neoplastic epithelial, glial or lymphoid cells
    • Inflammatory infiltrates may be present but do not usually contain large numbers of neutrophils
  • Mycobacterial infections:
    • Lesions contain a granulomatous inflammatory infiltrate dominated by epithelioid macrophages and multinucleated giant cells, usually associated with caseous necrosis (neutrophils may be conspicuous in acute lesions)
    • Acid fast (Ziehl-Neelsen or Fite) stains may demonstrate acid fast bacilli
  • Nonbacterial infections (some parasitic and fungal infections):
    • Inflammatory infiltrates may be granulomatous or predominantly lymphocytic, depending upon the pathogen; neutrophilic infiltrates are less common
    • Parasitic organisms usually visible in H&E sections, while pathogenic fungi are best demonstrated with silver stains (Grocott-Gomori methenamine silver)
  • Infarcts:
    • Early lesions contain necrotic red neurons and fragmented, necrotic neuropil accompanied in the initial phases (1 - 2 days) accompanied by predominantly neutrophilic infiltrates; no organisms are demonstrable in gram, acid fast or GMS silver stains
    • Macrophages begin to infiltrate lesion beginning at about 48 hours, with older lesions composed of variable numbers of lipid laden macrophages (gitter cells), cystic, liquified brain parenchyma and reactive astrocytes
  • Organizing hemorrhage:
    • Numerous erythrocytes and hemosiderin laden macrophages
    • Astrocytic reaction develops at the periphery of the lesion, which is typically well demarcated from the surrounding neuropil
  • Radiation necrosis:
    • Prominent areas of coagulation necrosis, usually confined to white matter, accompanied by vascular changes that include vascular telangiectasia, hyaline vascular sclerosis with luminal stenosis and occasional lipid laden macrophages in vessel walls
  • Tumefactive demyelination:
    • Rare
    • Horseshoe pattern of enhancement at periphery of lesion
    • Macrophagic infiltrates, striking myelin loss with relative axonal sparing (although some degree of axonal injury / loss is usually seen)
Board review style question #1

A 46 year old man with a history of intravenous drug abuse presented with a 3 day history of fatigue, shortness of breath, fever and headache. Clinical evaluation revealed evidence of severe aortic regurgitation. Blood cultures were obtained in the emergency department (ED) and the patient was started on empirical antimicrobial therapy for presumed bacterial endocarditis. During transport from the ED to the ward, the patient sustained a cardiac arrest. Resuscitation was unsuccessful. A postmortem examination confirmed the presence of infective endocarditis involving the aortic valve, associated with destruction of the valve leaflets. Postmortem examination of the brain revealed the lesions shown above. Which one of the following is the earliest event in the development of the brain lesion?

  1. Binding of bacteria to Toll-like receptors (TLRs)
  2. Disruption of the blood brain barrier
  3. Inactivation of MyD88
  4. Migration of neutrophils into the lesion
  5. Release of IL1α
Board review style answer #1
A. Binding of bacteria to Toll-like receptors (TLRs). Experimental models have demonstrated that binding of bacteria to TLRs is the initial step in the development of brain abscesses. TLR binding triggers an innate immune response via activation of MyD88 dependent pathways, which results in the release of cytokines and chemokines, disruption of the blood brain barrier, activation of astrocytes and resident microglia and migration of inflammatory cells to the site of infection.

Comment Here

Reference: Abscess
Board review style question #2
Which one of the following provides the most specific information in the evaluation of a patient with a suspected brain abscess?

  1. Cerebrospinal fluid culture
  2. Complete blood count
  3. Erythrocyte sedimentation rate
  4. Magnetic resonance imaging
  5. Peripheral blood culture
Board review style answer #2
D. Magnetic resonance imaging. MRI plays a critical role in the evaluation of a patient with a suspected brain abscess. Although biopsy may be necessary to distinguish brain abscesses from other processes causing contrast enhancing, space occupying brain lesions (e.g., neoplasms), of the choices listed, radiographic imaging, particularly MRI, provides the most specific information about the presence or absence of a brain abscess. Abnormal peripheral blood counts and elevated erythrocyte sedimentation rates can be seen in patients with brain abscesses but are nonspecific and such tests may be normal in patients with brain abscesses. A positive blood culture can be helpful in identifying the responsible pathogen in cases of brain abscess but is also nonspecific by itself. Blood cultures, moreover, may be negative. Lumbar puncture and cerebrospinal fluid (CSF) studies are seldom useful in the evaluation of suspected brain abscesses and are usually contraindicated due to the danger of herniation in patients with expanding intracranial masses of any type.

Comment Here

Reference: Abscess

Alcoholic cerebellar degeneration
Definition / general
  • Atrophy of the cerebellar vermis seen in the setting of chronic alcoholism
Essential features
  • Characterized clinically by ataxia and gait disturbances in the setting of chronic alcoholism
  • Pathologic features include cerebellar atrophy affecting the anterior / superior vermis with a loss of Purkinje cells and corresponding Bergmann gliosis
Epidemiology
  • Occurs in approximately 10% of alcoholic patients
Sites
  • Anterior / superior cerebellar vermis
Pathophysiology
  • Cerebellar changes may be related in part to thiamine deficiency
  • Alterations in GABA receptor dependent neurotransmission have also been proposed as a pathogenic mechanism
Clinical features
  • Truncal ataxia
  • Unsteady gait
  • Nystagmus
  • May be clinically asymptomatic in some individuals
Laboratory
  • No specific laboratory abnormalities
Radiology description
  • Atrophy of the anterior / superior vermis
Radiology images

Images hosted on other servers:

Marked diffuse cerebellar atrophy

Prognostic factors
  • Cerebellar damage remains even after abstinence from ethanol
  • Prevention of cerebellar damage by treatment of alcoholism is recommended
Case reports
Gross description
  • Atrophy of the anterior / superior cererellar vermis with widened sulci
Gross images

Contributed by Kymberly A. Gyure, M.D.

Alcoholic cerebellar degeneration

Microscopic (histologic) description
  • Loss of cerebellar Purkinje cells with corresponding Bergmann gliosis
  • Narrowing of the molecular layer and a reduced number of granular cells may also be seen
Microscopic (histologic) images

Contributed by Kymberly A. Gyure, M.D.

Alcoholic cerebellar degeneration

Differential diagnosis
  • Age related cerebellar atrophy (generally milder)
  • Other causes of cerebellar vermal atrophy: phenytoin use, heavy metal poisoning or a subset of the spinocerebellar ataxias

Alzheimer
Definition / general
  • Alzheimer disease is clinically characterized by a progressive dementia and neuropathologically characterized by amyloid plaques and neurofibrillary tangles (NFT)
  • Sporadic disease is more common, which typically affects patients over the age of 65 years; however, rare early onset familial forms occur, which affect younger patients
Essential features
  • Clinical history of progressive dementia consistent with Alzheimer disease
  • Alzheimer disease neuropathologic changes (amyloid plaques, neurofibrillary tangles)
  • Can occur in the presence or absence of other neurodegenerative diseases
Terminology
  • Senile dementia
  • Sporadic Alzheimer disease
  • Early onset Alzheimer disease
  • Alzheimer disease with amygdala predominant Lewy bodies
  • Mixed dementia
ICD coding
  • ICD-10
    • G30.0 - Alzheimer disease with early onset
    • G30.1 - Alzheimer disease with late onset
    • G30.8 - other Alzheimer disease
    • G30.9 - Alzheimer disease, unspecified
  • ICD-11
    • 6D80.0 - dementia due to Alzheimer disease with early onset
    • 6D80.1 - dementia due to Alzheimer disease with late onset
    • 6D80.2 - Alzheimer disease dementia, mixed type, with cerebrovascular disease
    • 6D80.3 - Alzheimer disease dementia, mixed type, with other nonvascular etiologies
    • 6D80.Z - dementia due to Alzheimer disease, onset unknown or unspecified
Epidemiology
Sites
Pathophysiology
  • While the definitive pathophysiology of Alzheimer disease is still under investigation, there are a number of theories which may help explain development of Alzheimer disease
  • Beta amyloid may work to drive neurodegeneration
    • Initial accounts of the disease from the early 1900s demonstrated senile plaques based upon silver based histochemical stains (Clin Anat 1995;8:429)
    • Following this discovery, the progression of Alzheimer disease was thought to be based on the generation of beta amyloid plaques (Clin Anat 1995;8:429)
    • This was supported by the presence of autosomal dominant mutations in the amyloid pathway which lead to clinical Alzheimer disease including APP and presenilin 1 and 2 genes (PSEN1 / PSEN2) mutations as well as trisomy 21
      • These mutations lead to an increase in the prevalence of soluble and insoluble beta amyloid isomers (Clin Anat 1995;8:429)
    • Additionally, the biologic therapies directed at beta amyloid have met some success in delaying the progression of dementia
    • Despite this evidence in support of the amyloid pathway, there is reproducible evidence showing that beta amyloid accumulation and deposition does not correlate well with neuronal loss and cognitive decline (Arch Neurol 2008;65:1509)
    • The study of beta amyloid neurotoxicity in vivo shows that it does not reproduce the in vitro toxicity and that beta amyloid likely acts synergistically with tau to drive neuronal loss (JAMA Neurol 2014;71:505)
  • Abnormal microtubule associated tau (MAP tau or tau) may drive functional cognitive loss by neuronal dysfunction
    • While beta amyloid aggregates are found years or decades before the onset of clinical dementia, tau neurofibrillary tangles are strongly correlated with neurodegeneration and cognitive impairment
    • Cognitive impairment often coincides with the spreading of tau neurofibrillary tangles out of the medial temporal lobes and into the surrounding isocortex (Nat Med 2018;24:29)
    • There is a stereotypical spread of tau that is believed to be secondary to cell to cell spread through tau transmission across synapses
      • This idea is supported by animal models and spatial tau PET signal spreading along distributions reminiscent of functional brain networks (Neuron 2012;73:685, Brain 2018;141:568)
  • Neuroinflammation is also theorized to contribute to the pathophysiology of Alzheimer disease
    • The role of inflammation’s contribution to Alzheimer disease has come into focus more recently with the recognition that beta amyloid activates microglial cells causing a release of proinflammatory cytokines
    • These cytokines induce production of amyloid precursor protein (APP) and thus the production of beta amyloid
    • This is further supported by genome wide analysis which suggests that several genes which increase risk of Alzheimer disease are associated with the clearance of misfolded proteins and immune responses
    • Clinical data show external factors including systemic inflammation and obesity are likely to interfere with these processes and further promote disease progression (Lancet Neurol 2015;14:388)
Etiology
  • The true etiology of Alzheimer disease is still under investigation; however, there are epidemiological risk factors that contribute to the development of the disease
  • In addition, there are genetically linked forms of autosomal dominant Alzheimer disease that will cause dementia including mutations in APP and PSEN1 / PSEN2
Diagrams / tables

Images hosted on other servers:
Sampling for Braak staging

Sampling for Braak staging

Stages of Braak NFT spread

Stages of Braak NFT spread

CERAD neuritic plaque assessment

CERAD neuritic plaque assessment

Neuropathologic change

Neuropathologic change

Clinical features
  • Patients typically present with a history of progressive short term memory disturbances for ~1 - 3 years which can be clinically characterized as mild cognitive impairment (Alzheimers Dement 2011;7:263)
  • It is common to have concurrent agitation or depression
Diagnosis
  • The diagnosis of Alzheimer disease requires both a clinical diagnosis as well as neuropathological changes
  • Clinical dementia is diagnosed as possible or probable Alzheimer disease dementia when there are cognitive or behavioral symptoms which (Alzheimers Dement 2011;7:263)
    • Interfere with the ability to function at work or usual activities
    • Represent a decline from previous levels of functioning
    • Are not explained by delirium or major psychiatric disorder
    • Involves a minimum of 2 cognitive domains listed below
      • Inability to acquire and remember new information
      • Impaired reasoning and handling of complex tasks, poor judgment
      • Impaired visuospatial abilities
      • Impaired language functions
      • Changes in personality, behavior or comportment
  • Alzheimer disease neuropathologic changes are graded using higher levels of pathology associated with dementia
  • Alzheimer disease neuropathologic changes are currently reported as an ABC score, which includes 3 separate diagnostic schemes (Alzheimers Dement 2012;8:1)
Laboratory
  • Elevated homocysteine levels are associated with the development of Alzheimer disease
  • Assays investigating beta amyloid 42/40 ratios in the cerebral spinal fluid (CSF) have been approved by the Food and Drug Administration (FDA) in patients over the age of 55 years who present with cognitive impairment for the evaluation of Alzheimer disease
  • Similar assays for the beta amyloid 42/40 ratio using blood plasma have cleared Clinical Laboratory Improvement Amendments (CLIA) regulations but have not yet been approved by the FDA
  • Assays for phosphorylated tau are being investigated for diagnostic purposes (Ann Neurol 2009;65:403)
  • Proteomic assays of the CSF have been reported and provide enhanced diagnostic and prognostic information in excess of the beta amyloid and tau assays (Sci Transl Med 2023;15:eadg4122)
  • Alzheimer disease biomarkers typically help establish the presence of Alzheimer disease neuropathologic changes but do not exclude the presence of other pathologies contributing to dementia (i.e., mixed dementia)
Radiology description
  • The current radiological workup takes advantage of multiple modalities to assess structural and functional metrics used in Alzheimer disease workups (Int J Mol Sci 2022;23:6079)
  • Structural magnetic resonance imaging (MRI) demonstrates cerebral atrophy and ventricular enlargement; hippocampal volumes can be helpful in both structural and functional assessments
  • Fluorodeoxyglucose positron emission tomography (FDG PET) imaging can highlight accumulations of amyloid and tau as well as glucose metabolism; these tests are FDA approved for diagnostic purposes
  • Functional MRI can be used for the detection of hyper and hypoactivation while investigating task related regions
  • Electroencephalography can be used to assess altered electrical function
Radiology images

Images hosted on other servers:
Cortical atrophy on magnetic resonance imaging

Cortical atrophy on MRI

Pittsburgh compound B PET imaging

Pittsburgh compound B PET imaging

Prognostic factors
  • Shorter survival in clinically diagnosed Alzheimer disease associated with older age, male sex, increasing comorbidities and medications, lower cognitive function at diagnosis or suspected mixed dementia (Neurology 2020;94:e538)
Case reports
Treatment
  • NMDA receptor antagonists (i.e., memantine) is used to slow the cognitive decline by blocking neuronal excitotoxicity (CNS Drug Rev 2003;9:275)
  • Acetylcholinesterase inhibitors (i.e., rivastigmine) can help symptomatically improve neuropsychiatric symptoms such as apathy through slowing the degradation of neurotransmitters
  • Monoclonal antibodies directed against beta amyloid isoforms have gained FDA approval and work to slow the progression of cognitive impairment
  • These include aducanumab and lecanemab (N Engl J Med 2023;388:9)
Clinical images
N/A
Gross description
  • Cortical atrophy can be externally observed with narrowing of the gyri and widening of the sulci
    • This can be associated with a reduced brain weight
    • The atrophy selectively affects the frontal, temporal and parietal lobes with relative sparing of the occipital lobe
  • There can be significant atrophy of the amygdala and hippocampus
  • Hydrocephalus ex vacuo, shown by symmetric dilation of the lateral ventricles and third ventricle, is common
  • Pallor of the locus ceruleus is common
  • Pallor of the substantia nigra may occur in ~25 - 33% of cases and may be associated with extrapyramidal signs (Neurology 2005;64:1397, Arch Pathol Lab Med 1993;117:132)
Gross images

Contributed by Bartholomew White, M.D.
Cortical atrophy

Cortical atrophy

Ventricular dilation

Ventricular dilation

Substantia nigra and locus ceruleus

Substantia nigra and locus ceruleus

Frozen section description
N/A
Frozen section images
N/A
Microscopic (histologic) description
  • Alzheimer disease is pathologically characterized by extracellular amyloid plaques and intracellular neurofibrillary tangles
    • Amyloid plaques are visible on H&E staining as a circular disorganization of the neuropil, which may be accompanied by activated microglia and a dense amorphous eosinophilic core
    • Amyloid plaques are dark on silver staining and are separated into 2 types: diffuse and neuritic
      • Neuritic plaques (NP) show abnormal dystrophic neurites associated with the plaque on silver or phosphorylated tau immunostaining; these plaques count toward the C and A scores
      • Diffuse plaques lack central cores or dystrophic neurites and can be detected on silver or amyloid staining; these count toward the A score
    • Neurofibrillary tangles are visible on H&E staining as pale fibrillar basophilic aggregates within neuronal cytoplasm
    • Neurofibrillary tangles are dark fibrillar aggregates in neurons on silver staining
    • Phosphorylated tau immunostaining can highlight both neurofibrillary tangles and more granular pretangle aggregates
    • Both aggregates are better visualized for staging purposes on silver staining or immunostaining; H&E is not recommended for staging
  • Alzheimer disease neuropathological change is currently assessed by the ABC scoring system using a 10x objective when density assessment is required (Alzheimers Dement 2012;8:1)
  • The A score is derived from the Thal phase, which assesses regional involvement by beta amyloid deposits (Neurology 2002;58:1791)
    • Thal phase 1 - 2 is designated A1 and includes beta amyloid in the neocortex (Thal 1) and extends into the entorhinal region and CA1 of the hippocampus (Thal 2)
    • Thal phase 3 is designated A2 and includes extension of beta amyloid into the thalamus and deep gray matter structures
    • Thal phase 4 - 5 is designated A3 and includes extension of beta amyloid into brainstem areas such as the substantia nigra (Thal 4) and molecular layer of the cerebellum (Thal 5)
  • The B score is derived from the Braak stage, which assesses regional involvement by neurofibrillary tangles (Acta Neuropathol 2006;112:389)
    • Braak stage 1 - 2 is designated B1, neurofibrillary tangles start in the transentorhinal cortex (Braak 1) and extend into the entorhinal cortex and hippocampus (Braak 2)
    • Braak stage 3 - 4 is designated B2, neurofibrillary tangles extend to involve the neocortex starting in the fusiform and lingual gyri (Braak 3) and extending in a fan-like distribution into other association areas (Braak 4)
    • Braak stage 5 - 6 is designated B3, neurofibrillary tangles extend into the peristriate (Braak 5) and then the primary visual cortex (Braak 6) in the occipital lobe
  • The C score is derived from the CERAD score and is a semiquantitative assessment of neuritic plaques in the frontal, parietal and temporal lobes (the occipital lobe is excluded from this scheme) (Neurology 1991;41:479)
    • CERAD sparse neuritic plaques are designated C1
    • CERAD moderate neuritic plaques are designated C2
    • CERAD frequent neuritic plaques are designated C3
    • This grading scheme only recognizes neuritic plaques; diffuse plaques are not included in the criteria
  • A low level of Alzheimer disease neuropathologic changes is unlikely to account for a clinical history of dementia but can contribute to dementia in combination with other neurodegenerative or vascular pathologies
  • Cerebral amyloid angiopathy is a common finding associated with Alzheimer disease pathology
    • Characterized by amorphous eosinophilic beta amyloid accumulation around cerebral vessels in a double barrel pattern
    • More common in individuals with APOE ɛ4 allele (Alzheimers Dement 2012;8:1)
  • Lewy bodies, aggregates of alpha synuclein in neurons, restricted to the olfactory bulb and amygdala can occur with Alzheimer disease neuropathologic changes (J Neuropathol Exp Neurol 2006;65:685, Acta Neuropathol 2008;116:17)
  • Vacuolization of the superficial layers of cortex is commonly associated with atrophy in Alzheimer disease but is not specific
  • Hirano bodies and granulovacuolar degeneration can be seen at higher levels in the hippocampus associated with Alzheimer disease neuropathologic changes but are nonspecific
  • Other age related vascular and neurodegenerative pathologies commonly co-occur with Alzheimer disease but are reported as separate diagnoses
Microscopic (histologic) images

Contributed by Bartholomew White, M.D.
Granulovacuolar degeneration

Granulovacuolar degeneration

Hirano bodies

Hirano bodies

Diffuse plaque

Diffuse plaque

Neuritic plaque

Neuritic plaque

Neurofibrillary tangle

Neurofibrillary tangle

Diffuse plaque

Diffuse plaque


Neuritic plaque

Neuritic plaque

Moderate neuritic plaques Moderate neuritic plaques

Moderate neuritic plaques

Beta amyloid aggregates

Beta amyloid aggregates

Neurofibrillary tangle Neurofibrillary tangle

Neurofibrillary tangle

Virtual slides
N/A
Cytology description
N/A
Cytology images
N/A
Immunofluorescence description
N/A
Immunofluorescence images
N/A
Positive stains
  • Beta amyloid immunostaining highlights beta amyloid deposition in both diffuse plaques and neuritic plaques
  • Thioflavin S can be used to highlight amyloid plaques and neurofibrillary tangles
  • Phosphorylated tau immunostaining highlights pretangle aggregates, neurofibrillary tangles and dystrophic neurites in neuritic plaques
  • Silver staining methods can highlight amyloid plaques and neurofibrillary tangles, as well as other pathologic processes
Negative stains
  • Presence of additional neurodegenerative disease pathology leads to the diagnosis of dementia with mixed pathologies
  • Alpha synuclein immunostaining should be negative for aggregates outside of the amygdala and olfactory bulb
  • TDP-43 immunostaining should not show additional aggregates unless other neurodegenerative pathologies are present
Electron microscopy description
N/A
Electron microscopy images
N/A
Molecular / cytogenetics description
  • Major genetic risk allele for sporadic Alzheimer disease is associated with the ɛ4 allele of the APOE gene, while the ɛ2 allele is protective (Ann Neurol 1996;39:395)
    • Other genetic risk alleles confer less risk compared with APOE
  • Early onset Alzheimer disease is most commonly associated with mutations in the presenilin 1 and 2 genes (PSEN1, PSEN2), mutations in the amyloid precursor protein gene (APP) or with trisomy 21 (APP is located on chromosome 21)
Molecular / cytogenetics images
N/A
Videos
N/A
Sample pathology report
  • Adult brain (1300 grams), [procedure]:
    • Alzheimer disease neuropathological changes, high level (A3, B3, C3) (see comment)
    • External examination: The fresh brain weight is 1300 grams. A portion of dorsal cerebral dura is available for examination and shows no significant epidural or subdural hemorrhage. The superior sagittal sinus is patent. The leptomeninges are thin and translucent without significant subarachnoid hemorrhage. There is moderate generalized cortical atrophy. The circle of Willis has a normal anatomic configuration and no atherosclerosis. No aneurysms are identified. The cranial nerves occupy appropriate locations without anomalies or lesions. There is no evidence of cingulate gyrus, uncal or cerebellar tonsillar herniation. The brainstem and cerebellum have unremarkable external appearances.
    • Coronal sections: Serial coronal sections of the cerebral hemispheres reveal the cortical and subcortical gray and white matter structures to be of normal configuration without hemorrhages, infarcts or mass lesions. There is mild atrophy of the hippocampus. The basal ganglia, thalamus, amygdalae and mamillary bodies bilaterally are of normal size, color and consistency. The ventricular system shows moderate hydrocephalus of the lateral and third ventricles. Serial sections of the brainstem and cerebellum reveal no lesions and show pallor of the locus ceruleus. The substantia nigra is well pigmented.
    • Microscopic description: H&E stained sections show superficial spongiosis of the gray matter with scattered amyloid plaques. Neurofibrillary tangles are present in the entorhinal cortex, hippocampus and locus ceruleus. Hirano silver stains show frequent neuritic plaques in the frontal, parietal and temporal cortices, which is considered C3 using the updated National Institute on Aging Alzheimer's Association (NIA AA) criteria (Alzheimers Dement 2012;8:1). Neurofibrillary tangles are frequent in the entorhinal cortex, hippocampus and temporal cortex; while moderate neurofibrillary tangles are present in the frontal cortex, parietal cortex and nonstriated occipital cortex. Sparse neurofibrillary tangles are present in the striate cortex, corresponding to B3 using the updated NIA AA criteria (Alzheimers Dement 2012;8:1). Amyloid beta immunohistochemistry is positive in multiple areas, including the thalamus, midbrain and cerebellum, which corresponds to A3 in the NIA AA system (Alzheimers Dement 2012;8:1). According to the 2012 NIA AA criteria, this level of amyloid, NFT and NP pathology (A3, B3, C3) is considered a high level.
    • Comment: The high level of Alzheimer disease neuropathologic changes support the clinical impression of Alzheimer disease.
Differential diagnosis
  • Dementing diseases span a broad spectrum; many are classified by the predominant protein which is thought to drive the disease
    • Non-Alzheimer tauopathies (progressive supranuclear palsy [PSP], corticobasal degeneration [CBD], Picks disease, etc.)
      • Clinical history does not match Alzheimer disease
      • More pronounced frontal and temporal lobe atrophy or atrophy of specific subcortical brain regions (subthalamic nucleus, substantia nigra, areas involved with progressive supranuclear palsy [PSP])
      • Abundant tau inclusions
      • Amyloid is usually absent
    • Alpha synucleinopathies (Parkinson disease, Lewy body dementia, multiple system atrophy, etc.):
      • Clinical history does not match Alzheimer disease
      • Cortical atrophy is not as pronounced
      • Alpha synuclein positive aggregates
      • More frequently associated with a pale substantia nigra and locus ceruleus
      • Atrophy of the cerebellum or putamen (multiple system atrophy)
    • TDP-43 proteinopathies (limbic predominant age related TDP-43 encephalopathy [LATE], frontotemporal lobar degeneration [FTLD], etc.):
      • LATE can be seen in the setting of Alzheimer disease
      • The clinical findings of frontotemporal lobar degeneration do not usually overlap with Alzheimer disease
      • TDP-43 protein aggregates are present
    • Vascular dementia:
      • Marked by infarcts
      • Can commonly occur with Alzheimer disease
      • May present clinically with a step-like neurocognitive decline
Board review style question #1

Evidence of which protein pathology is required for the diagnosis of Alzheimer disease?

  1. Alpha synuclein
  2. Beta amyloid
  3. GFAP
  4. TDP-43
Board review style answer #1
B. Beta amyloid. The diagnostic criteria for the assessment of Alzheimer disease hinges on both beta amyloid and tau pathologies. Answer A is incorrect because alpha synuclein is commonly seen with Lewy body dementia and Parkinson disease. Answer C is incorrect because GFAP is typical of reactive astrocytosis which can be seen with vascular dementia. Answer D is incorrect because TDP-43 is present in both frontotemporal lobar degenerations and limbic associated, age related TDP-43 encephalopathy (LATE disease).

Comment Here

Reference: Alzheimer
Board review style question #2
Which genetic marker is most associated with sporadic Alzheimer disease?

  1. APP mutations
  2. APOE ɛ4
  3. Presenilin mutations
  4. Trisomy 21
Board review style answer #2
B. APOE ɛ4. APOE ɛ4 is commonly associated with sporadic Alzheimer disease with the risk for the disorder increasing with the number of alleles present. Answers A and C are incorrect because APP mutations and presenilin mutations are associated with autosomal dominant Alzheimer disease. Answer D is incorrect because trisomy 21 is associated with early onset Alzheimer disease.

Comment Here

Reference: Alzheimer
Board review style question #3
Which is the most common form of dementia?

  1. Alzheimer disease
  2. Lewy body dementia
  3. Picks disease
  4. Progressive supranuclear palsy
Board review style answer #3
A. Alzheimer disease. Alzheimer disease is the most common form of dementia and is responsible for 60 - 70% of cases. Answer B is incorrect because Lewy body dementia is attributed to ~20% of cases. Answers C and D are incorrect because the remaining 10% is comprised of a diverse set of dementing diseases.

Comment Here

Reference: Alzheimer

Alzheimer (pending)
Table of Contents
Definition / general
Definition / general
[Pending]

Amyotrophic lateral sclerosis
Definition / general
  • Amyotrophic lateral sclerosis (ALS) is a chronic, progressive neurologic disease characterized by degeneration of upper and lower motor neurons in the central nervous system
  • Motor neuron loss results in progressive and irreversible loss of motor function, muscle weakness and wasting and ultimately death, usually due to respiratory failure
Essential features
  • Neurodegenerative disease that affects motor neurons; incurable and uniformly fatal
  • Clinically, characterized by upper and lower motor neuron signs / symptoms
  • Key pathologic findings: gross atrophy of anterior spinal nerve roots, neuronal loss and gliosis affecting anterior horn of spinal cord and primary motor cortex, pallor of corticospinal tracts, Bunina bodies (cystatin C positive) and skein-like inclusions (TDP-43 and ubiquitin positive) in motor neurons
  • Familial ALS: C9orf72 expansion and SOD1 mutation are the 2 most common inherited genetic alterations
Terminology
  • Motor neuron disease: in the U.S., motor neuron disease includes ALS and other related disorders; in the U.K., motor neuron disease is preferred term for ALS
  • Upper motor neurons (UMN): eponymously known as Betz cells; originate in layer 5 of the primary motor cortex (located in the precentral gyrus) and project to lower motor neurons in brainstem and spinal cord
  • Lower motor neurons (LMN): originate in brainstem motor nuclei and the anterior column of the spinal cord; project to skeletal muscle end plates
  • Corticospinal tracts: white matter consisting of myelinated fibers of descending upper motor neurons
  • Progressive bulbar palsy (PBP): syndrome characterized by progressive dysarthria and dysphagia
  • Progressive muscular atrophy (PMA): syndrome characterized by lower motor neuron signs in the absence of upper motor neuron signs
  • Primary lateral sclerosis (PLS): syndrome characterized by upper motor neuron signs in the absence of lower motor neuron signs, with pathologic changes restricted to the motor cortex and corticospinal tracts
  • Some consider progressive bulbar palsy, progressive muscular atrophy and primary lateral sclerosis to be related variants of ALS; others consider ALS, progressive bulbar palsy and progressive muscular atrophy to be variants of the same clinicopathologic entity, while primary lateral sclerosis is considered distinct
  • Frontotemporal dementia (FTD): clinically heterogeneous group of dementia syndromes with pathology characterized by preferential atrophy of the frontal and temporal lobes, typically mediated by pathologic accumulation of protein (e.g., TDP-43); may copresent with ALS
  • ALS is popularly known as Lou Gehrig disease
ICD coding
  • ICD-10: G12.21 - amyotrophic lateral sclerosis
  • ICD-11: 8B60.0 - amyotrophic lateral sclerosis
Epidemiology
Sites
  • Primary motor cortex
  • Brainstem motor nuclei
  • Spinal cord: anterior horn motor neurons and corticospinal tracts
  • Skeletal muscles are affected by motor neuron loss
Pathophysiology
  • Loss of motor neurons results in skeletal muscle denervation and progressive loss of motor function
Etiology
Clinical features
  • Progressively worsening muscle weakness, leading to loss of mobility and respiratory failure
  • Upper motor neuron specific signs and symptoms
    • Brisk tendon reflexes
    • Spasticity
    • Pseudobulbar signs (dysarthria, dysphagia, face and tongue weakness, emotional lability)
  • Lower motor neuron specific signs and symptoms
    • Skeletal muscle weakness and wasting
    • Fasciculations
Diagnosis
  • ALS diagnosis relies on history, clinical neurologic exam, electrophysiologic evaluation and neuropathologic evaluation (generally via autopsy examination of brain and spinal cord)
  • El Escorial criteria for ALS diagnosis was originally published in 1994 and revised in 2000 to include electrophysiology; Awaji criteria further integrates clinical exam with electrophysiology (J Neurol Sci 1994;124:96, Amyotroph Lateral Scler Other Motor Neuron Disord 2000;1:293, Amyotroph Lateral Scler 2009;10:53)
  • Current Awaji criteria:
    • ALS diagnosis requires the following:
      • Evidence of lower motor neuron degeneration by clinical, electrophysiological or neuropathological examination
      • Evidence of upper motor neuron degeneration by clinical examination
      • Progressive spread of symptoms or signs within a region or to other regions, as determined by history, physical examination or electrophysiologic tests
    • And the absence of:
      • Electrophysiological or pathological evidence of other disease processes that might explain the signs of lower or upper motor neuron degeneration and
      • Neuroimaging evidence of other disease processes that might explain the observed clinical and electrophysiological signs
    • Diagnostic categories:
      • Clinically definite ALS: clinical or electrophysiological evidence of lower as well as upper motor neuron signs in the bulbar region and at least 2 spinal regions or the presence of lower and upper motor neuron signs in at least 3 spinal regions
      • Clinically probable ALS: clinical or electrophysiological evidence of lower and upper motor neuron signs in at least 2 regions with some upper motor neuron signs necessarily rostral to the lower motor neuron signs
      • Clinically possible ALS: clinical or electrophysiological evidence of upper and lower motor neuron signs in only 1 region or upper motor neuron signs found alone in at least 2 regions or lower motor neuron signs found rostral to upper motor neuron signs
      • El Escorial criteria previously included Clinically Probable Laboratory Supported ALS but this category was deemed redundant and eliminated from Awaji criteria
Laboratory
  • Routine laboratory studies may be helpful for ruling out metabolic or toxic etiology of symptoms
  • Genetic testing can identify familial ALS
Radiology description
  • Imaging may be more useful to rule out other etiologies than to provide direct evidence of ALS
  • Notable features that have been described: increased FLAIR and T2 signal in corticospinal tracts and hypodensity on T2 of motor cortex (Radiology 1999;212:763, Radiology 1993;189:843)
Prognostic factors
Case reports
Treatment
  • Disease is progressive, incurable and uniformly fatal
  • 2 FDA approved disease modifying therapies:
  • Supportive care: management of symptoms (muscle spasms, fatigue, pain), mobility and functional assistance for activities of daily living, goals of care planning for patient and caregivers (wishes for nutritional or respiratory support)
Gross description
Gross images

Contributed by Hannes Vogel, M.D.
Spinal cord atrophy

Spinal cord atrophy

Microscopic (histologic) description
Microscopic (histologic) images

Contributed by Eileen Bigio M.D. and Qinwen Mao, M.D. Ph.D.
Spinal nerve roots

Spinal nerve roots

ALS versus normal spine

ALS versus normal spine

Corticospinal tract

Corticospinal tract

Motor cortex

Motor cortex

Bunina bodies

Bunina bodies

Lewy-like body

Lewy-like body



Contributed by Hannes Vogel, M.D., Matthew McCord, M.D., Christina Appin, M.D. and Missia Kohler, M.D.
Inclusions immunoreactive for TDP-43

Inclusions immunoreactive for TDP-43

ALS / FTD overlap

ALS / FTD overlap

C9orf72 p62 positive inclusions

C9orf72 p62 positive inclusions

Positive stains
  • TDP-43: positive in skein-like inclusions and Lewy-like inclusions
  • Ubiquitin: positive in skein-like inclusions and Lewy-like inclusions
  • Myelin stain (e.g., Luxol fast blue): loss of staining highlights pallor of corticospinal tracts
  • p62: distinctive star shaped positive inclusions in cerebellar Purkinje cells and hippocampal dentate gyrus in C9orf72 linked ALS / frontotemporal dementia (Acta Neuropathol 2011;122:691)
Negative stains
Electron microscopy description
Molecular / cytogenetics description
Sample pathology report
  • Brain, autopsy:
    • Histopathologic alterations compatible with amyotrophic lateral sclerosis (ALS):
      • Upper motor neuron pathology, mild
      • Lower motor neuron pathology, moderate
    • Histopathologic alterations compatible with frontotemporal lobar degeneration with TDP-43 proteinopathy (FTLD TDP), type B
    • Cerebrovascular disease, mild (nonocclusive):
      • Mild atherosclerosis
      • Mild arteriolosclerosis
      • No cerebral amyloid angiopathy
      • Up to moderate subcortical white matter pallor
    • No Lewy body pathology
    • Clinical information:
      • The decedent is a 65 year old woman with an 8 year history of frontotemporal dementia and progressive motor neuron disease.
    • Gross description:
      • Brain weight: The unfixed brain weighs 1,376 g.
      • Vascular: Examination of the circle of Willis shows mild cerebrovascular atherosclerosis. There are no large or lacunar infarcts and no hemorrhages.
      • General: External examination of the brain reveals mild atrophy of the bilateral frontal and temporal cortices (symmetric) and no atrophy of the bilateral parietal or occipital cortices. Coronal sections of the cerebral hemispheres reveal moderate atrophy of the bilateral hippocampi (symmetric) and no atrophy of the bilateral caudate nuclei. There is mild dilatation of the ventricular system. External examination and transverse sections of the brainstem reveal no atrophy of the pons, mild pallor of the substantia nigra and moderate pallor of the locus coeruleus. External examination and sagittal and parasagittal sections of the cerebellum reveal no atrophy of the bilateral dentate nuclei. Spinal cord is not received for evaluation.
    • General microscopic description:
      • Microscopic sections are prepared from 23 regions, including bilateral dorsolateral prefrontal cortices (BA8-9), bilateral inferior frontal (Broca) gyri (BA44), bilateral superior temporal gyri (BA22), bilateral inferior parietal lobules (BA39), primary motor cortex (BA4), primary somatosensory cortex (BA3), primary visual cortex (BA17), anterior cingulate gyrus (BA24), posterior cingulate gyrus (BA23), striatum, basal ganglia with nucleus basalis of Meynert, thalamus with subthalamic nucleus, bilateral mammillary bodies, amygdala with entorhinal cortex, anterior hippocampus with entorhinal cortex, bilateral posterior hippocampi, cerebellum with dentate nucleus, cerebellar vermis, midbrain, pons and medulla. Sections are all from the left except as noted and are evaluated using H&E and fluorescent Thioflavin S and immunohistochemical preparations for tau (AT8), p62 (BD Biosciences), phosphorylated TDP-43 (CosmoBio), phosphorylated alpha synuclein (phospho S129) and beta amyloid (4G8).
      • Microscopic examination reveals subcortical white matter pallor as summarized in the table below. There is mild hyaline arteriolosclerosis. Thioflavin S stains reveal no cerebral amyloid angiopathy. There is no Fahr vascular mineralization. There are no microinfarcts or lacunar infarcts.
      • Microscopic examination reveals cortical layer II microvacuolation and gliosis and overall neuronal loss and gliosis as summarized in the table below. There is no pigment spheroid degeneration. No ballooned neurons are observed. There are no Pick bodies in the dentate gyri.
      • Upper and lower motor neuron system:
        • Microscopic examination reveals motor subcortical and descending corticospinal tract white matter pallor or sclerosis as summarized in the table below. There is mild neuronal loss and gliosis of the primary motor cortex, with no neuronophagia appreciated. In lower motor neurons, there is moderate neuronal loss and gliosis.

    General microscopic description summary
    (Cortical and subcortical areas left sided unless otherwise indicated)
    White matter myelin pallor or sclerosis Motor cortex ++
    Nonmotor cortex Up to ++
    Internal capsule +
    Cerebral peduncle CSTs +, (left worse than right)
    Basis pontis CSTs Min
    Medullary pyramids +
    Cerebellum +
    Hyaline arteriolosclerosis +
    Fahr vascular mineralization 0
    Microinfarcts 0
    Lacunar infarcts 0
    Superficial microvacuolation and gliosis Entorhinal cortex 0
    Frontal cortex Up to ++
    Temporal cortex 0
    Motor cortex Min
    Neuronal loss & gliosis Hippocampus + (L)
    Entorhinal cortex L+, R++
    Amygdala +
    Nonmotor cortex Up to +
    Motor cortex +
    Caudate +
    Putamen Min
    Globus pallidus 0
    Nucleus basalis of Meynert +
    Thalamus 0
    Subthalamic nucleus 0
    Substantia nigra Min
    Locus coeruleus Min
    Cerebellar Purkinje cells 0
    Cerebellar dentate nucleus +
    Hypoglossal nucleus ++
    Pigment spheroid degeneration 0
    Ballooned neurons 0
    Pick bodies 0
    Neuronophagia, motor cortex 0
    Other significant histologic findings None
    0 = absent, Min = minimal, + = mild, ++ = moderate, +++ = severe, L = left, R = right, B = bilateral,
    HGN = hypoglossal nucleus, CSTs = corticospinal tracts, C = cervical, T = thoracic, L = lumbar

    • TDP-43 pathology:
      • Phosphorylated TDP-43 immunostains of the neocortex highlight neuronal cytoplasmic inclusions (NCIs), both compact and granular, glial cytoplasmic inclusions (GCIs) and short, dystrophic neurites (DNs). Tau and alpha synuclein immunostains are negative in the inclusions and neurites. Threads and dots (ThDs) are also present in the background. TDP-43 pathology involves the full thickness of the cortex. No neuronal intranuclear inclusions (NIIs) are appreciated. DNs are of the short type, with long dystrophic neurites not appreciated. Semi quantitative estimates of NCIs / GCIs and DNs / ThDs are summarized in the table below. TDP-43 pathology in motor neurons is summarized separately, in the subsequent section.

    TDP-43 pathology summary
    (Cortical and subcortical areas left sided unless otherwise indicated)
    Region CIs NIIs DN / ThDs
    Cerebral cortex
    Middle frontal gyrus L++, R+ B 0 L+, R Min
    Superior temporal gyrus L+, R+ (left worse) B 0 L Min, R+
    Inferior parietal lobule L+, R+ B 0 L+, R+
    Sensory cortex Min 0 Min
    Entorhinal cortex L+++, R+++ B 0 L+, R++
    Subcortical areas
    Amygdala +++ 0 ++
    Dentate gyrus L+++, R+++ B 0 B 0
    Hippocampus / subiculum L+, R+ B 0 L Min, R Min
    Caudate +++ 0 ++
    Putamen ++ 0 Min
    Globus pallidus +++ 0 +++
    Internal capsule 0 N/A 0
    Nucleus basalis of Meynert + 0 Min
    Brainstem and cerebellum
    Inferior olivary nucleus +++ 0 +
    Cerebellar dentate nucleus 0 0 0
    Cerebellar white matter 0 0 0
    CIs = cytoplasmic inclusions (including neuronal and glial cytoplasmic inclusions), NIIs = neuronal intranuclear inclusions,
    DN / ThDs = dystrophic neurites or threads and dots, 0 = absent, Min = minimal, + = sparse, ++ = moderate, +++ = frequent,
    L = left, R = right, B = bilateral, N/A = not applicable, *focal

    • ALS typical pathology and TDP-43 IHC:
      • ALS pathology includes the above mentioned: neuronal loss and gliosis in the primary motor cortex; white matter rarefaction of motor subcortical white matter and descending corticospinal tracts; and neuronal loss and gliosis in the hypoglossal nuclei. The motor cortex and hypoglossal nuclei show chromatolysis, Bunina bodies and TDP-43 positive inclusions (or the lack thereof) as summarized in the table below.

    Motor neuron disease pathology summary
    Chr BBs TDP+ NCIs* TDP+ LLBs TDP+ GCIs
    Motor cortex N/A N/A + N/A +
    Hypoglossal nucleus 0 ++ ++ 0 +
    0 = absent, + = sparse, ++ = moderate, +++ = frequent, N/A = not applicable, Chr = chromatolysis, BBs = Bunina bodies,
    NCIs = neuronal cytoplasmic inclusions, LLBs = Lewy-like bodies, GCIs = glial cytoplasmic inclusions,
    *NCIs include granular and skein-like inclusions

    • Lewy body pathology:
      • H&E sections reveal no Lewy bodies in the brainstem or nucleus basalis of Meynert. Alpha synuclein immunostains of the left anterior and posterior cingulate gyrus, left amygdala with entorhinal cortex and right middle frontal gyrus reveal no Lewy bodies.
    • p62 immunostains:
      • p62 immunostains of the left middle frontal gyrus, left anterior hippocampus, bilateral posterior hippocampi and cerebellum highlight a subset of aging pathology only (FTLD TDP-43 and ADNC). There are no p62 positive / TDP-43 negative inclusions of the type seen in C9orf72 repeat expansion.
    • Summary and comment:
      • The histologic findings are compatible with the pathologic diagnosis of amyotrophic lateral sclerosis (ALS), also known as motor neuron disease. The upper motor neuron pathology is mild and the lower motor neuron pathology is moderate. Neocortical TDP-43 pathology is present, of the type seen in frontotemporal lobar degeneration with TDP-43 proteinopathy (FTLD TDP), type B (Am J Pathol 2006;169:1343, Acta Neuropathol 2006;112:539, Acta Neuropathol 2011;122:111). TDP-43 proteinopathy may have different presentations, which vary across or even within specific pathology patterns. The pattern now recognized as FTLD TDP, type B may present with pure ALS, pure dementia or a combination of the 2 syndromes, as in this case (Acta Neuropathol 2006;112:539, Acta Neuropathol 2011;122:111). The patient tested negative for C9orf72 hexanucleotide expansion and there is no evidence of the distinctive p62 positive inclusions seen with this variant (Neuron 2011;72:245, Neuron 2011;72:257).
      • This case was discussed at neurodegenerative disease clinicopathologic correlation conference and the above diagnoses were agreed upon.
    • Microscopic sections (all from left side except as noted):
      • A1 middle frontal gyrus (BA8-9)
      • A2 inferior frontal gyrus (BA44)
      • A3 superior temporal gyrus (BA22)
      • A4 inferior parietal lobule (BA39)
      • A5 anterior (BA24) and posterior (BA23) cingulate, posterior notched
      • A6 motor (BA4) and somatosensory (BA3) cortex, sensory notched
      • A7 amygdala and entorhinal cortex
      • A8 anterior hippocampus and entorhinal cortex
      • A9 bilateral posterior hippocampi, right notched
      • A10 striatum
      • A11 basal ganglia / nucleus basalis of Meynert
      • A12 bilateral mammillary bodies
      • A13 thalamus with subthalamic nucleus
      • A14 occipital (visual) cortex (BA17)
      • A15 cerebellum with dentate nucleus / vermis
      • A16 left midbrain
      • A17 pons with locus coeruleus
      • A18 medulla with inferior olives
      • A19 right middle frontal gyrus (BA8-9)
      • A20 right inferior frontal gyrus (BA44)
      • A21 right superior temporal gyrus (BA22)
      • A22 right inferior parietal lobule (BA39)
      • A23 right midbrain
Differential diagnosis
  • Multifocal motor neuropathy:
    • Immune mediated pathology of peripheral nerves rather than neurodegenerative disease
    • Diagnosed clinically; nerve biopsy not often done
    • Characterized by asymmetric weakness with subacute onset
    • Lower motor neuron signs only
    • Motor nerve conduction studies show conduction block, with normal sensory conduction
    • Serology: may be associated with anti-GM1 antibodies
  • Compressive radiculomyelopathy:
    • Spondylolysis with spinal cord and nerve root compression
    • Lower motor neuron signs corresponding to level of abnormality
    • Radiology is key to diagnosis
  • Inclusion body myositis:
    • Disproportionate finger flexor weakness
    • Electromyography shows myopathic signs
    • Muscle biopsy classically shows variable inflammation, chronic myopathic features (e.g., regenerating fibers, endomysial fibrosis) and rimmed vacuoles (low sensitivity)
  • Myasthenia gravis:
    • Can present as bulbar syndrome with dysphagia and dysarthria, similar to ALS
    • Clinical features favoring myasthenia gravis:
      • Absence of either upper or lower motor neuron features
      • Ocular findings
      • Diurnal variation in symptom severity
    • Serology: acetylcholine receptor binding antibodies, muscle specific tyrosine kinase antibodies
    • Electromyography: cranial muscles show evidence if denervation / reinnervation in bulbar onset ALS but not in myasthenia gravis
  • X linked bulbospinal neuronopathy:
    • Usually progresses more slowly than ALS
    • Genetic testing reveals CAG repeat expansion in androgen receptor gene
  • Reference: Goldman: Cecil Textbook of Medicine, 21st Edition, 1999
Board review style question #1

The image above shows inclusions in the hypoglossal nucleus in an autopsy brain from patient who succumbed to sporadic amyotrophic lateral sclerosis (ALS). Which of the following immunostains is represented in the image?

  1. 3 repeat tau
  2. 4 repeat tau
  3. Alpha synuclein
  4. Cystatin C
  5. TDP-43
Board review style answer #1
E. TDP-43. This protein is thought to be central to pathogenesis in sporadic ALS and skein-like inclusions found in motor neurons are positive. Neuronal inclusions positive for 3 repeat tau are found in Pick disease and 4 repeat tau positive inclusions are found in corticobasal degeneration and progressive supranuclear palsy. Lewy bodies and glial cytoplasmic inclusions of multiple system atrophy are positive for alpha synuclein. Cystatin C is a component of Bunina bodies.

Comment Here

Reference: Amyotrophic lateral sclerosis
Board review style question #2
In a case of amyotrophic lateral sclerosis (ALS), neuronal loss and gliosis would be most severe in which of the following CNS regions?

  1. Anterior horn of the spinal cord
  2. Basal ganglia
  3. Dentate nucleus of the cerebellum
  4. Posterior horn of the spinal cord
  5. Somatosensory cortex
Board review style answer #2
A. Anterior horn of the spinal cord. This region contains cell bodies of efferent lower motor neurons and is severely affected by neuronal loss and gliosis in ALS. The posterior horn of the spinal cord carries afferent sensory neurons and is usually preserved in ALS. The primary motor cortex (not the somatosensory cortex) also shows prominent neuronal loss and gliosis in ALS. The basal ganglia often show prominent neuronal loss and gliosis in multiple system atrophy, and sometimes in corticobasal degeneration. The dentate nucleus of the cerebellum shows severe neuronal loss and gliosis (grumose degeneration) in progressive supranuclear palsy.

Comment Here

Reference: Amyotrophic lateral sclerosis

Arnold-Chiari malformation
Definition / general
  • Chiari malformations, types I-IV, refer to a spectrum of congenital hindbrain abnormalities affecting the structural relationships between the brainstem, cerebellum, upper cervical cord and bony cranial base
  • Hans Chiari, an Austrian pathologist, classified Chiari malformations into types I through III in 1891
  • Chiari's German colleague, Julius Arnold, made additional contributions to the definition of Chiari II malformation
  • Therefore the type II malformation is commonly referred to as the Arnold-Chiari malformation
  • Later, other investigators added a fourth (Chiari IV) malformation
Essential features
  • Scale of severity is rated I - IV with IV being the most severe
  • Chiari type I malformation: elongation of one or both cerebellar tonsils, which protrude through the foramen magnum of the spinal canal and may become sclerotic; there may be associated syringomyelia or syringobulbia
  • Chiari type II malformation: most common cause of congenital hydrocephalus
    • Cerebellar tonsils extend far down into the spinal canal, accompanied by an elongated fourth ventricle
    • There is a lumbar spina bifida with a meningomyelocele
    • Hydrocephalus is associated with enlargement of the lateral and third ventricle, a paper thin tentorium and falx cerebri
    • The outer surface of the cerebral hemispheres is flattened and the cortex is thinned
    • The vault of the skull is large or enormous, the fontanelles are very wide and the posterior fossa is small
    • Lower limb muscles show varying degrees of denervation atrophy
  • Chiari type IIII malformation: further herniation of the cerebellum below the foramen magnum forming an encephalocele, in addition to spina bifida, syringomyelia and hydrocephalus
  • Chiari type IV malformation: hypoplasia or aplasia of the cerebellum in addition to other features
  • Conditions associated with Chiari malformation include hydrocephalus, syringomyelia, spinal curvature, tethered spinal cord syndrome and connective tissue disorders such as Ehlers-Danlos syndrome
Diagrams / tables

Contributed by Erdener Özer, M.D., Ph.D.

Types of Chiari malformations

Terminology
  • Chiari or Chiari’s malformation
Epidemiology
  • There are no population based studies on the incidence or prevalence of Chiari malformations
  • From clinical series, prevalence has been estimated at 0.1 to 0.5%
  • However with routine use of magnetic resonance imaging, Chiari malformation is discovered with increasing frequency
  • Types III and IV are very rare (J Genet Couns 2003;12:297)
Pathophysiology
  • Symptoms of Chiari type I malformation develop due to compression of medulla and upper spinal cord, compression of cerebellum or disruption of cerebrospinal fluid flow through the foramen magnum
  • The pathophysiology of Chiari type II malformation is more complex; although compressive mechanisms likely play a role, additional mechanisms may be functional
Etiology
  • Chiari type I:
    • Based on analysis of familial aggregation, a genetic basis has been suggested due to alterations in chromosomes 9 and 15
    • It may be a disorder of para-axial mesoderm, which subsequently results in formation of a small posterior fossa
    • Development of the cerebellum within this small compartment results in herniation of the cerebellar tonsils (Am J Med Genet A 2006;140:2776, J Genet Couns 2003;12:297)
  • Chiari type II:
    • Theories must take into account its invariable association with myelomeningocele
    • The "CSF loss" theory claims that escape of CSF fluid through the open placode in myelomeningocele results in an inadequate stimulus for mesenchymal condensation at the skull base
    • Disordered and inadequate growth of the posterior fossa results in downward herniation of brainstem
Clinical features
  • Chiari’s type I malformation is usually seen in older children and adults; type II is seen in infants and younger children
  • Some patients are asymptomatic and the malformation does not interfere with activities of daily living
  • Symptomatic patients often present with headaches, fatigue, muscle weakness, difficulty swallowing, vomiting, dizziness, nausea, tinnitus, impaired coordination, neck pain and speech problems
Diagnosis
  • Diagnosis requires patient history, neurological examination and medical imaging
  • Magnetic resonance imaging is considered the best imaging modality for Chiari malformation
  • Diagnosis of Chiari type II malformation can be made prenatally through ultrasound (J Neurosurg Pediatr 2015;16:159)
Radiology description
  • Neuroradiological investigation is used to rule out intracranial conditions responsible for tonsillar herniation, to evaluate the cerebellar tonsil position, the crowding of neural structures within the posterior cranial fossa and their impact on the foramen magnum (J Neurosurg Pediatr 2015;16:159)
Radiology images

Images hosted on other servers:
Missing Image

Sagittal FLAIR MRI scan

Treatment
  • Vitamin A and folic acid supplementation in pregnancy reduces the risk of its development
  • In symptomatic cases, medications may ease certain symptoms such as pain
  • Surgical shunting and decompression of the foramen magnum are the only treatments which reduce the impact of hydrocephalus and the progression of damage to the central nervous system
Clinical images

Contributed by Erdener Özer, M.D., Ph.D.

Chiari type II malformation

Differential diagnosis
  • In pseudo-Chiari malformation, leaking of cerebrospinal fluid may cause displacement of the cerebellar tonsils and resemble Chiari’s type I malformation

Aspergillosis
Definition / general
  • Cerebral aspergillosis is an aggressive, rare form of invasive aspergillosis characterized by brain parenchymal or meningeal invasion by Aspergillus species
Essential features
  • Invasion of either brain parenchyma or meninges by septate, branching fungal hyphae of Aspergillus spp. with associated inflammatory reaction
Terminology
  • Neuroaspergillosis
  • CNS aspergillosis
ICD coding
  • ICD-10: B44.9 - aspergillosis, unspecified
Epidemiology
  • Cerebral aspergillosis is a rare disease with poor prognosis and a high fatality rate
  • Aspergillus fumigatus is the most common organism; however, Aspergillus flavus, Aspergillus niger and Aspergillus terreus are also known to cause infection of the central nervous system
  • Aspergillus fumigatus mostly causes infection in immunocompromised individuals, while Aspergillus flavus, Aspergillus niger and Aspergillus terreus are mostly seen in immunocompetent hosts (Infect Dis Now 2021;51:654)
  • Incidence, morbidity and mortality are on the rise, especially in patients with malignancies being treated with immunosuppressive drugs and in patients undergoing organ transplantation (Drugs 2007;67:1567)
  • Post COVID-19 cases of cerebral aspergillosis have also been noted (Ann Med Surg (Lond) 2022;80:104122)
Sites
Pathophysiology
  • Hematogenous spread is more common in immunocompromised individuals
  • Direct spread from infection of adjacent areas (paranasal sinuses, ear) can also occur
  • Aspergillus spp. produce blood brain barrier altering toxins that ultimately damage neurons and astrocytes (Mycotoxin Res 2018;34:257)
  • Aspergillus spp. are angiotropic and have the propensity to cause angioinvasion, leading to thrombosis, infarcts, aneurysms and meningitis (Front Med (Lausanne) 2017;4:113)
  • Immunocompetent hosts may develop granulomas, brain abscesses and meningitis
Etiology
  • Mostly affects immunocompromised individuals (transplant recipients)
  • Patients with neutropenia associated with acute leukemia and its therapy (Med Mycol Case Rep 2019;27:17)
  • Patients on prolonged corticosteroid therapy
  • Can occur due to cranial trauma or neurosurgery in immunocompetent hosts
Clinical features
  • Clinical features vary depending upon site and extent of infection
  • Patients may present with fever, headache, focal neurological deficit, cranial nerve deficit, features of meningitis or intracranial bleed
  • CNS infection is often associated with hemorrhage or infarcts (Medicine (Baltimore) 2020;99:e22911)
  • Disease can be slowly progressive and with symptoms present for weeks to months
  • In immunocompetent hosts, the presentation in brain parenchyma is often a mass lesion (aspergilloma) (Medicine (Baltimore) 2020;99:e22911)
Diagnosis
Laboratory
Radiology description
  • CNS aspergillosis can be divided into 2 forms: parenchymal lesions and meningeal lesions
  • Neuroimaging usually shows brain abscesses, meningitis, cerebral infarcts, aneurysms and intradural abscesses
  • CT findings are nonspecific (Rofo 2018;190:967)
  • Most common MRI findings are T2W hyperintense abscess wall with onion layer-like hypointense zone; ring enhancement is noted after contrast media application (PLoS One 2016;11:e0152475)
Radiology images

Images hosted on other servers:

CT / MRI scan of brain for neuroaspergillosis

Prognostic factors
Case reports
Treatment
  • Antifungal therapy with voriconazole (triazoles) as first line therapy (Stud Mycol 2021;100:100115)
  • In cases of azole resistant infection, monotherapy (or a combination) with amphotericin B or echinocandin can be given (Mycoses 2018;61:656)
  • In cases of CNS aspergilloma, surgical resection with subsequent antifungal therapy can be sought (Surg Neurol Int 2020;11:211)
Gross description
  • Usually, small biopsies are received
  • Aspergilloma has an appearance of a necrotic mass
  • Autopsy shows areas of hemorrhage and infarction in angioinvasive aspergillosis
Frozen section description
  • Mixed inflammation, associated Aspergillus hyphae which are dichotomously branching (acute angle branching) and septate
  • Associated granulomatous response may be appreciated
  • Usually only performed if necessary to rule out neoplasm
Microscopic (histologic) description
  • Characteristic Aspergillus species comprising septate, branching hyphae (at acute angles) are seen
  • Surrounding tissue usually shows acute on chronic inflammation
  • Granulomatous response can be seen with adjacent areas of necrosis and an associated foreign body type of giant cell response
  • Reference: Clin Microbiol Rev 1998;11:341
Microscopic (histologic) images

Contributed by Khurram Minhas, M.B.B.S.
Necrotizing granulomatous inflammation Necrotizing granulomatous inflammation

Necrotizing granulomatous inflammation

GMS stain

GMS stain

PASD stain

PASD stain

Virtual slides

Images hosted on other servers:

55 year old man with brain lesion

Positive stains
Molecular / cytogenetics description
  • ISH using the probe for Aspergillus species and a low viscosity hybridization buffer solution (Intern Med 1999;38:563)
Sample pathology report
  • Brain mass, biopsy:
    • Granulomatous inflammation with associated fungal infection, morphologically consistent with Aspergillus spp. (see comment)
    • Comment: The biopsy reveals brain tissue showing numerous epithelioid granulomata with multinucleated giant cells. There are numerous septate, dichotomous, fungal hyphae, branching at acute angles. No angioinvasion is seen. PASD stain highlights the fungal hyphae.
Differential diagnosis
Board review style question #1
A 64 year old diabetic patient presented with altered sensorium and seizures. The cerebrospinal fluid (CSF) examination shows increased protein, decreased glucose and increased CSF galactomannan. These CSF findings are suggestive of an infection by which of the following?

  1. Aspergillus species
  2. Mucor species
  3. Neisseria meningitides
  4. Viral infection
Board review style answer #1
A. Aspergillus species. CSF galactomannan is raised in CSF and sometimes in serum in the patients of neuroaspergillosis.

Comment Here

Reference: Aspergillosis
Board review style question #2

A 40 year old woman, with a known case of systemic lupus erythematosus, presented with high grade fever and severe headache. Brain MRI showed a heterogenous appearing infiltrative lesion involving bilateral frontal lobes, with avid postcontrast enhancement. A biopsy for histopathological evaluation was performed and the microscopic features are shown above. Which of the following is the correct diagnosis?

  1. Cerebral aspergillosis
  2. Fungal meningitis secondary to Candida species
  3. Mucormycosis
  4. Tuberculous meningitis
  5. Viral meningitis
Board review style answer #2
A. Cerebral aspergillosis. The presence of septate fungal hyphae branching at acute angles is suggestive of cerebral aspergillosis.

Comment Here

Reference: Aspergillosis

CMV infections
Definition / general
  • Cytomegalovirus (CMV) is the largest of the human herpes viruses and may cause a variety of clinical manifestations ranging from asymptomatic to fatal
  • Severe, often fatal central (CNS) and peripheral nervous system infections are usually seen in patients with compromised immune systems or in maternal fetal infections in utero (Ther Adv Neurol Disord 2012;5:279)
Epidemiology
  • The vast majority of CMV CNS infections occur in patients with severe suppression of cell mediated immunity; infections are only rarely reported in apparently immunocompetent patients
  • Prior to combined antiretroviral therapy, CMV nervous system infections usually occurred in AIDS patients; currently, they are seen as complications of organ transplantation, especially bone marrow and hematopoietic stem cell (Rev Inst Med Trop Sao Paulo 2010;52:305, PLoS One 2013;8:e77805)
  • Cytomegalovirus is also the most common pathogen causing congenital fetal infection, resulting in severe neurological damage
Sites
  • May infect all levels of the nervous system but classically, CMV encephalitis causes damage to subependymal regions of the cerebrum, cerebellum and brainstem
  • Spinal cord and nerve root infection is also common in immunocompromised hosts
Pathophysiology
  • Cytomegalovirus is easily transmitted from person to person via a variety of secretions
  • Maternal fetal infection usually occurs transplacentally but may also be transmitted via general secretions or breast milk
  • CMV may also be transmitted by transplanted organs
  • In the general population, asymptomatic infection usually occurs early in life and remains latent in immunocompetent individuals
  • Impaired cell mediated immunity appears to be required for viral infection of the mature nervous system (Curr Immunol Rev 2010;6:38, PLoS Pathog 2012;8:e1002849)
Etiology
  • Lesions are caused by lytic destruction of infected cells within the brain, spinal cord or peripheral nerve roots
Clinical features
  • Adult cytomegaloviral encephalitis usually produces no unique symptomatology, beyond that of clinical encephalitis
  • Similarly, spinal cord and nerve root infections demonstrate no symptomatology specific to this virus
  • Congenital cytomegaloviral infection often results in a constellation of abnormalities referred to as cytomegalic inclusion disease, including microcephaly, hearing loss, intracranial calcifications, hepatosplenomegaly and thrombocytopenic purpura
Diagnosis
  • Due to the absence of specific clinical symptomatology, diagnosis usually requires polymerase chain reaction amplification of cytomegaloviral genome from cerebrospinal fluid or other specimens (Emerg Infect Dis 2013;19:1470)
Laboratory
  • Cytomegaloviral infection of the nervous system generally manifests with typical CSF findings of increased protein and pleocytosis
  • Features specific to CMV have not been identified
Radiology description
  • Cytomegaloviral encephalitis tends to localize to the ependymal and subependymal regions
  • Although MRI may demonstrate signal changes along the lateral ventricles, septum pellucidum, corpus callosum and fornices, these changes are usually quite subtle on conventional T1 and T2 weighted images, even after contrast administration
  • Recent studies indicate that diffusion weighted imaging abnormalities along the ependymal lining is a more sensitive indication of cytomegaloviral encephalitis
Radiology images

Images hosted on other servers:

Ventricular dilatation

Curvilinear high signal intensities

Prognostic factors
  • Specific prognostic factors have not been extensively elucidated but recent studies indicate that T cell depleted hematopoietic stem cell transplantation appears to predispose to central nervous system infection (BMC Infect Dis 2012;12:238)
Case reports
Treatment
Gross description
  • At autopsy, coronal sectioning through the thalamic nuclei of an adult with AIDS and central nervous system Cytomegalovirus infection demonstrates necrotizing ependymitis with fibrinous exudate lining the ventricular system
  • Hydrocephalus can also be appreciated (especially in the temporal horns of the lateral ventricles), which is secondary to aqueductal stenosis consequent to similar inflammatory necrosis within the ependymal lining (shown in the second gross image)
  • Congenital cytomegalic inclusion disease secondary to transplacental fetal infection manifests as microencephaly with hydrocephalus (gross image 3 and 4)
Gross images

Contributed by Mark Cohen, M.D.

Coronal section

Inflammatory necrosis

Microencephaly with hydrocephalus

Microscopic (histologic) description
  • Although immunohistochemical staining for Cytomegalovirus may be used as a confirmational assay, the distinctive nuclear and cytoplasmic cytopathic changes on routinely stained sections are diagnostic
Microscopic (histologic) images

Contributed by Mark Cohen, M.D.

Low and high power, H&E

Differential diagnosis
  • As in other organs, the differential diagnosis includes a variety of viral infections, though none demonstrate the characteristic cytological cytomegaly with both cytoplasmic and nuclear viral inclusions
  • In congenital Cytomegalovirus inclusion disease, gross differential diagnosis includes a variety of other encephaloclastic fetal diseases, most notably intrauterine ischemia

CNS-gross (pending)
[Pending]

Carbon monoxide injury
Definition / general
Sites
  • Affects globus pallidus and white matter
Pathophysiology / etiology
  • Carbon monoxide binds to hemoglobin, forming carboxyhemoglobin
  • This causes a decrease in the oxygen carrying capacity of the blood, leading to tissue hypoxia
  • Sources of exposure to carbon monoxide include faulty or inadequately ventilated heating sources and engine exhaust
  • Exposure is most commonly accidental or due to a suicide attempt
Clinical features
  • Acute: headache, myalgia, dizziness, psychological impairment
  • Chronic: delayed neuropsychiatric syndrome ranging from subtle personality changes or mild cognitive deficit to severe dementia, psychosis, parkinsonism
Diagnosis / laboratory
  • Measurement of carboxyhemoglobin levels
Radiology description
  • High signal intensity in globus pallidus bilaterally on T2 weighted MR images
  • Restricted diffusion in white matter (diffusion weighted imaging [DWI], a form of MR imaging)
Radiology images

Images hosted on other servers:

16 hours after injury

Case reports
Treatment
  • Administration of hyperbaric oxygen
Gross description
Gross images

Contributed by Kymberly A. Gyure, M.D.

Carbon monoxide injury

Microscopic (histologic) description
  • Foci of ischemic or hemorrhagic necrosis in globus pallidus (Wikipedia: Globus Pallidus)
  • Perivascular foci of demyelination in deep white matter with sparing of arcuate fibers
Differential diagnosis

Cerebral amyloid angiopathy
Definition / general
  • Vasculopathy characterized by the accumulation of beta amyloid protein within small to medium sized blood vessels of the brain and leptomeninges
  • Renders affected blood vessels fragile, leading to intraparenchymal hemorrhage
  • Distinct from other systemic forms of amyloidosis
Essential features
  • Progressive disease of older individuals, predisposing them to intracerebral hemorrhage
  • May coincide with or be independent of Alzheimer disease neuropathologic change (AD NC) or other small to medium vessel cerebrovascular disease (e.g., arteriolosclerosis)
  • May present as noninflammatory or inflammatory subtypes
Terminology
  • Congophilic angiopathy
  • Dysphoric angiopathy
ICD coding
  • ICD-10: I68.0 - cerebral amyloid angiopathy
  • ICD-11: 8B22.3 - isolated cerebral amyloid angiopathy
Epidemiology
  • Sporadic disease (J Neurol Sci 2021;424:117425):
    • More common in elderly patients, particularly in the seventh and eighth decades of life
    • May coincide with (or be independent of) arteriolosclerotic cerebrovascular disease or Alzheimer disease neuropathologic change
  • A large retrospective autopsy series detected sporadic disease in 73% of all subjects and 98% of patients with Alzheimer disease (J Neurol 2008;255:70)
  • APOE mutations and cerebral amyloid angiopathy (CAA) (Lancet Neurol 2021;20:68):
    • Mutations in the ε4 allele are associated with sporadic onset CAA and increased severity of disease
    • Mutations in the ε2 allele may be protective
  • Familial / hereditary disease (Arch Neurol 2010;67:987):
    • Hereditary cerebral hemorrhage with amyloidosis - Dutch type (HCHWA D)
      • Earlier onset (sixth decade) CAA symptoms with cerebral hemorrhages
      • Autosomal dominant mutation of APP gene (E693Q) on chromosome 21
    • Hereditary cerebral hemorrhage with amyloidosis - Icelandic type (HCHWA I)
      • Cerebral hemorrhages in young to middle aged adults
      • Mutations in cystatin C / gamma trace gene
      • Also known as hereditary cystatin C amyloid angiopathy (HCCAA)
      • Associated with mural deposition of gamma trace protein in arterioles
    • Familial British and Danish forms
      • Mutations in BR12 gene (chromosome 13)
      • Less consistently associated with intraparenchymal hemorrhage
    • Flemish form: APP gene mutations at codon 692 (Neurobiol Dis 1998;5:281)
    • APP gene mutations at codon 694 (Alzheimer disease neuropathologic change with prominent CAA) (Ann Neurol 2001;49:697)
    • Finnish type: autosomal dominant mutations in GSN gene with gelsolin deposition in vessels (Crit Rev Biochem Mol Biol 2012;47:282)
    • Hereditary transthyretin amyloidosis (ATTR): systemic amyloidosis that can affect the central nervous system (Acta Neuropathol 2023;145:113)
    • Additional familial cohorts have been described (Int J Mol Sci 2020;21:3435, Arch Neurol 2010;67:987)
Sites
  • Central nervous system: small to medium sized arteries and veins of the cortex and leptomeninges
Pathophysiology
  • Progressive deposition of beta amyloid protein in the arteries and veins of the cortex and leptomeninges
  • Leads to weakness / fragility of affected vessel walls, predisposing individuals to intracerebral (lobar) hemorrhages (J Neurol Sci 2021;424:117425)
Etiology
  • Deposition of beta amyloid in arterial media and adventitia
  • Genetic factors (see Epidemiology) may predispose to the development of CAA, including more severe familial forms (Brain Pathol 2002;12:343)
Clinical features
  • Intraparenchymal (lobar) hemorrhage (Neurology 1984;34:730)
  • Additionally, there have been associations described with (Neurology 1993;43:2073)
    • Cognitive decline
    • Transient focal neurologic events
    • Cortical atrophy
    • Small cortical infarcts
    • Leukoencephalopathy
  • If coexisting Alzheimer disease neuropathologic change, CAA may act synergistically in its effects on clinical presentation, including memory disturbances (Ann Neurol 2011;69:320)
  • Inflammatory CAA often presents more acutely with focal / multifocal neurologic deficits or seizure (J Neurol Sci 2021;424:117425)
    • Patients tend to be younger than in noninflammatory CAA
    • Includes CAA related inflammation (CAAri) and Aβ related angiitis (ABRA)
  • Subarachnoid hemorrhage and subpial hemosiderosis have been described in patients with leptomeningeal disease (AJNR Am J Neuroradiol 2008;29:184)
  • Leukoencephalopathy may result from chronic hypoperfusion (Front Aging Neurosci 2022;14:1019088)
Diagnosis
  • Combination of clinical findings, radiographic criteria (Boston criteria) and histology (biopsy or autopsy)
  • Boston criteria: radiographic criteria for CAA diagnosis (Stroke 2018;49:491, J Clin Neurol 2011;7:1)
    • Definite CAA
      • Full postmortem examination demonstrating
        • Lobar, cortical or cortical - subcortical hemorrhage
        • Severe CAA with vasculopathy
        • Absence of another diagnostic lesion
    • Probably CAA with supporting pathology
      • Clinical data and pathologic tissue (evacuated hematoma or cortical biopsy) demonstrating
        • Lobar, cortical or cortical - subcortical hemorrhage including intracerebral (ICH), cerebral microbleeds (CMB) or cortical superficial siderosis (cSS)
        • Some degree of CAA in tissue specimen
        • Absence of another diagnostic lesion
    • Probable CAA
      • Clinical data and MRI or CT demonstrating
        • Multiple hemorrhages (intracerebral, cerebral microbleeds) restricted to lobar, cortical or cortical - subcortical regions (cerebellar hemorrhage allowed) or single lobar, cortical or cortical - subcortical hemorrhage and cortical superficial siderosis (focal or disseminated)
        • Age: ≥ 55 years
        • Absence of other causes of hemorrhage
    • Possible CAA
      • Clinical data and MRI or CT demonstrating
        • Single lobar, cortical or cortical - subcortical intracerebral, cerebral microbleeds or cortical superficial siderosis (focal or disseminated)
        • Age: ≥ 55 years
        • Absence of other cause of hemorrhage
    • Other causes of hemorrhage (differential diagnosis of lobar hemorrhages) to exclude for either probable or possible CAA:
      • Antecedent head trauma, hemorrhagic transformation of an ischemic stroke, arteriovenous malformation, hemorrhagic tumor, warfarin therapy with international normalization ratio (INR) > 3, vasculitis
Laboratory
Radiology description
Radiology images

Images hosted on other servers:

Intracerebral hemorrhage and hemosiderosis

Arterial wall enhancement

Prognostic factors
Case reports
Treatment
  • Noninflammatory CAA (Int J Stroke 2021;16:356, Int J Mol Sci 2021;22:3869)
    • Blood pressure control to prevent intracerebral hemorrhage
    • Evacuation of hemorrhage can be performed in some cases of lobar hemorrhage to relieve intracranial pressure
    • Experimental therapies currently under review
  • Inflammatory CAA (J Neurol Sci 2021;424:117425):
    • Not fully established but includes immunosuppressive therapy such as corticosteroids
Gross description
Gross images

Images hosted on other servers:

Temporoparietal lobar hemorrhage

Microscopic (histologic) description
  • Small to medium sized cortical and leptomeningeal vessels with mural thickening and deposition of homogenous, acellular, eosinophilic material in vessel walls (Ann Neurol 1991;30:637, Handb Clin Neurol 2017;145:79, J Clin Neurol 2011;7:1)
  • Noninflammatory CAA with 2 historical subtypes (J Neurol Sci 2021;424:117425)
    • CAA type 1 (microangiopathy, drusige Endartung, plaque-like angiopathy of Scholz)
      • Affects cortical parenchymal capillaries
      • Perivascular parenchymal involvement by amyloid
    • CAA type 2 (Congophilic angiopathy of Pantelakis)
      • Affects leptomeningeal or superficial cortical perforating arteries
      • Advanced disease may result in double barrel appearance due to the separation of intima from media (Int J Mol Sci 2021;22:3869)
    • Grading (J Clin Neurol 2011;7:1):
      • Grading severity of the disease is made on histologic grounds, though ancillary staining techniques may aid in the identification of affected vessels and characterize source of injury
      • Method 1 (proposed by Olichney et al.) (Arch Neurol 1995;52:702)
        • 0: no amyloid beta positive blood vessels
        • 1: scattered amyloid beta positivity in either leptomeningeal or intracortical blood vessels
        • 2: strong, circumferential amyloid beta positive in either some leptomeningeal or intracortical blood vessels
        • 3: widespread, strong, circumferential amyloid beta positivity in leptomeningeal and intracortical blood vessels
        • 4: same as 3 but with additional leakage into surrounding parenchyma
      • Method 2 (proposed by Vonsattel et al.) (Ann Neurol 1991;30:637)
        • Mild: amyloid restricted to tunica media without significant destruction of smooth muscle cells
        • Moderate: tunica media is replaced by amyloid and thickened
        • Severe: extensive amyloid deposition with focal wall fragmentation or double barreling of the vessel wall, microaneurysms, fibrinoid necrosis or leakage of blood
  • Inflammatory CAA (Stroke 2015;46:e210)
    • CAAri
      • Perivascular inflammation that does not involve the vessel wall
      • Lymphocytes may cuff the vessel and be associated with foreign body giant cells lying internal and external to the deposits
    • Amyloid beta related angiitis
      • Vasculitis with inflammation involving the vessel wall and often associated granuloma formation
      • May result in complete destruction of vessel wall in severe cases
      • Inflammatory infiltrates are variably composed of lymphocytes, plasma cells, epithelioid histocytes and occasionally multinucleated giant cells
Microscopic (histologic) images

Contributed by Jared T. Ahrendsen, M.D., Ph.D. and Nicolas Kostelecky, M.D.

Small and medium sized blood vessels

Hemorrhagic cerebral amyloid

Cortical arterioles

Beta amyloid IHC


Congophilic leptomeningeal vessels

Bright green fluorescence of cortical arterioles

Virtual slides

Images hosted on other servers:

Brain: amyloid beta related angiitis

Positive stains
  • Amyloid beta: also highlights parenchymal amyloid plaques, if present; highly sensitive and reliable marker
  • Congo red: apple green birefringence under polarized light; can give equivocal or unreliable results
  • Thioflavin S: strong green fluorescence
  • Reference: Neuropathol Appl Neurobiol 2003;29:106
Sample pathology report
  • Brain and leptomeninges, left parieto-occipital lobe, biopsy:
    • Cerebral amyloid angiopathy (see comment)
    • Comment: Amyloid deposition is present in small and medium sized blood vessels of the cortex and leptomeninges. There is no granuloma formation or significant inflammation. Additionally, occasional diffuse and neuritic type amyloid plaques are present in the cortical parenchyma. While not diagnostic, these changes could indicate Alzheimer disease neuropathologic change. Clinical correlation is recommended.
Differential diagnosis
  • Hypertensive cerebrovascular disease (Handb Clin Neurol 2017;145:79):
    • Hypertensive hemorrhagic strokes generally occur in deep seated brain regions (thalamus, basal ganglia, pons, cerebellum)
    • Thickened blood vessels are more commonly found in white matter and deep gray matter nuclei (rather than superficial cortex and leptomeninges) and are negative for beta amyloid deposition
  • Primary angiitis of the CNS (PACNS) (J Neurol Sci 2021;424:117425, Neurol Sci 2020;41:3135):
    • Clinical syndrome of gradual onset headache, altered mental status and focal neurologic deficits
    • MRI shows bilateral, multifocal supratentorial lesions with restricted diffusion
    • Shows overlapping clinical features with inflammatory CAA
    • Histology shows necrotizing vasculitis without beta amyloid deposition in vessels
Board review style question #1

A 67 year old man in otherwise good health presents with a catastrophic lobar hemorrhage. Microscopic sections at brain autopsy show thickened vessel walls with a double barrel appearance, as shown in the photomicrograph above. What stain will confirm your histologic suspicion?

  1. Alpha synuclein
  2. Beta amyloid
  3. GFAP
  4. Luxol fast blue
  5. Tau
Board review style answer #1
B. Beta amyloid will be positive in small to medium caliber blood vessels in the cerebral cortex and leptomeninges in patients with cerebral amyloid angiopathy (CAA). Alpha synuclein can be helpful to highlight Lewy bodies. GFAP is a marker of astrocytes and glial tumors. Luxol fast blue highlights myelin and can be helpful to identify areas with demyelination and tau can be helpful to highlight neurofibrillary tangles. Tau can be helpful to highlight neurofibrillary tangles.

Comment Here

Reference: Cerebral amyloid angiopathy
Board review style question #2
Postmortem examination of a patient who died of intracerebral hemorrhage reveals cortical vessels with associated vessels. Beta amyloid immunohistochemistry establishes this to be a case of cerebral amyloid angiopathy (CAA). How might the inflammatory subtypes of CAA (including cerebral amyloid angiopathy related inflammation [CAAri] and amyloid beta related angiitis [ABRA]) be distinguished?

  1. ABRA and CAAri are indistinguishable on histopathologic findings alone
  2. ABRA with fibrinoid vessel necrosis; CAAri with perivascular lymphocytic cuffing with extravascular foreign body giant cell reaction
  3. ABRA with noncaseating granulomatous inflammation; CAAri with fibrinoid vessel necrosis
  4. ABRA with predilection for optic chiasm; CAAri with a predilection for cerebellum
Board review style answer #2
B. ABRA with fibrinoid vessel necrosis; CAAri with perivascular lymphocytic cuffing with extravascular foreign body giant cell reaction. Answer A is incorrect, as these subtypes are distinguishable by the following features: fibrinoid necrosis and granuloma formation (ABRA) and perivascular inflammation and foreign body giant cell reaction without vessel involvement by inflammation (CAAri). Answer C is incorrect because the inflammation does not involve the vessel wall in CAAri. Answer D is incorrect because in ABRA, there is a predilection for the posterior aspect of the brain. The optic chiasm is more commonly affected in neurosarcoidosis. In CAAri, there is a predilection for the posterior aspect of the brain.

Comment Here

Reference: Cerebral amyloid angiopathy

Chronic traumatic encephalopathy (pending)
Table of Contents
Definition / general
Definition / general
[Pending]

Congenital hydrocephalus (pending)

Corticobasal degeneration (pending)
Table of Contents
Definition / general
Definition / general
[Pending]

Dandy-Walker Malformation
Definition / general
  • Dandy-Walker syndrome (DWS) is a congenital brain malformation involving the cerebellum
  • Note: Dandy-Walker does not represent a single entity, as there are three identified types of Dandy-Walker complexes: DWS malformation, DWS mega cisterna magna and DWS variant
Essential features
  • Dandy-Walker malformation (DWM) has six features (Childs Nerv Syst 2011;27:1665)
    1. Large, median posterior fossa cyst communicating to the fourth ventricle
    2. Absence of the lower portion of the vermis at different degrees
    3. Hypoplasia, anterior rotation and upward displacement of the remnant of the vermis
    4. Antero-lateral displacement of normal or hypoplastic cerebellar hemispheres
    5. Large bossing posterior fossa with elevation of the torcular
    6. Absence or flattening of the angle of the fastigium (highest point of fourth ventricle)

  • All six features are not found in each case
  • The essential features are: enlargement of the posterior fossa, cystic dilatation of the fourth ventricle and agenesis of the vermis
  • Hydrocephalus is present in 80% of the cases and should not be considered a specific component of the malformation
  • Other CNS malformations can be observed like occipital encephaloceles, corpus callosum agenesis, schizencephaly and glial heterotopias

  • Dandy–Walker variant (DWV): cystic posterior mass with variable hypoplasia of the cerebella vermis and no enlargement of the posterior fossa (
  • Dandy Walter mega-cisterna magna: enlarged cisterna magna with normal cerebellar vermis and fourth ventricle
Terminology
  • Dandy-Walker malformation, Dandy-Walker syndrome
Epidemiology
Pathophysiology
  • DWM may be caused by many conditions affecting the brain development in an early stage
  • The type of insult is less important than the timing and duration of exposure to the noxious agent (J Child Neurol 2011;26:1483)
Etiology
Dandy-Walker malformation can occur due to:
  • Mendelian conditions: Walker-Warburg syndrome, Mohr syndrome, Meckel-Gruber syndrome
  • Chromosomal aberrations: several duplications involving 5p, 8p, 8q; trisomy 9, duplication on 17q, Turner syndrome
  • Environmentally induced malformation syndrome: prenatal exposure to rubella, cytomegalovirus, toxoplasmosis, coumadin, alcohol and maternal diabetes
  • Also multifactorial and sporadic disorders: see Congenit Anom (Kyoto) 2007;47:113, Childs Nerv Syst 2011;27:1665
Clinical features
  • The symptoms are related to hydrocephalus, cerebellar and cranial nerves dysfunction and to the presence of associated anomalies
  • 80 - 90% present in the first year
  • DWM is often associated with other brain or systemic anomalies (Cerebellum Ataxias 2016;3:1)
Diagnosis
  • May be diagnosed in utero by using 3D ultrasound as early as 14 weeks of gestation
  • Fetal MRI is indicated in case of posterior fossa malformation suspected by ultrasonography
Radiology description
  • MRI with sagittal views and T2 weighted images are mandatory for studying precisely the content of the posterior fossa
Radiology images

Images hosted on other servers:

Variant DWS
with dysplasia
of pons and
cerebellum

Prognostic factors
  • The clinical course is very variable and depends on the severity of the associated central nervous system malformations, with neurologic development ranging from normal to severely intellectually disabled
Treatment
  • The goal of treatment is the control of the hydrocephalus and of the posterior fossa cyst
Gross description
  • In classic DWM, a huge csyt occupies almost the entire posterior fossa, displacing the brainstem forward and flattening the pons against the clivus
  • The neuropathological picture is completed by aplasia of the vermis, heterotopia of the cerebellar cortex and enlargement of the posterior fossa, with a high position of the tentorium and transverse sinuses
Gross images

Contributed by Jean-François Hirsch, M.D.

Dandy-Walker malformation

Differential diagnosis
  • Different cystic lesions may originate in the posterior fossa
  • Lesions that do not meet the diagnostic criteria should NOT be considered as DWM, i.e. cysts that do not communicate directly with the fourth ventricle, cystic fourth ventricle associated with a normal posterior fossa and normally inserted tentorium

Dementia with Lewy bodies
Definition / general
  • Dementia is defined as a progressive cognitive decline of sufficient magnitude to interfere with normal social or occupational functions or usual daily activities (Neurology 2017;89:88)
  • Lewy body dementia indicates dementia attributed to Lewy body pathology in the neocortex and limbic regions
    • Cognitive changes should occur before or simultaneously with Parkinsonian motor symptoms (see 1 year rule below)
    • Lewy body dementia and Parkinson disease with dementia are distinguished based on clinical criteria; pathologically the 2 conditions cannot be distinguished
Essential features
  • Lewy body dementia (or dementia with Lewy bodies [DLB]) is a clinicopathological term that encompasses a clinical dementia phenotype that precedes or coincides with Parkinsonism within a year of onset and the presence of Lewy bodies in supratentorial distribution (i.e., limbic and neocortical Lewy body pathology subtypes)
  • Misfolded aggregated alpha synuclein is the main constituent of Lewy bodies
  • Alzheimer disease neuropathologic changes frequently co-occur with Lewy body dementia and the relative burden of the 2 pathologies is thought to influence the clinical presentation
Terminology
  • Diffuse / neocortical Lewy body disease (neuropathologic term; recommended)
  • Dementia with Lewy Bodies
  • Lewy body variant of Alzheimer disease (historical term; not recommended)
  • Senile dementia of Lewy body type (historical term; not recommended)
ICD coding
  • ICD-10: G31.83 - dementia with Lewy bodies
  • ICD-11:
    • 8A22 - Lewy body disease
    • 6D82 - dementia due to Lewy body disease
Epidemiology
Sites
Pathophysiology
  • Alpha synuclein aggregates in neuronal cell bodies and neurites in brainstem, limbic and neocortical structures (Neurology 2005;65:1863)
    • Synuclein aggregates in limbic and neocortical areas are associated with cognitive and neuropsychiatric symptoms of dementia with Lewy bodies
    • Synuclein aggregates in brainstem areas are associated with Parkinsonian symptoms
  • While alpha synuclein aggregates are thought to progress in an ascending fashion in Parkinson disease (with or without dementia), whether this is true for Lewy body dementia remains unclear (Neurobiol Aging 2003;24:197, Mol Neurodegener 2019;14:5)
  • Alpha synuclein aggregates are thought to form in a 2 step process (Biochemistry 2008;47:2413, Mol Neurodegener 2019;14:5)
    • Nucleation phase: association of soluble monomers into intermediate oligomers or protofibrils
    • Elongation phase: protofibrils are incorporated into insoluble, β sheet rich fibrils
    • Fragments of the intermediate fibrils (i.e., seeds) may be capable of templating conformational conversion of the native protein, which could explain the spread of pathology across cells in the brain
      • This seeding capability is the basis for studies of synuclein RT-QuIC assays for biomarkers of synuclein pathology (Mol Neurodegener 2019;14:5)
Etiology
Diagrams / tables

Images hosted on other servers:
Alpha synuclein and tau aggregation model

Alpha synuclein and tau aggregation model

Alpha synuclein staging system

Alpha synuclein staging system

Clinical features
  • Unlike Alzheimer disease, dementia with Lewy bodies tends to affect attention, executive function and visuospatial function early in disease progression (J Neuropsychiatry Clin Neurosci 2000;12:425)
  • Besides dementia, DLB patients must present with ≥ 2 core clinical features (Neurology 2017;89:88)
    • Cognitive fluctuations
    • Visual hallucinations: early clinical sign; useful clinical feature in the differential diagnosis of dementia with Lewy bodies versus Alzheimer dementia (Brain 2006;129:729)
    • Rapid eye movement (REM) sleep behavior disorder
    • Parkinsonism: present in 70 - 85% (J Neuropsychiatry Clin Neurosci 2001;13:374, Neurology 2017;89:88)
      • 1 year rule: DLB patients show Parkinsonian motor symptoms within 1 year of the dementia diagnosis; Parkinson disease with dementia is diagnosed when the cognitive symptoms occur in longstanding (i.e., > 1 year) Parkinsonism
  • Supportive clinical features
    • Marked sensitivity to antipsychotic agents
    • Postural instability
    • Repeated falls
    • Syncope or other episodes of unresponsiveness
    • Severe autonomic dysfunction (e.g., urinary incontinence)
    • Hypersomnia
      • Polysomnographic evidence of REM sleep without atonia is an indicative biomarker (Neurology 2017;89:88)
    • Hyposmia
    • Hallucinations in other modalities
    • Systematized delusions
    • Apathy, anxiety and depression
Diagnosis
  • Brain autopsy is the gold standard for Lewy body pathology diagnosis (see Microscopic (histologic) features)
  • Correlation of Lewy body pathology distribution and dementia is the cornerstone of the integrated clinicopathologic diagnosis (Neurology 2005;65:1863)
    • In the presence of both Alzheimer disease neuropathologic changes (ADNC) and Lewy body pathology, the National Institute on Aging (NIA) Reagan score or Braak score is weighed against the Lewy body pathology to establish a probabilistic statement on the individual pathology contribution to the dementia presentation
  • Dementia is an essential feature (see Definition / general)
  • Current clinical criteria are formulated according to clinical features and biomarkers (Neurology 2017;89:88)
  • Clinically probable dementia with Lewy bodies
    • A) 2 or more core clinical features (see Clinical features)
    • B) 1 core clinical feature and ≥ 1 indicative biomarker determined by single photon emission computed tomography (SPECT), PET, iodine meta iodobenzylguanidine (MIBG) myocardial scintigraphy, electroencephalography (EEG) / polysomnography
  • Clinically possible dementia with Lewy bodies
    • A) 1 core clinical feature is present, without biomarker evidence
    • B) 1 or more biomarkers are present but no core clinical features are present
Laboratory
Radiology description
Radiology images

Images hosted on other servers:
Decreased dopamine transporter uptake

Decreased dopamine transporter uptake

Low uptake in myocardial scintigraphy

Low uptake in myocardial scintigraphy

Prognostic factors
  • In a systematic review, dementia with Lewy bodies (1.9 - 6.3 years) showed a shorter survival compared to Alzheimer disease (3.2 - 6.6 years) (Lancet Neurol 2017;16:390)
  • Cognitive decline rate appears to be greater in dementia with Lewy bodies compared to Alzheimer disease in a large multicenter, international cohort (J Alzheimers Dis 2017;57:787)
Case reports
  • 59 year old woman with a novel SNCA E83Q mutation associated with dementia with Lewy bodies and severe frontotemporal atrophy (Neuropathology 2020;40:620)
  • 65 year old man with incidental neocortical Lewy body pathology without involvement of olfactory bulb and brainstem (J Korean Med Sci 2022;37:e195)
  • 75 year old woman with dementia with Lewy bodies and Fahr disease copathology at autopsy (Clin Neuropathol 2020;39:227)
Treatment
  • Involves a multidisciplinary team including providers from neurology, neuropsychology, psychiatry, social work and palliative medicine
  • No pharmacologic approach modifies the disease progression
  • Cholinesterase inhibitors (e.g., donepezil, rivastigmine) are used for the management of cognitive and behavioral symptoms (Am J Psychiatry 2015;172:731)
  • Antipsychotic drugs are used only in severe and refractory behavioral cases
  • Levodopa or carbidopa levodopa for cases with Parkinsonian symptoms
Gross description
  • Brain weight is frequently within normal limits or slightly decreased
  • Pallor of substantia nigra and locus coeruleus is common (Mol Neurodegener 2019;14:5)
  • Cortical atrophy can be present, especially with co-occurring Alzheimer disease neuropathologic changes
  • Hippocampal atrophy is not a gross feature of pure Lewy body pathology
Gross images

Contributed by Javier Redding-Ochoa, M.D.
Brainstem pathology

Brainstem pathology

Cerebrum

Cerebrum

Microscopic (histologic) description
  • Frequent loss of dopaminergic neurons and gliosis in the substantia nigra
  • Lewy bodies vary in morphology according to the brain region involved (Mol Neurodegener 2019;14:5)
    • Cortical and limbic associated Lewy bodies are often poorly demarcated, round or reniform, hyaline, amphophilic and have neuronal perikaryal inclusions
    • Most commonly found in deep cortical layers
    • May not be appreciable on standard H&E staining
    • Brainstem Lewy bodies show a central eosinophilic core, surrounded by a clear halo in pigmented neurons (J Geriatr Psychiatry Neurol 2002;15:210)
  • Pale bodies are faintly stained neuronal cytoplasmic inclusions without a halo, which are composed of alpha synuclein and proposed as a possible precursor to Lewy bodies (Neuropathology 2020;40:30)
  • Glial cells commonly show synuclein pathology in dementia with Lewy bodies, particularly in cortical and limbic areas (Acta Neuropathol 2003;105:163)
  • Alzheimer disease neuropathologic changes (i.e., amyloid plaques and neurofibrillary tangles) occur frequently in dementia with Lewy bodies
  • Lewy body pathology is scored in semiquantitative method in multiple brain regions (Neurology 2005;65:1863)
    • Lewy body (LB) pathology is scored at low power field (10x) in each region from 0 - 4
      • 0 = none
      • Stage 1 = sparse LBs or Lewy neurites (i.e., 1 LB)
      • Stage 2 = moderate (> 1 LBs and sparse neurites)
      • Stage 3 = severe (≥ 4 LBs and scattered neurites)
      • Stage 4 = very severe (numerous LBs and numerous neurites)
    • Scored regions: cranial nerve nuclei IX / X, locus coeruleus, substantia nigra, nucleus basalis of Meynert, amygdala, transentorhinal cortex, cingulate cortex, temporal cortex, frontal cortex and parietal cortex
    • Scores are used to categorize into brainstem predominant, limbic or neocortical Lewy body disease
    • Higher Lewy body pathology burden in cortical (temporal, frontal and parietal cortices) and limbic (nucleus basalis of Meynert, amygdala, transentorhinal cortex, cingulate cortex) areas are related to a clinical presentation consistent with dementia with Lewy bodies
    • Higher Alzheimer disease pathology scores are thought to inversely correlate with the likelihood of clinical presentation consistent with dementia with Lewy bodies (Neurobiol Aging 1997;18:S1, Neurology 2005;65:1863)
Microscopic (histologic) images

Contributed by Javier Redding-Ochoa, M.D.
Lewy bodies in amygdala

Lewy bodies in amygdala

Poorly demarcated cortical Lewy body

Poorly demarcated cortical Lewy body

Classical Lewy body

Classical Lewy body

Synuclein pathology in amygdala

Synuclein pathology in amygdala

Neocortical alpha synuclein pathology

Neocortical alpha synuclein pathology

Substantia nigra neuron loss

Substantia nigra neuron loss

Positive stains
Negative stains
  • Amyloid beta: may label coexisting Alzheimer pathology, negative in Lewy bodies
  • Phospho-tau: may label coexisting tau pathology, can coaggregate in a minority of Lewy bodies (J Neuropathol Exp Neurol 2003;62:389)
  • TDP-43: may label coexisting pathology, negative in Lewy bodies
Electron microscopy description
  • Electron microscopy assessment is not used for diagnosis of Lewy body pathology
  • Correlative light and electron microscopy (CLEM) assessment of Lewy bodies and neurites demonstrated disrupted lipid membranes, distorted organelles (e.g., mitochondria) and vesicles (Nat Neurosci 2019;22:1099)
Sample pathology report
  • Brain, autopsy:
    • Lewy body disease, diffuse (neocortical) type (see comment)
    • Alzheimer disease neuropathologic changes, intermediate level (A2, B2, C2)
    • Gross description:
      • The fresh brain weight is 1,400 g. External examination of the brain is remarkable for mild atrophy. The base of the brain is remarkable for mild multifocal nonocclusive atherosclerosis. The cranial nerves I - XII are normal and the brainstem and cerebellum are externally unremarkable.
      • On coronal sections, the cortical thickness is normal and subjacent white matter is well demarcated. The ventricular system is patent and shows moderate expansion of the lateral ventricles. The basal ganglia, thalamus and hypothalamus are normal in color, shape and size. The amygdalae and hippocampi are unremarkable. In the brainstem, the locus coeruleus and substantia nigra show marked pallor.
    • Microscopic description:
      • H&E stained sections of the brainstem are remarkable for Lewy bodies with severe loss of pigmented neurons in the substantia nigra and locus coeruleus and associated gliosis. In the limbic regions, the amygdala and cingulate cortex show neurons with poorly demarcated spherical eosinophilic cytoplasmic inclusions. Neocortical sections show sparse neurons with perikaryal expansion and eosinophilic inclusions. Additionally, sparse extracellular amyloid plaques are noted in the cortical gray matter.
      • Immunostains for alpha synuclein show Lewy bodies and Lewy neurites with a score of 3 in the substantia nigra, locus coeruleus, cranial nerve nuclei IX / X, nucleus basalis of Meynert, transentorhinal cortex and cingulate cortex. Lewy body and Lewy neurite pathology are present with a score of 4 in the amygdala and with a score of 2 in the temporal and frontal cortex. Lewy body and neurite pathology is present with a score of 1 in the parietal cortex.
      • Silver stains demonstrate moderate numbers of neurofibrillary tangles in the entorhinal cortex, amygdala and hippocampus, with sparse neurofibrillary tangles in the temporal cortex (B2). Moderate neuritic plaques are noted in the frontal, temporal and parietal cortices (C2). Immunostaining for beta amyloid is positive in the thalamus but negative in the midbrain and cerebellum (A2).
    • Comment: This corresponds to a diffuse / neocortical type Lewy body disease. The likelihood that the pathologic findings are associated with a dementia with Lewy bodies clinical syndrome based on the degree of Lewy body and Alzheimer neuropathologic changes is high (Neurology 2005;65:1863).
Differential diagnosis
  • Parkinson disease with dementia:
    • Neuropathologically indistinguishable
    • Clinically, Parkinsonism precedes cognitive changes by more than 1 year
  • Alzheimer disease:
  • Multiple system atrophy:
    • Glial cytoplasmic inclusions of alpha synuclein predominate
    • Neuronal synuclein pathology is not characteristic, although it can be encountered
  • Progressive supranuclear palsy:
    • Glial and neuronal phospho-tau pathology predominantly in perirolandic cortex, putamen, pallidum, subthalamic nuclei, brainstem and cerebellar dentate nucleus
    • Synuclein pathology is not a feature
  • Neurodegeneration with brain iron accumulation (NBIA):
    • Some subtypes show Lewy bodies (e.g., neuroaxonal dystrophy due to PLA2G6 mutations)
    • Age of onset in childhood or early adulthood
    • Iron accumulation in globus pallidus and substantia nigra with spheroids are a major component
  • Postencephalitic Parkinsonism:
    • Tauopathy with severe loss of neurons in the substantia nigra postencephalitis
    • Synuclein pathology is not a feature
Board review style question #1
What feature is essential for a diagnosis of dementia with Lewy bodies?

  1. Dementia
  2. Lewy bodies in the brainstem
  3. Lewy bodies in the hippocampus
  4. Parkinsonism
Board review style answer #1
A. Dementia. Dementia with Lewy bodies is a clinicopathological term that should include a clinical diagnosis of dementia and distribution of Lewy body pathology that explains the neuropsychiatric manifestations (i.e., neocortical and limbic structures) (Neurology 2005;65:1863). Answers B and C are incorrect because Lewy body pathology alone is insufficient for a diagnosis of dementia with Lewy bodies. Answer D is incorrect because Parkinsonism, while a clinical feature of dementia with Lewy bodies, is not essential for the diagnosis.

Comment Here

Reference: Dementia with Lewy bodies
Board review style question #2

Which of the following is the main component of Lewy bodies?

  1. Alpha synuclein
  2. Disrupted organelles
  3. Neurofilaments
  4. Synaptophysin
Board review style answer #2
A. Alpha synuclein. Lewy bodies contain a multitude of components, however, their main constituent is misfolded alpha synuclein (Nature 1997;388:839). Answers C and D are incorrect because although neurofilaments and synaptophysin have been detected in Lewy bodies, these proteins show either weak IHC expression (i.e., synaptophysin) or haven’t been shown to be implicated as the main structural constituent or as a pathogenic driver (Mol Neurobiol 2013;47:495, J Comp Neurol 1991;309:150, Brain Res 1994;634:339). Answer B is incorrect because although lipid structures such as disrupted organelles are abundant in Lewy bodies, other lipid structures are also involved such as general vesicles and other membrane fragments (e.g., autophagosomes) (Nat Neurosci 2019;22:1099). The proportional contributions of these lipid structures are not clear and their pathogenic role in protein seeding, oligomerization or Lewy body maturation is not elucidated.

Comment Here

Reference: Dementia with Lewy bodies

Focal cortical dysplasia (epilepsy related malformations)
Definition / general
  • Focal cortical dysplasia (FCD) represents a phenotypically diverse group of developmental disorders marked by abnormalities of cortical architecture
Essential features
  • Focal cortical dysplasia is marked by cortical architectural abnormalities secondary to disruptions of cortical development
  • Most patients clinically present with epilepsy and in a subset of pharmacoresistent cases, surgery may be employed in an attempt to control the seizures
  • Some focal cortical dysplasia patterns may be grossly evident (e.g., polymicrogyria, pachygyria, heterotopia)
  • The most common morphologic patterns encountered include disruptions in the cortical architecture in either a horizontal or vertical orientation; a subset of cases may demonstrate dysmorphic cortical neurons or balloon cells
  • Focal cortical dysplasia may arise in association with other pathologies, most notably hippocampal sclerosis and tumors (gangliogliomas, dysembryoplastic neuroepithelial tumors)
Terminology
  • Malformations of cortical development, microdysgenesis
ICD coding
  • ICD-10: Q04.8 - other specified congenital malformations of brain
Epidemiology
Sites
  • Any of the cortical lobes may be affected (frontal and temporal most common)
Pathophysiology
  • Not known in most cases
Etiology
Clinical features
  • Seizures most commonly may be associated with developmental delays
    • May be asymptomatic
    • May not always be evident on imaging (Mod Pathol 2013;26:1051)
    • In patients with severe or diffuse lesions: sleep apnea, poor feeding, hypotonia, opisthotonus, spastic quadriplegia, intellectual impairment
Diagnosis
  • EEG and imaging studies (high resolution MRI) studies may be useful in localizing seizures
  • Pathologic examination of excised tissues to confirm the diagnosis
Radiology description
  • Subtle types may not be evident on imaging
  • Cortical thickness changes, irregular cortical folding, abnormalities of the gray-white interface, white matter volume reduction, increased signals on FLAIR and T2 images, transmantle sign
Radiology images

Images hosted on other servers:
Missing Image

Micropolygyria marked by a focal small gyri

Missing Image

Transmantle sign, right superior frontal sulcus

Missing Image

Increased signal intensity

Missing Image

Cortical tubers of tuberous sclerosis

Prognostic factors
  • International League Against Epilepsy (ILAE) type II focal cortical dysplasia (with dysmorphic neurons or balloon cells demonstrate a high degree of epileptogenicity and do not do as well clinically)
  • Widespread or multifocal disease generally also portends a worse prognosis (Pol J Radiol 2012;77:35)
Case reports
Treatment
  • Pharmacologic management is first line treatment
  • When epilepsy is pharmacoresistent, surgery may be employed
  • Seizure free status range following surgery is 11 - 70%; dependent on a variety of factors including multifocality, presence of multiple pathologies and extent of resection (Neurosurg Clin N Am 2002;13:135)
  • mTOR pathway inhibitors may be a novel targeted therapeutic option (Int J Mol Med 2016;38:1327)
Gross description
  • Agyria (lissencephaly) and pachygyria: abnormally thickened cortical ribbon
  • Polymicrogyria (micropolygyria): small, irregularly formed gyri with shallow sulci
  • Heterotopias: nodular or band-like / laminar
  • Hemimegalencephaly: asymmetric enlargement of the cerebrum
  • Cortical tubers of tuberous sclerosis: expanded gyrus, firm to palpation due to gliosis and calcification
  • Most forms show more subtle abnormalities including blurring of the gray-white interface or may not be grossly evident (Perry: Practical Surgical Neuropathology - A Diagnostic Approach, 2nd Edition, 2018)
Gross images

Images hosted on other servers:
Missing Image

Lissencephaly marked by absence of gyral formation

Missing Image

Area of gyral expansion

Missing Image

Resection showing disorganized architecture

Frozen section description
  • Frozen sections not typically used, unless there is a coexistent lesion (e.g., tumor) that is also being excised
Microscopic (histologic) description
  • Agyria / pachygyria: 4 layer cortex
  • Polymicrogyria: 2 to 4 layer cortex
  • Heterotopias: disorganized gray matter tissue (nodular or laminar) situated in a location where it normally is not seen (such as within the white matter)
  • Rare cases (some associated with Aicardi syndrome) are marked by accumulation of proteins in astrocytic cytoplasm (hyaline protoplasmic astrocytopathy) (Am J Clin Pathol 2016;146:503)
  • ILAE Classification schema outlines patterns seen in focal cortical dysplasia (Epilepsia 2011;52:158):
    • Type IA: isolated lesion presenting as radial dyslamination of neocortex
    • Type IB: isolated lesion presenting as tangential dyslamination of neocortex
    • Type IC: isolated lesion presenting as both radial and tangential dyslamination
    • Type IIA: isolated lesion characterized by cortical dyslamination and dysmorphic neurons without balloon cells
    • Type IIB: isolated lesion characterized by cortical dyslamination and dysmorphic neurons with balloon cells
    • Type IIIA: focal cortical dysplasia in combination with hippocampal sclerosis
    • Type IIIB: focal cortical dysplasia in combination with an epilepsy associated tumor
    • Type IIIC: focal cortical dysplasia in combination with a vascular malformation
    • Type IIID: focal cortical dysplasia in combination with another epileptogenic lesion acquired early in life (traumatic injury, ischemic injury or encephalitis)
Microscopic (histologic) images

Contributed by Richard Prayson, M.D.
Missing Image

IA pattern

Missing Image

IB pattern

Missing Image

IIA pattern

Missing Image

IIB pattern


Missing Image

Hyaline protoplasmic astrocytopathy

Missing Image

Nodular heterotopia

Missing Image

Cortical tuber

Missing Image

NeuN

Positive stains
  • NeuN highlights neurons and can be helpful in assessing cortical architecture
  • GFAP can highlight patterns of gliosis
  • MAP2 may be positive in normal neurons and their processes
  • CD34 can highlight balloon cells (Epilepsia 2011;52:158)
Molecular / cytogenetics description
  • ILAE type II focal cortical dysplasia and hemimegalencephaly: mosaic mTORopathies (abnormalities related to mTOR signalling dysregulation); glycosylation defects seen in a subset (about 30%) of type I focal cortical dysplasia (Acta Neuropathol 2019;138:885)
  • Differential hierarchical cluster analysis of DNA methylation can distinguish major focal cortical dysplasia types (Epilepsia 2019;60:1091)
  • Pharmacoresistence may be related to a number of factors: MDR1, MRP1, major vault protein (Public Health Nutr 2017;20:786)
Sample pathology report
  • Temporal lobe mass, partial excision:
    • Ganglioglioma, WHO grade I
    • Focal cortical dysplasia (ILAE type IIIB)
Differential diagnosis
  • Ganglioglioma:
    • May be seen in association with focal cortical dysplasia (ILAE FCD IIIB)
    • Have an atypical glioma component (GFAP, SOX10 or Olig2 positive) in addition to the atypical neuronal cell component
    • May have eosinophilic granular bodies and may be BRAF V600E +
  • Gangliocytoma:
    • Presents as a mass, pure ganglion cell tumor
  • Architectural changes adjacent to a remote infarct (especially early in life during brain development)
Board review style question #1

A 6 year old female patient presents with a history of temporal lobe epilepsy since age 3 years. A variety of pharmacologic treatment regimens have been attempted to control the seizures with varying temporary success. On imaging, she has an area of vague enhancement in the right temporal lobe. Concerned about a possible tumor, the area of enhancement is excised. The pathology is shown here. What is the best diagnosis for the lesion seen here?

  1. Dysembryoplastic neuroepithelial tumor
  2. Focal cortical dysplasia ILAE type I
  3. Focal cortical dysplasia ILAE type II
  4. Ganglioglioma
  5. Pleomorphic xanthoastrocytoma
Board review style answer #1
C. Focal cortical dysplasia ILAE type II. The lesion is marked by the presence of dysmorphic neurons and occasional balloon cells, characteristic of focal cortical dysplasia IIB. Focal cortical dysplasia type I does not contain dysmorphic neurons or balloon cells. Dysembryoplastic neuroepithelial tumors are low grade glioneuronal neoplasms marked by multinodularity and a mixture of rounded oligodendroglial-like cells intermixed with normal appearing neuronal cells, often arranged against a microcytic background. Gangliogliomas have an atypical glial cell component along with dysmorphic ganglion cells. Pleomorphic xanthoastrocytomas are characterized by prominent cellularity with an atypical astrocytic component and variable numbers of xanthoastrocytes.

Comment Here

Reference: Focal cortical dysplasia (epilepsy related malformations)
Board review style question #2
Which of the following tumors is most likely to be associated with coexistent focal cortical dysplasia?

  1. Fibrillary astrocytoma
  2. Ganglioglioma
  3. Meningioma
  4. Oligodendroglioma
  5. Pilocytic astrocytoma
Board review style answer #2
B. Ganglioglioma. Tumors known to be associated with coexistent focal cortical dysplasia include ganglioglioma, dysembryoplastic neuroepithelial tumor, angiocentric glioma and rare cases of pleomorphic xanthoastrocytoma. The other tumors listed as options with the question are not known to be associated with focal cortical dysplasia.

Comment Here

Reference: Focal cortical dysplasia (epilepsy related malformations)

Frontotemporal (pending)
Table of Contents
Definition / general
Definition / general
[Pending]

Fungal infections (pending)
Aspergillosis
[Pending]
Cryptococcus
[Pending]

Cytology images

Contributed by Cristina Aguilar, M.D., Allegheny Valley Hospital, PA (USA)

Pap stain

Pap stain

Mucin

PAS fungal stain

Dematiaceous fungi
[Pending]
Dimorphic fungi
[Pending]
Mucormycosis
[Pending]

HIV encephalitis
Definition / general
  • HIV encephalitis refers to cognitive impairment resulting from productive cerebral infection by the human immunodeficiency virus
  • It does not apply to opportunistic infections resulting from generalized cell mediated immunodeficiency
Terminology
  • Prior to successful implementation of combined antiretroviral therapy, severe cognitive impairment related to HIV encephalitis was also referred to as HIV associated dementia (HAD) or the AIDS dementia complex
  • Currently, other terms used to describe this illness include HIV associated neurocognitive disorder (HAND) and HIV associated neurocognitive impairment
  • It has been proposed that HIV neurocognitive impairment be divided into type I (corresponding to neurocognitive disability resulting from active HIV encephalitis) and type II (corresponding to mild cognitive impairment, for which the neuropathological substrate has not been established)
Epidemiology
  • Although formally encountered as a severe dementia occurring in at least 20% of end stage AIDS patients with critically low CD4 counts and high viral loads, in the current era of combined antiretroviral therapies, the disease is now most commonly manifested as mild neurocognitive disability in clinically stable patients
  • Prevalence estimates for HIV associated neurocognitive disorder range from 35 to 100%
  • References: PLoS One 2012;7:e46178, J Infect Dis 2008;197:S294
Sites
  • Although HIV encephalitis may occur anywhere within the nervous system, there is a distinct predilection for pathologic changes to be found predominantly within subcortical white matter and basal ganglia
Pathophysiology
  • Monotropic strains of the HIV virus gain entrance to the nervous system within monocytes which easily traverse the blood brain barrier
  • Infected monocytes then fuse with resident microglial cells, resulting in activation with release of cytokines and other molecules which result in secondary brain injury
  • In addition, HIV associated glycoproteins may interfere with synaptic activity
  • However, neurons are not productively infected by the virus
  • References: PLoS Pathog 2011;7:e1002286, Am J Pathol 2010;176:893, Am J Pathol 2011;179:1623, Neurology 2013;80:1415
Diagnosis
  • Currently, the diagnosis is made through the neuropsychometric performance testing, in combination with neuroimaging
Laboratory
  • Classical HIV disease biomarkers (low CD4 count, high viral load) are no longer closely associated with neurological impairment although markers of systemic inflammation appear to correlate with cognitive impairment
Radiology description
  • Characteristic neuroimaging appearance of HIV associated neurocognitive disorder is increased white matter signal on fluid level attenuated inversion recovery (FLAIR) images although these are only apparent in the more advanced stages of cognitive impairment
Radiology images

Images hosted on other servers:

Cranial MRI

Symmetric patchy periventricular

Case reports
Treatment
  • Although implementation of combined antiretroviral therapy has reduced the prevalence of severe neurocognitive impairment, optimized HIV therapy is not sufficient to avert cognitive impairment (even when CNS penetrating antivirals are utilized)
  • It is believed that in addition to combined antiretroviral therapy, treatments that will address inflammation and cardiovascular risks may provide benefit
Gross description
  • Brain of patients with uncomplicated HIV encephalopathy shows no discernible gross abnormalities
  • In fact, the presence of gross abnormalities suggests the presence of additional pathologies related to immunosuppression
Microscopic (histologic) description
  • Similar to other viral infections of the nervous system, HIV encephalitis is characterized by the presence of microglial nodules
  • Unlike other viral infections, HIV stimulates fusion of microglial cells to form multinucleated microglial cells
  • In addition, the amount of lymphocytic inflammation is generally considerably less than that seen in viral infections of immunocompetent hosts
Microscopic (histologic) images

Images hosted on other servers:

Stained sections of the white matter

Positive stains
  • Several antibodies are available that recognize HIV associated glycoproteins
  • p24 shows reactivity within both perivascular mononuclear and multinucleate microglial cells
Differential diagnosis
  • Although the presence of superimposed opportunistic infections or neoplasms must always be kept in mind, the presence of subcortical microglial nodule encephalitis with multinucleated microglial cells is virtually pathognomonic for HIV encephalitis

Hepatic encephalopathy
Definition / general
  • Spectrum of neurocognitive abnormalities occurring in patients with acute or chronic liver disease or portosystemic shunting
Essential features
  • Clinical pattern and pathology are not dependent on the etiology of liver disease
  • Characterized by neurocognitive abnormalities
  • Elevated blood ammonia levels considered to play central role
  • Histologic hallmark - Alzheimer type II astrocyte - enlarged, pale nuclei with a rim of chromatin
Terminology
  • Hepatic encephalopathy (HE)
  • Portosystemic encephalopathy
ICD coding
  • ICD-9:
    • 572.2 - hepatic encephalopathy
  • ICD-10:
    • K72.91 - hepatic failure, unspecified with coma
    • K70.41 - alcoholic hepatic failure with coma
    • K72.11 - chronic hepatic failure with coma
    • K71.11 - toxic liver disease with hepatic necrosis, with coma
    • K72.01 - acute and subacute hepatic failure with coma
Epidemiology
  • Chronic alcoholism, acute drug poisoning, status post portosystemic bypass, hepatic failure patients
Sites
  • Gray matter astrocytes
Pathophysiology
  • Acute hepatic encephalopathy: cerebral edema and increased intracranial pressure (cause of death)
  • Pathophysiology of hepatic encephalopathy is multifactorial
  • Postulated agents include ammonia (NH3), inflammatory cytokines, manganese deposition in the basal ganglia, benzodiazepine-like compounds (GABA), microbiota and aromatic amino acids (Drugs 2019;79:17)
  • Ammonia has been considered to play a central role in HE (see Diagrams / tables)
Etiology
  • HE type A: HE associated with acute liver failure (e.g. acetaminophen overdose)
  • HE type B: HE associated with portosystemic bypass in the absence of hepatic disease
  • HE type C: HE associated with cirrhosis and portal hypertension or portosystemic bypass
  • Reference: Hepatology 2002;35:716
Diagrams / tables

Contributed by Palgun Nisarga, M.D.
Ammonia and hepatic encephalopathy

Ammonia and hepatic encephalopathy

Clinical features
  • Disorientation, behavioral and personality changes, including somnolence, agitation, anger and restlessness
  • Motor abnormalities, including Parkinsonism, choreoathetosis, spastic paraparesis with hyperreflexia and slowness of speech
  • Dementia-like pattern and sleep disturbances
  • May be precipitated by infection, gastrointestinal bleeding, medication noncompliance, electrolyte imbalance / renal failure or acute liver injury
  • Clinical patterns: coma, rapidly developing confusion (somnolent or agitated), continuous mild mental dysfunction with recurrent episodes of severe confusion, predominant motor disorder (pyramidal and extrapyramidal) or mild brain dysfunction (J Clin Exp Hepatol 2018;8:432)
  • Asterixis in alcoholic liver disease
Diagnosis
  • Based predominantly on clinical features
  • West Haven criteria used to grade severity (Hepatology 2002;35:716)
Laboratory
  • Serum ammonia levels may be elevated
Radiology description
  • Symmetric, hyperintense signal in the basal ganglia, especially the globus pallidus, on T1 weighted MR images
  • Restricted diffusion in the cerebral cortex (diffusion weighted imaging)
  • Reference: AJNR Am J Neuroradiol 2008;29:1612
Prognostic factors
  • Prognosis based on percent of viable hepatic parenchyma in acute hepatic failure
  • Necrosis > 50% in liver biopsy - threshold for liver transplantation (Clin Liver Dis 2018;22:257)
  • Poor prognosis independent of severity of liver disease and despite treatment advances; 1 year survival approximately 20 - 40% (World J Gastroenterol 2020;26:2221)
Case reports
Treatment
  • Treatment of precipitating event
  • Reduce blood ammonia concentration using nonabsorbable disaccharides or antibiotics
  • Liver transplantation
  • Reference: Aliment Pharmacol Ther 2010;31:537
Gross description
  • Acute hepatic encephalopathy: cerebral edema with herniation
  • Chronic hepatic encephalopathy: brain is typically normal grossly
  • Reference: N Engl J Med 2016;375:1660
Microscopic (histologic) description
  • Alzheimer type II astrocytes: enlarged, pale nuclei with a rim of chromatin and a prominent nucleoli
  • Astrocyte doublets / pairs and triplets are seen
  • Severe disease: astrocyte nucleus appears lobulated and contains glycogen granules
  • Most commonly seen in gray matter regions, including globus pallidus, putamen, pons, thalamus, dentate nucleus and cortical gray matter (deep layers)
  • Acute hepatic encephalopathy: brain edema due to swelling of perivascular astrocytes (Hepatology 1992;15:1060)
  • Reference: J Neuropathol Exp Neurol 1994;53:213
Microscopic (histologic) images

Contributed by Kymberly A. Gyure, M.D. and Palgun Nisarga, M.D.
Alzheimer type II astrocyte Alzheimer type II astrocyte

Alzheimer type II astrocyte

Positive stains
Negative stains
  • Severe disease: astrocytes lose immunoreactivity to GFAP
Sample pathology report
  • History of cirrhosis with hepatic encephalopathy
    • Alzheimer type II astrocytosis
Differential diagnosis
  • Differential diagnosis is investigated from a clinicoradiological standpoint
  • Clinically should be distinguished from stroke
  • Alzheimer type II astrocytes also seen in uremic encephalopathy, hypercapnic encephalopathy and in infants with hypoxia and hypoglycemia; their distinction from hepatic encephalopathy is based on clinical and radiologic features, not pathology
Board review style question #1
Which of the following is the most likely cause of death in a patient with acute hepatic encephalopathy?

  1. Acute renal failure
  2. Bleeding diathesis
  3. Cerebral edema
  4. Hypoglycemia
  5. Myocardial necrosis
Board review style answer #1
C. Cerebral edema and herniation are common in acute hepatic encephalopathy

Comment Here

Reference: Hepatic encephalopathy
Board review style question #2

Which part of the CNS most commonly shows increased Alzheimer type II astrocytes?

  1. Basal ganglia
  2. Brain stem
  3. Cerebellum
  4. Cerebral cortex
  5. Internal capsule
Board review style answer #2
A. Alzheimer type II astrocytes are most commoly found in the basal ganglia, especially the globus pallidus

Comment Here

Reference: Hepatic encephalopathy

Heroin leukoencephalopathy
Definition / general
  • Toxic leukoencephalopathy secondary to the use of inhaled heroin or "chasing the dragon"
Sites
  • White matter including cerebellar white matter
Pathophysiology
  • Heating of heroin is thought to activate a lipophilic compound that damages white matter
Clinical features
  • Stage 1: predominantly cerebellar symptoms
  • Stage 2: cerebellar and extrapyramidal symptoms
  • Stage 3: progressive spasms, akinetic mutism and death
Radiology description
  • Increased signal on T2 weighted and FLAIR MR images in white matter, including cerebellar white matter with sparing of the dentate nucleus
Radiology images

Images hosted on other servers:
Missing Image

Axial FLAIR sequences

Missing Image

Increased T2 signal

Missing Image

Axial gadolinium enhanced T1 sequences

Case reports
Treatment
  • Supportive care
Microscopic (histologic) description
  • Myelin loss / vacuolation - "spongiform encephalopathy"
  • Inflammatory infiltrate consisting of macrophages with associated perivascular chronic inflammation
Microscopic (histologic) images

Images hosted on other servers:
Missing Image

Cerebral cortical gray matter

Missing Image

Perivascular inflammation: lymphocytes and macrophagess

Missing Image

Gradual loss of myelination

Missing Image

Marked loss of axons

Missing Image

Intense macrophage infiltration

Differential diagnosis

Herpes simplex encephalitis
Definition / general
  • Inflammation of the brain parenchyma with neurologic dysfunction due to involvement of the brain by the herpes simplex viruses (HSV)
Essential features
  • HSV encephalitis is the most common cause of acute necrotizing encephalitis
  • HSV1 is responsible for > 90% of cases and affects older adults, with a median age of 57 years (J Neurol 2017;264:1204)
  • HSV2 is responsible for disseminated neonatal encephalitis
  • Polymerase chain reaction (PCR) is the diagnostic method of choice with a sensitivity of > 95% and specificity of ~99%
  • Mortality without treatment is 70% (Clin Microbiol Rev 2020;33:e00105)
  • With treatment the mortality drops to ~20% but 70% of survivors show neurological deficits (Clin Microbiol Rev 2020;33:e00105)
Terminology
  • HSV encephalitis
ICD coding
  • ICD-10: B00.4 - herpesviral encephalitis
Epidemiology
Sites
  • Asymmetric limbic system (medial temporal and frontal lobes), cingulate and insular cortex
  • More diffuse presentation in neonatal HSV encephalitis
Pathophysiology
  • Classic herpes simplex virus (HSV) encephalitis typically arises from HSV1, although HSV2 accounts for a minority
  • Spread of HSV1
    • Contact with infected saliva or respiratory secretions
    • Primary infection by HSV1 typically involves the oropharyngeal mucosa
    • Less common sites include the skin, particularly fingers and shoulders of wrestlers (herpes gladiatorum) (Sports Health 2013;5:423)
  • Latent infection is caused by retrograde axonal transport along sensory fibers, leading to colonization of the primary sensory (such as trigeminal) ganglia (Mol Ther Methods Clin Dev 2020;18:33)
  • 3 proposed routes of entry to the brain
    • Spread of virus along olfactory nerve fibers and tracts
    • Reactivation of latent virus in the trigeminal ganglia and its spread along either the centrally projecting nerve roots into the brain stem or the peripherally projecting sensory fibers that innervate the meninges of the anterior and middle cranial fossae
      • Localized proliferation of the virus, accompanied by neuritic transport to the skin or mucosal tissues, results in the manifestation of cold sores
      • Mechanisms of reactivation are unclear
      • Reactivation may seem to be spontaneous, precipitated by local stimuli (e.g., trauma and ultraviolet [UV] irradiation) involving skin or mucosa innervated by latently infected neurons or may result from a range of systemic factors (e.g., pyrexia, emotional stress, physiological fluctuations in hormone levels during the menstrual cycle and immunosuppression)
    • Reactivation of virus that has previously established latent infection within the brain (World Neurosurg 2016;89:731.e1)
  • Neonatal HSV encephalitis
    • HSV infection in neonates is infrequent
    • Both HSV1 and HSV2 are implicated in causing neonatal herpes infection (Semin Perinatol 2018;42:168)
  • Granulomatous HSV encephalitis
    • HSV can cause granulomatous encephalitis in children, though its occurrence in adults has been rarely reported (Pract Neurol 2017;17:140)
  • HSV induced autoimmune encephalitis
    • Secondary immune mediated neurological deterioration triggered by HSV infection of the central nervous system (CNS)
    • Mediated by neuronal autoantibodies, most commonly against the N-methyl-D-aspartate receptor (NMDAR) for the neurotransmitter glutamate (Clin Microbiol Rev 2020;33:e00105)
Etiology
Clinical features
  • Combination of nonspecific features of encephalitis, along with focal neurological signs attributed to the engagement of the temporal and frontal lobes
  • Fever, headache and confusion
  • Focal neurological signs include aphasia and focal seizures
  • While the onset may be fulminant, a substantial number of patients experience an influenza-like prodromal illness
  • Personality changes and behavioral abnormalities might occur over several days (Neurol Sci 2013;34:1879)
Diagnosis
  • Cerebrospinal fluid (CSF) HSV PCR
  • Magnetic resonance imaging (MRI) of brain is the preferred imaging test
Laboratory
  • Lumbar puncture, CSF
    • May be normal in early stage
    • Opening pressure: normal or slightly elevated
    • Moderate lymphocytic pleocytosis (10 - 200/mm3)
    • Variable numbers of red blood cells
    • Elevation of protein, usually to 80 - 100 mg/dL
    • Normal glucose
    • HSV antibody titer in the CSF: 4 fold or greater rise
      • Sensitivity is relatively poor and serological confirmation may not be possible for some weeks
    • Culture of CSF is usually negative for HSV, although viral antigen can be demonstrated in the CSF in a high proportion of cases (Neurotherapeutics 2016;13:493)
    • Increased levels of IFNγ, TNFα, IL6, IL2 receptor and CCL2
    • PCR based detection of HSV DNA in the CSF is the diagnostic method of choice
      • Sensitivity: > 95%
      • Specificity: ~99%
      • Remains positive for several days after the commencement of treatment with acyclovir (Neurotherapeutics 2016;13:493)
  • Brain biopsy is sensitive but aggressive and is replaced by PCR (Neurotherapeutics 2016;13:493)
  • Elevated level of IL10 in serum is associated with a high Glasgow coma score (GCS) (Clin Microbiol Rev 2020;33:e00105)
  • IgG antibodies against N-methyl-d-aspartate receptor (NMDAR) in the serum / CSF (Clin Microbiol Rev 2020;33:e00105)
Radiology description
  • MRI with and without contrast
    • Most sensitive and specific imaging method
    • T1: edema can be seen
      • T1 with contrast: enhancement of temporal lobe in the affected area
    • T2: asymmetric hyperintense lesions corresponding to areas of edema in the affected area
    • Diffusion weighted imaging (DWI)
      • More sensitive than T2 (in acute infection)
      • Associated restricted diffusion may be seen due to edema (Acta Trop 2010;116:206)
  • Computed tomography (CT) scans
    • Hypodense lesions (typically in the temporal lobe), edema
    • Low sensitivity, particularly early in the course of the illness
  • Electroencephalography (EEG)
Radiology images

Contributed by Serguei Bannykh, M.D.
Temporal lobe involvement with restricted diffusion

Temporal lobe involvement with restricted diffusion

Typical radiologic appearance of brain involvement in HSV encephalitis

Brain involvement in HSV encephalitis

Brain edema in HSV encephalitis

Brain edema in HSV encephalitis



Images hosted on other servers:
Brain MRI in acute HSV1 encephalitis

Brain MRI in acute HSV1 encephalitis

Asymmetric medial temporal hyperintensities

Asymmetric medial temporal hyperintensities

FLAIR hyperintensities

Bilateral FLAIR


Abnormal enhancement

Hemorrhages in both temporal lobes

Bilateral symmetric cortical swelling

High signal intensity lesions

Prognostic factors
Case reports
  • 23 year old woman presented with epileptic attack followed by psychiatric symptoms of confusion and irritation as well as cognitive deficits (Neurol Sci 2021;42:857)
  • 27 year old man presented with severe headache and altered mental status 3 days after administration of ChAdOx1 nCoV-19 vaccine (BMC Infect Dis 2022;22:217)
  • 27 year old man with a 3 day history of partial loss of consciousness and falls (J Neurovirol 2020;26:138)
  • 78 year old woman with declining mental status 11 days after a posterior C3 T1 tumor resection (Case Rep Surg 2019;2019:2065716)
Treatment
Gross description
  • Acute phase
    • Bilateral, usually asymmetrical, hemorrhagic necrosis involving the temporal lobes, especially anteriorly and inferiorly, and to a greater or lesser extent, the insulae, cingulate gyri, posterior orbital frontal cortices and underlying white matter
    • Brain is usually edematous
    • In the early phase, the lesions can be confined to the inferomedial part of 1 temporal lobe only or brain may even appear grossly normal
  • Chronic phase
    • In patients dying some weeks after the onset of untreated or unsuccessfully treated disease, the hemorrhagic necrosis progresses to cavitation and atrophy, with indentation or collapse of the affected temporal lobes, yellow to brown discoloration of the surrounding brain tissue and overlying meninges
    • In longer term survivors, the affected parts of the brain appear shriveled and brown
  • References: Love: Greenfield’s Neuropathology, 9th Edition, 2015, Clin Neuropathol 1995;14:45
Gross images

Contributed by Serguei Bannykh, M.D.
Acute HSV encephalitis with hemorrhagic transformation

Acute HSV encephalitis with hemorrhagic transformation

Medial temporal lobes involvement

Medial temporal lobe involvement

Subacute neonatal HSV encephalitis

Subacute neonatal HSV encephalitis

Subacute neonatal HSV encephalitis

Chronic HSV encephalitis

Microscopic (histologic) description
  • Early phase of acute infection
    • Little or no parenchymal inflammation
    • Perivascular neutrophils and lymphocytes
    • Affected neurons, glia and endothelial cells tend to have slightly hypereosinophilic cytoplasm
    • Neuronal intranuclear eosinophilic or basophilic inclusions composed of nucleic acid and protein (Cowdry A inclusions)
    • Clumps of eosinophilic inclusions in the cytoplasm
  • Advanced phase of acute infection
    • Ferrugination of necrotic neurons
    • Resolving hemorrhage
    • Perivascular and interstitial infiltrates of lymphocytes and sheets of foamy macrophages
    • Neuronophagia and microglial nodules
    • Nuclear inclusions are sparse at this stage (Acta Neuropathol 2016;132:433)
Microscopic (histologic) images

Contributed by Mahzad Azimpouran, M.D. and Serguei Bannykh, M.D.
Red neurons, microglial nodule

Red neurons, microglial nodule

Occasional neuronal intranuclear inclusions

Occasional neuronal intranuclear inclusions

Predominantly lymphocytic inflammation

Predominantly lymphocytic inflammation

Calcific changes in neurons

Calcific changes in neurons

Intranuclear inclusions

Intranuclear inclusions

IHC staining for HSV1

IHC staining for HSV1

Positive stains
Electron microscopy description
  • HSV hexagonal viral particles appear as arrays and scattered single particles (size: 185 - 225 nm) in the nucleus of a neurons
Electron microscopy images

Images hosted on other servers:
herpes nucleocapsids within the nucleus of a neuron

Herpes nucleocapsids within nucleus of a neuron

Molecular / cytogenetics description
Videos

HSV encephalitis

Sample pathology report
  • Brain, right temporal, biopsy:
    • Acute lymphocytic herpes virus encephalitis
    • Positive for HSV1 by IHC
Differential diagnosis
  • Primary or secondary CNS infections caused by mycobacteria / atypical organisms / viruses / prion / fungus or parasites:
    • Gram or GMS stain helps to differentiate
  • HIV encephalitis:
    • Presence of subcortical microglial nodules with multinucleated microglial cells is virtually pathognomonic for HIV encephalitis in settings of HIV seropositivity
  • Autoimmune or paraneoplastic limbic encephalitis:
    • Onset and evolution tend to be more subacute than acute
    • Nonnecrotizing histopathology consisting of perivascular lymphocytic infiltrates and microglia nodules
    • Identification of autoantibodies helpful in this differential
  • Primary intracranial or metastatic tumors:
    • Biopsy or surgical resection required for definitive diagnosis
  • Intravascular lymphoma:
    • Highly atypical CD20 positive cell inside microvasculature
  • Acute infarcts:
    • Neurons with acute hypoxic / ischemic type histologic changes found in both
    • No microglial nodules, neuronophagia or viral cytopathic effect
    • Negative HSV staining
    • Negative PCR test
  • Hypoxemic and septic encephalopathies:
    • Requires clinical correlation
    • HSV negative on immunostaining
    • No microglial nodules, neuronophagia or viral cytopathic effect
  • Metabolic causes, including hepatic / uremic encephalopathies, Wernicke encephalopathy, mitochondrial encephalopathies, hypoglycemia, hyponatremia or hypernatremia, hypocalcemia or hypercalcemia:
    • Neurons are relatively spared within many metabolic lesions
    • HSV negative on immunostaining
    • No microglial nodules, neuronophagia or viral cytopathic effect
    • Requires clinical correlation for more definite diagnosis
  • Vasculitis, systemic lupus erythematosus (SLE), Behçet disease:
    • HSV negative on immunostaining
    • No microglial nodules, neuronophagia or viral cytopathic effect
    • Destructive granulomatous or lymphocytic vasculitis present for vasculitic lesions
    • Microthrombi can be present in SLE associated CNS disease (Rheumatology (Oxford) 2017;56:77)
Board review style question #1
Which diagnostic method is commonly employed for the confirmation of herpes simplex virus (HSV) encephalitis, specifically involving the examination of cerebrospinal fluid?

  1. Magnetic resonance imaging (MRI)
  2. Microscopic analysis of brain biopsy
  3. Polymerase chain reaction (PCR)
  4. Serological testing for antibodies
Board review style answer #1
C. Polymerase chain reaction (PCR). PCR is a highly sensitive and specific technique that can detect and amplify specific DNA sequences. In the context of HSV encephalitis, PCR is used to amplify and identify HSV DNA in the cerebrospinal fluid (CSF). PCR allows for the early and rapid detection of viral DNA, providing a quicker diagnosis compared to other methods. This is crucial in the case of encephalitis, where prompt treatment is essential.

Answer D is incorrect because serological testing is valuable for detecting antibodies in the blood but may not directly confirm the presence of the virus in the CSF, which is crucial for diagnosing HSV encephalitis. Answer A is incorrect because while MRI is an important imaging modality for identifying brain abnormalities, it does not directly detect the presence of HSV in the CSF. It is often used as a complementary tool for diagnosis but is not the primary method for confirming HSV encephalitis. Answer B is incorrect because although brain biopsy can provide direct tissue samples for analysis, it is generally not the first line approach for diagnosing HSV encephalitis due to its invasiveness; moreover, CSF analysis through PCR is more commonly employed for its sensitivity and specificity in detecting HSV DNA.

Comment Here

Reference: Herpes simplex encephalitis
Board review style question #2

A 35 year old woman who delivered a healthy baby 2 weeks prior presented with confusion, memory problems and altered mental status. A subsequent MRI revealed a T2 hyperintense left medial temporal focally enhancing lesion and cerebrospinal fluid (CSF) analysis is pending. Given the possibility of uncal herniation, an open biopsy was performed and a representative H&E image is displayed above. Which of the following statements are accurate regarding the diagnosis of HSV (herpes simplex virus) encephalitis based on the provided information?

  1. Brain biopsy is indispensable for diagnosis of HSV encephalitis
  2. False negative rate of HSV1 / 2 PCR on CSF analysis is 50%
  3. Histologic findings in the above image are not supportive of a diagnosis of HSV encephalitis
  4. HSV encephalitis can result in short term memory failure
Board review style answer #2
D. HSV encephalitis can result in short term memory failure. HSV encephalitis can result in confusion and altered mental status and especially a short term memory failure as observed in this case. Confusion and altered mental status are common clinical manifestations of HSV encephalitis. The information provided in the question, where the patient presented with confusion, memory problems and altered mental status, aligns with the typical neurological symptoms associated with HSV encephalitis.

Answer B is incorrect because in HSV encephalitis, the sensitivity of PCR test is reported as 95%, therefore the false negative rate constitutes 5%. Answer A is incorrect because in an appropriate clinical setting with supporting CT or MRI findings, PCR based cerebrospinal fluid (CSF) analysis for HSV is sufficient for the diagnosis of HSV encephalitis, precluding a need for invasive brain biopsy. Answer C is incorrect because the characteristic features of viral encephalitis, such as inflammation and cellular changes, as shown in the above image, are indeed observed in the histological examination.

Comment Here

Reference: Herpes simplex encephalitis

Holoprosencephaly (pending)

Huntington disease
Definition / general
  • Autosomal dominant neurodegenerative disorder characterized by abnormal movements, cognitive impairment and neuropsychiatric disturbances
  • Secondary to expanded CAG trinucleotide repeat within the HTT gene encoding for the huntingtin protein
Essential features
  • Hyperkinetic movement disorder causing choreiform movements, rigidity, cognitive decline, dementia and neuropsychiatric disturbances
  • Autosomal dominant, polyglutamine disorder characterized by trinucleotide repeat expansion (CAG)n within the HTT gene on chromosome 4 (> 39 repeats resulting in disease)
  • Characteristic atrophy of the striatum, particularly the caudate nucleus
  • Variable atrophy of other brain regions, though cortex is often affected
  • Loss of neurons and reactive gliosis, especially within affected areas of the basal ganglia
Terminology
  • Huntington chorea
ICD coding
  • ICD-10: G10 - Huntington disease
  • ICD-11: 8A01.10 - Huntington disease
Epidemiology
Sites
Pathophysiology
  • Mutant huntingtin protein (mHTT) has direct and indirect downstream effects leading to neuronal loss and dysfunction (Eur J Neurol 2018;25:24)
    • Abnormal intranuclear aggregation of mHTT causes secondary dysregulation of transcription
    • Abnormal cytoplasmic aggregation of mHTT causes impairment of proteostasis and impairment of synaptic function, mitochondrial function / energy metabolism and axonal transport
  • Striatal neurons, specifically medium spiny neurons (MSNs), are selectively vulnerable to the effects mHTT (Curr Opin Neurobiol 2015;33:53)
  • Selective early loss of striatal GABA / enkephalin neurons projecting to the globus pallidus externa (GPe) correlates with chorea, while nonselective loss is more associated with akinetic rigid phenotypes (Mov Disord 1999;14:398)
Etiology
  • Autosomal dominant trinucleotide CAG repeat in the huntingtin gene (HTT) on chromosome 4 leading to a mutant protein product mHTT (Eur J Neurol 2018;25:24, Curr Opin Neurobiol 2015;33:53)
  • Disease is notable for variable penetrance based on the number of CAG repeats and is notable for a process called anticipation (Eur J Neurol 2018;25:24)
    • < 27 repeats: normal range
    • 27 - 35 repeats: intermediate number
    • 36 - 39 repeats: disease with reduced penetrance
    • > 39 repeats: disease with full penetrance, with greater numbers resulting in earlier disease onset
    • Anticipation phenomenon
      • Paternal inheritance results in greater numbers of CAG repeats in offspring and subsequent earlier disease onset with more severe symptoms
Diagrams / tables

Images hosted on other servers:
Pathogenic effects of mHTT

Pathogenic effects of mHTT

Clinical disease progression

Vonsattel grading; gross pathology

Clinical features
  • Disease progression can be divided into phases (Lancet 2007;369:218)
    • Presymptomatic / prediagnostic phase
      • Neuropsychiatric: irritability, disinhibition
    • Diagnostic phase
      • Hyperkinetic phenotype: prominent chorea (uncontrollable jerking / writhing movements) and dystonia (involuntary muscle contractions, often painful)
      • Hypokinetic phenotypes: bradykinesia (slowness of movement), gait disturbance, imbalance
      • Cognitive dysfunction: poor executive function and speech impairment
      • Neuropsychiatric: depression and suicidal ideation
Diagnosis
Laboratory
  • Biomarkers to measure disease burden are under investigation and show promise, including measurement of mHTT in cerebrospinal fluid (CSF) (Eur J Neurol 2018;25:24)
Radiology description
Radiology images

Images hosted on other servers:

Caudate volume HDL2 and HD versus normal

Case reports
Treatment
  • Key management strategies revolve around a multidisciplinary approach to target the main symptoms of Huntington disease; a curative treatment is not currently available (Eur J Neurol 2018;25:24)
    • Motor symptoms: treatment with amine transport inhibitors (e.g., tetrabenazine, deutetrabenazine)
    • Psychiatric symptoms: includes pharmacologic and nonpharmacologic interventions to treat depressive symptoms and irritability
    • Cognitive symptoms: difficult to target with currently approved therapies
Gross description
Gross images

Contributed by Jared T. Ahrendsen, M.D., Ph.D.
Vonsattel grade 3 atrophy

Vonsattel grade 3 atrophy



Images hosted on other servers:

Vonsattel grading

Microscopic (histologic) description
  • Neostriatal atrophy with notable loss of neurons and astrogliosis (J Neuropathol Exp Neurol 1998;57:369)
  • Vonsattel grading scheme (J Neuropathol Exp Neurol 1985;44:559, Ann Neurol 2001;49:29, Brain Pathol 2016;26:726)
    • Grade 0: microscopy shows minimal to mild neuronal loss in the head of the caudate; ubiquitinated inclusions present
    • Grade 1: microscopy shows mild neuronal loss and astrocytosis affecting the caudate head and putamen; ubiquitinated inclusions present
    • Grade 2: microscopy shows neuronal loss and gliosis in the ventrolateral caudate; increased oligodendrocyte density appreciable
    • Grade 3: microscopy shows neuronal loss and astrocytosis throughout striatum with sparing of nucleus accumbens
    • Grade 4: microscopy shows severe neuronal loss and astrocytosis with involvement of nucleus accumbens
  • Cortical neuronal loss and white matter thinning, with neuronal loss being most pronounced in cortical layers III, V and VI (Brain Pathol 2016;26:726)
Microscopic (histologic) images

Contributed by Jared T. Ahrendsen, M.D., Ph.D.
Caudate, neuronal loss Caudate, neuronal loss

Caudate, neuronal loss

Caudate, reactive gliosis

Caudate, reactive gliosis

Ubiquitin+ inclusions

Ubiquitin positive inclusions

Positive stains
Negative stains
Molecular / cytogenetics description
Sample pathology report
  • Brain and spinal cord, autopsy examination:
    • Neuropathologic findings in keeping with patient's known history of Huntington disease (see comment)
    • Comment: The caudate nucleus shows marked gross atrophy with microscopic evidence of neuronal loss and prominent reactive astrogliosis. These findings are consistent with patient's known history of Huntington disease.
Differential diagnosis
  • Huntington phenocopies:
  • Huntington disease-like (HDL) syndromes:
    • HDL1:
      • Autosomal dominant inheritance
      • Octapeptide repeats in prion protein gene (PRNP)
      • Prominent personality changes, chorea, rigidity and dysarthria
      • Neuropathology shows basal ganglia atrophy and prion deposition without significant spongiosis
    • HDL2:
      • Autosomal dominant inheritance caused by CTG CAG triplet repeat expansion in JPH3 gene
      • Anticipation phenomenon more common if maternally inherited
      • More common among black Africans
      • Similar neuropathology to HD
    • HDL4:
      • Also classified as spinocerebellar ataxia type 17 (SCA17)
      • Caused by triplet repeat expansion in TATA box binding protein (TBP)
      • Heterogenous clinical phenotype and neuropathology
    • Dentatorubral pallidoluysian atrophy (Curr Opin Neurol 2013;26:420):
      • Secondary to expanded CAG repeat in ATN1 gene
      • Heterogenous clinical presentation
      • MRI notable for cerebellar and brainstem atrophy
      • More common in Japan
    • Neuroacanthocytosis (Lancet 2007;369:218):
      • Areflexia, raised creatine kinase and presence of acanthocytes
    • Spinocerebellar ataxias
    • Neuroferritinopathy (Curr Opin Neurol 2013;26:420):
      • Caused by mutations in FTL (ferritin light chain), resulting in an autosomal dominant basal ganglia disease similar to Huntington disease
      • MRI notable for cystic degeneration of the basal ganglia ad T2 hypointense lesions secondary to iron deposition
      • More common in Cumbrian region of northern England
  • Isolated chorea may be secondary to other causes:
    • Chorea of pregnancy
    • Systemic lupus erythematosus
    • Thyrotoxicosis
    • Polycythemia vera
    • Friedreich ataxia (Br Med Bull 2017;124:19):
      • Autosomal recessive caused by homozygous GAA triplet repeat in frataxin (FXN)
      • Gait and limb ataxia, dysarthria and loss of lower extremity reflexes
      • Progressive disease may show scoliosis, pes cavus and talipes equinovarus
      • Cardiomyopathy is common
      • Symptom onset in adolescence
Board review style question #1

A man of late middle age is found deceased by a passerby. An investigation reveals him to be a regular in the neighborhood and was known for abnormal, jerking movements and erratic obsessions. Examination of the heart demonstrated a transmural myocardial infarction. Examination of the brain revealed the gross findings shown in the image above. Which of following is the most likely diagnosis?

  1. Alzheimer disease
  2. Chronic infarct
  3. Huntington disease
  4. Parkinson disease
  5. Wernicke-Korsakoff syndrome
Board review style answer #1
C. Huntington disease. Although several of these answer choices can display global effects, the combination of motor and neuropsychiatric disturbance in a patient with striatal atrophy favors Huntington disease. Answer A is incorrect because Alzheimer disease presents with cognitive decline and memory deficits and is not associated with choreiform movements. Gross examination would reveal variable global atrophy. Answer D is incorrect because Parkinson disease is associated with cogwheel rigidity and a resting tremor but not choreiform movements. Gross examination would demonstrate pallor of the substantia nigra. Answer B is incorrect because chronic infarction would be associated with a neurologic deficit, particularly hemi or paraplegia but not choreiform movements. Gross examination would show cavitary tissue infarction within a vascular territory. Answer E is incorrect because Wernicke-Korsakoff syndrome may be associated with ataxic movements but not choreiform movements, with notable symptoms including vision difficulties and memory deficits. Gross examination would expect to find hemorrhage of the mamillary bodies or around the third ventricle.

Comment Here

Reference: Huntington disease
Board review style question #2
Which of the following trinucleotide repeats is associated with Huntington disease?

  1. CAG
  2. CGG
  3. CTG
  4. CTG CAG
  5. GAA
Board review style answer #2
A. CAG. The CAG trinucleotide repeat in the HTT gene is associated with Huntington disease. Answer C is incorrect because the CTG trinucleotide repeat in the DMPK gene is associated with myotonic dystrophy. Answer D is incorrect because the CTG CAG repeat is associated with Huntington disease-like syndrome type 2. Answer E is incorrect because the GAA trinucleotide repeat in the FXN gene is associated with Friedrich ataxia. Answer B is incorrect because the CGG trinucleotide repeat in the FMR1 gene is associated with fragile X syndrome.

Comment Here

Reference: Huntington disease

Ischemic stroke / infarct
Definition / general
  • Episode of acute neurological dysfunction caused by focal cerebral, spinal cord or retinal infarction (Stroke 2013;44:2064)
    • CNS infarction is defined as cell death due to ischemic injury in the CNS based on imaging, neuropathology or clinical assessment
Essential features
  • Brain infarcts are common and the leading cause of neurologic disability worldwide
  • Commonly associated with hypertension, cardioembolic events and atherosclerosis
  • Infarct morphology depends on the interval between stroke onset and death
    • Acute changes → blurring of gray-white matter junction (gross), edema and red neurons (micro)
    • Subacute → cracking artifact (gross), dense macrophage infiltration and neovascularization (micro)
    • Chronic → cavitated lesions (gross) with macrophages and surrounding gliosis (micro)
Terminology
  • Brain / spinal cord infarct
ICD coding
  • ICD-10: I63 - cerebral infarction
  • ICD-11:
    • 8B11 - cerebral ischemic stroke
    • 8B21 - silent cerebral infarct
Epidemiology
  • Leading cause of neurologic disability worldwide (Lancet Neurol 2019;18:459)
  • 2.8 - 5.5 million deaths worldwide per year (Lancet Glob Health 2013;1:e259, Lancet Neurol 2019;18:459)
  • Nonmodifiable risk factors include
    • Increasing age (Lancet 2014;383:245)
    • Male sex, with 133 male cases versus 99 female cases per 100,000 person years (Circ Res 2017;120:439)
    • Rare genetic causes of stroke, such as cerebral autosomal dominant arteriopathy with subcortical infarcts and leukoencephalopathy (CADASIL) or its autosomal recessive counterpart, CARASIL
  • Modifiable risk factors account for 91.5% of population attributed risk and include (Lancet 2010;376:112)
    • Hypertension, the strongest risk factor (odds ratio [OR] 3.14, 99% confidence interval [CI] 2.67 - 3.71)
    • Sedentary lifestyle, diet, obesity, smoking, psychosocial stress, depression, diabetes, high alcohol consumption, high ApoB:ApoA1 ratio, cardiac disease
Sites
  • Ischemic stroke can affect any site in the central nervous system
  • Large vessel ischemic strokes occur within the vascular territory supplied by the large artery affected
  • Large vessel strokes are commonly secondary to atherosclerosis, either with a local thrombus or embolic event (Nat Rev Dis Primers 2019;5:70)
    • Internal carotid artery is the most frequently affected in western countries
    • Intracranial atherosclerosis accounts for 30 - 50% of ischemic strokes in Asian populations
    • Aortic arch, neck and intracranial vessels are common embolic sources
    • Plaques are located predominantly at branching points, bifurcations and vessel curvatures (e.g., tip of the basilar artery, close to the bifurcation of the common carotid artery) (AJR Am J Roentgenol 2000;174:1657)
  • Arterial dissection often occurs in extracranial carotids and vertebral arteries (Nat Rev Dis Primers 2019;5:70)
  • Strokes from small vessel disease classically involve perforating arteries that supply deep subcortical (e.g., basal ganglia, thalamus) and brainstem structures (Nat Rev Dis Primers 2019;5:70)
  • Infarcts due to more global decreases in blood flow or blood pressure (hypotension, hypoperfusion) affect regions at the border between arterial territories (watershed infarcts)
    • Atherosclerosis and embolic events can contribute to watershed type infarcts (Stroke 2006;37:841)
  • Full term or mature infant brains show injury predominantly in the neocortex (i.e., laminar necrosis), including the watershed regions, basal ganglia and thalamus, subcortical white matter and brainstem (e.g., midbrain tectum) (Ann Neurol 1987;21:202, Pediatr Clin North Am 1993;40:1061)
  • In preterm or immature infant brains with hypoxic ischemic injury, the most affected regions are
Pathophysiology
  • Liquefactive necrosis is the major type of necrosis in the brain
  • Ischemic stroke results from insufficient blood flow to an area of the central nervous system, resulting in cell death
  • Blood flow can be decreased due to occlusion of a vessel, most commonly from thromboembolic events (Nat Rev Dis Primers 2019;5:70)
    • Atherosclerosis: include in situ arterial obstruction of blood flow or production of embolic fragments derived from unstable atherosclerotic plaques
      • Emboli travel to distal vascular territories, obstruct the blood supply and cause ischemia; necrosis ensues when the collateral blood supply is insufficient to meet the tissue's metabolic demands (e.g., end arteries)
      • Atherosclerotic occlusion in the parent vessel of a perforator artery can be a cause of lacunar infarct (Stroke 2010;41:2822)
    • Cardiac disease: atrial fibrillation, flutter, patent foramen ovale, infective endocarditis, hypokinetic heart segment with associated thromboembolism
      • Atrial fibrillation and flutter cause blood to stagnate in the atria, inducing clots and thromboembolism with downstream obstruction of CNS vessels
  • Mild to moderate ischemia or short duration may result in selective neuronal necrosis
  • Severe or long duration of the ischemic insult results in necrosis, irrespective of cell type
  • Impaired cerebral flow autoregulation in preterm infants is closely associated with periventricular leukomalacia and intraventricular hemorrhage, the most common forms of preterm encephalopathies (Pediatrics 2000;106:625)
    • In preterm infants, the timing and severity of the ischemic insults along pregnancy are major determinants of the neuropathological picture and clinical outcomes (Clin Perinatol 2014;41:1)
Etiology
  • Most frequently associated with atherosclerosis, cardiac pathology and hypertension (Nat Rev Dis Primers 2019;5:70)
  • Less common causes of ischemic infarcts include
    • Vasculitis
    • Coagulation disorders
    • Arterial dissection
    • Reversible vasoconstriction syndrome
    • HIV associated arteriopathy
    • Moyamoya disease
    • Fat embolism
  • Ischemic insults in utero are the most common cause of cerebral injury among preterm / very low body weight infants (Clin Perinatol 2014;41:1)
Diagrams / tables

Images hosted on other servers:
Stroke pathophysiology spatiotemporal relationships

Stroke pathophysiology spatiotemporal relationships

Cell pathophysiology summary

Cell pathophysiology summary

CNS vascular territories

CNS vascular territories

Clinical features
  • Mostly dependent on the location of the ischemic brain tissue
  • Common symptoms include
    • Contralateral hemiparesis (unilateral weakness of the upper and lower limbs, opposite to the site of the infarct)
    • Hemianesthesia (absence of unilateral sensations)
    • Aphasia (inability to understand or to express oral language; lesion at the dominant hemisphere)
  • Different clinical syndromes are associated with the occlusion of major cerebral arteries (QJM 2013;106:607)
  • Middle cerebral artery: presentation depends on whether the ischemic lesion affects the dominant or nondominant hemisphere
    • Dominant hemispheric lesions: aphasia, motor and sensory alterations; complete hemiplegia if internal capsule is involved
    • Nondominant hemispheric lesions: neglect, anosognosia, motor and sensory deficits, homonymous hemianopia
  • Anterior cerebral artery: motor or sensory deficit (leg > face, arm), primitive reflexes (i.e., grasp and sucking, abulia, gait apraxia)
  • Posterior cerebral artery: homonymous hemianopia, alexia without agraphia (dominant hemisphere), visual hallucinations, sensory loss, choreoathetosis, spontaneous pain in thalamic lesions
  • Vascular dementia: any type of dementia that results from cerebrovascular lesions (i.e., infarcts)
    • 2 main clinical syndromes are recognized (Clin Sci (Lond) 2017;131:1059)
      • Poststroke vascular cognitive impairment
        • Cognitive alterations as an immediate result of a known stroke
      • Vascular cognitive impairment without recent stroke
        • Cognitive impairment is the result of infarcts that were not detected in the acute setting and tissue injury is documented by imaging or neuropathology
        • No alternative causes of dementia explain the observed cognitive decline
Diagnosis
  • Diagnosis of ischemic stroke is made by clinical and imaging findings
    • Noncontrast CT
      • Primary imaging modality for acute assessment in most centers (Lancet 2018;392:1247)
      • Rapid, accessible and highly sensitive diagnosis of intracranial hemorrhage (major contraindication for recanalization therapy)
      • Sensitivity is suboptimal (64%) for early ischemic infarct signs in lesions that are < 6 hours from onset (Radiology 2001;219:95)
    • CT angiography
    • MRI
      • Diffusion weighted MRI: most accurately measures the infarct core (i.e., nonsalvageable tissue) (Radiology 2009;251:627)
      • Perfusion MRI
    • PET
      • Gold standard for viable (i.e., penumbra) identification
      • Replaced by diffusion weighted imaging due to long processing time and radioactivity exposure (Front Cardiovasc Med 2022;9:861913)
    • Vascular dementia
      • Diagnostic criteria for neuropathological validation are lacking
      • Features of vascular lesions involved in development of dementia are number, volume, location and origin (Acta Neuropathol 2016;131:659)
Laboratory
  • Coagulation tests (i.e., aPTT, PT, TT) are relevant to rule out coagulation disorders (contraindication for thrombolysis)
Radiology description
  • CT unenhanced
    • Early / acute signs: loss of definition between gray and white matter, cortical hypoattenuation and effacement of the sulci (Radiology 2005;235:444)
    • Loss of the insular cortex in acute middle cerebral artery (MCA) infarcts (Radiology 1990;176:801)
    • Hyperdensity of the MCA (i.e., hyperdense artery sign) is observed in 30 - 40% of patients with acute MCA infarcts (Stroke 1992;23:317)
    • Chronic infarcts show cavitation sometimes with mineralization
  • MRI
    • Diffusion weighted imaging (DWI): hyperintense signal in the acutely infarcted tissue; low signal on apparent diffusion coefficient (ADC) map
    • Infarcts are often T2 hyperintense
    • Collateral flow on FLAIR: linear or serpentine hyperintensities distal to the site of obstruction (Neurology 2009;72:1134)
    • Perfusion weighted imaging (PWI) - DWI mismatch: used to estimate the salvageable tissue (i.e., penumbra) in acute infarct (Neuroimaging Clin N Am 2021;31:177, AJNR Am J Neuroradiol 2015;36:32)
      • Ischemic areas on PWI are compared with areas shown by DWI (i.e., the infarct core or nonsalvageable tissue); the mismatch represents the volume of salvageable tissue with reperfusion therapy
    • Chronic infarcts show absence of contrast enhancement in strokes older than 4 months (Radiographics 2012;32:1285)
Radiology images

Contributed by Javier Redding-Ochoa, M.D. and Thomas Zaikos, M.D., Ph.D.
Recent infarcts (T1 postcontrast MRI)

Recent infarcts (T1 postcontrast MRI)

Recent infarcts (diffusion weighted MRI)

Recent infarcts (diffusion weighted MRI)

Recent infarcts (T2 FLAIR MRI)

Recent infarcts (T2 FLAIR MRI)

Remote infarct (MRI FLAIR)

Remote infarct (MRI FLAIR)

Prognostic factors
Case reports
Treatment
  • 2 common forms of treatment for acute ischemic stroke available if within 6 - 8 hours of onset (Neurologia 2014;29:102)
    • Systemic thrombolysis
    • Thrombectomy
  • Neuroimaging guides decision making for thrombolysis in patients with an unknown time of onset
Gross description
  • Time between stroke and death is a major determinant of the gross features
    • Completely accurate dating of an infarct based on the gross features is not possible, especially cavitated lesions
  • < 8 hours to 48 hours
    • Undetectable if the infarct occurred < 8 hours before death
    • Congestion of gray matter and edema (Histopathology 2011;58:333)
      • Dusky discoloration and blurring of the gray-white matter junction
      • Ill demarcated borders
  • Reperfusion of an ischemic lesion where vessels are also affected can become hemorrhagic (Stroke 1986;17:586)
    • Hemorrhagic discoloration can be limited to a specific vascular territory
    • Commonly observed as secondary infarcts in the setting of brain herniations (e.g., obstruction of pericallosal artery in subfalcine herniation, obstruction of posterior cerebral artery in uncal herniation)
  • 2 days to a few months (Histopathology 2011;58:333)
    • Marked tissue softening
    • Cracking artifact: demarcates the necrotic area
    • Tissue edema (e.g., midline shift, narrowing of the sulci, convexity flattening)
    • Dusky discoloration and blurring of gray-white matter junction
    • Cerebellar infarcts show effacement of the folia
  • Chronic (months - years)
    • Cavitation (i.e., cystic infarct)
    • Thin cortical remnant overlying the cavitation
    • In long term survivors of severe global hypoxic ischemic encephalopathy, a markedly thin cortex can be observed due to laminar necrosis
    • Lateral ventricle asymmetry can be seen (i.e., ex vacuo ventricular dilation of the affected hemisphere)
    • Anterograde loss white matter volume due to an ischemic insult (e.g., atrophy of the ipsilateral cerebral peduncle and pyramid after an MCA infarct with degeneration of the corticospinal tract), ipsilateral brainstem atrophy after extensive supratentorial stroke (corticospinal tract and frontopontine fiber degeneration) and subsequent contralateral cerebellar atrophy due to transsynaptic degeneration (AJNR Am J Neuroradiol 2008;29:354, Eur Radiol 2006;16:592)
    • Postinfarct anterograde white matter loss of volume occurs due to Wallerian degeneration or transsynaptic degeneration (AJR Am J Roentgenol 1998;171:813)
  • Lacunar infarct: arbitrarily defined as cystic infarcts < 10 mm in length
    • Predominantly found at basal ganglia, internal capsule, pons
  • Binswanger disease
    • White matter structures, including corpus callosum, corona radiata, internal capsule and anterior commissure shows marked loss of volume, discoloration, softening and granular texture predominating in the periventricular areas (Neurology 1995;45:626, J Neurol Sci 2010;299:9)
    • Lacunar or large infarcts are frequently observed
    • Cerebellar white matter is also commonly affected
    • Associated with vascular dementia
Gross images

Contributed by Javier Redding-Ochoa, M.D., Koping Chang, M.D. and Olga Pletnikova, M.D.
Subacute infarct

Subacute infarct

Bilateral occipital infarcts

Bilateral occipital infarcts

Chronic infarct

Chronic infarct

Lacunar infarct

Lacunar infarct

Chronic ACA infarct

Chronic ACA infarct

Watershed infarcts

Watershed infarcts

Microscopic (histologic) description
  • Acute infarct (1 - 4 days) (Histopathology 2011;58:333)
    • Neuronal changes: hypereosinophilic perikaryon (red dead neurons), cell body shrinkage, pyknosis (i.e., nuclear hyperchromasia), loss of demarcation of the nuclear features in later phases
      • Neurons are more susceptible to ischemia than glia
    • Neuropil vacuolation (i.e., tissue edema)
    • May have some neutrophilic infiltrate
    • Pannecrosis: all cell populations (i.e., neurons, glia, blood vessels) are necrotic; dead cells remain visible as hypereosinophilic structures that preserve the cell and nuclear outlines (pale neurons or "ghosts")
    • Laminar necrosis
      • Occurs due to variable susceptibility to hypoxia among the cortical histological layers
      • In the mature brain, the most vulnerable neurons are located in cortical layers III, V, VI, Purkinje cells in the cerebellum and pyramidal cells of CA1 field in the hippocampus proper
    • Geographic necrosis: liquefied necrotic tissue is well demarcated from the adjacent viable tissue
  • Subacute (5 - 14 days)
    • Dense macrophage infiltration and scattered siderophages
    • Variable neutrophilic infiltration
    • Peripheral reactive astrocytosis and microglial activation (i.e., rod shaped microglia)
    • Hypereosinophilic neurons are still present in gray matter
    • Neovascularization of necrotic tissue and reactive endothelial cells
  • Chronic (15 days - years)
    • Cavitated lesion with vessels and macrophages surrounded by a glial scar
    • Reactive astrocytes in the edge of the cavitation
    • Can be piloid gliosis with Rosenthal fibers, particularly in brainstem infarcts
    • Scattered hemosiderin laden macrophages typically present
  • Axonal balloons can occur in all phases of ischemic injury (i.e., dilation of axons indicate injury with subsequent defective axonal transport)
  • Neuronal ferrugination occurs occasionally in subacute to chronic infarcts, characterized by perykaryal mineralization (i.e., prominent basophilia)
  • Vascular dementia
    • Diagnosis is based on the absence of a primary neurodegenerative disease known to cause dementia (e.g., Alzheimer disease, Lewy body dementia)
    • Multivariable model based likelihood showed that the presence of 1 or 2 microinfarcts on routine neuropathological sections indicates a burden of hundreds of microinfarcts in that brain (Neurology 2013;80:1365)
  • Binswanger disease
    • Multiple infarcts, typically chronic, with associated white matter rarefaction, myelin loss, loss of oligodendroglia and reactive gliosis
    • Subcortical U fibers are usually spared
    • Penetrating arteries' walls are markedly thickened and hyalinized with enlarged perivascular spaces
    • CAA and CADASIL are uncommonly associated as the underlying cause of Binswanger disease (Neurology 1995;45:626, J Neurol Sci 2010;299:9)
  • Reference: J Neuropathol Exp Neurol 1993;52:481
Microscopic (histologic) images

Contributed by Javier Redding-Ochoa, M.D.
Geographic necrosis

Geographic necrosis

Acute infarct

Acute infarct

Red neurons

Red neurons

Acute infarct with focal hemorrhage

Acute infarct with focal hemorrhage

Subacute infarct

Subacute infarct


Endothelial reaction (subacute infarct)

Endothelial reaction (subacute infarct)

Lacunar infarct

Lacunar infarct

Macrophage infiltration

Macrophage infiltration

Neuronal ferrugination

Neuronal ferrugination

Axonal balloons

Axonal balloons

Positive stains
  • CD68: highlights macrophages and microglia in subacute and chronic infarcts
  • GFAP: positive in reactive astrocytes
  • CD61: highlights platelets and thrombi in blood vessels (Histopathology 2011;58:333)
  • Neurofilament protein: highlights axonal spheroids in the vicinity of the lesion indicating axonal injury; shows loss of staining in the infarcted areas
Negative stains
  • Luxol fast blue: shows loss of myelination associated with the necrotic area; subacute infarcts show loss of axons in the affected area (see demyelination in Differential diagnosis)
  • Special stains and immunostains for infectious etiologies are negative
Videos

Brain old infarcts versus cerebral abscess

Brain infarcts tutorial

Sample pathology report
  • Brain, autopsy:
    • Acute infarct, 10.5 x 6 x 4 cm, left frontal, temporal, parietal lobes (see comment)
    • Gross description: External examination of the cerebrum shows a 10.5 x 6 cm area with marked vascular congestion, narrowing of the sulci and softening extends from the left frontal lobe to the superior temporal lobe and parietal lobe. The area of softening spares the superior frontal gyrus. At the base of the brain, severe atherosclerosis is observed in the left middle cerebral artery with > 90% of stenosis.
    • On coronal sections, this area is characterized by mild dusky discoloration in the cortical ribbon and in the subjacent white matter with blurring of the gray-white matter junction measuring up to 4 cm in depth. The adjacent left lateral ventricle shows decreased caliber compared with the right lateral ventricle.
    • Microscopic description: Low power examination of the frontal, parietal and temporal cortices show marked vacuolation of the neuropil and pericellular retraction artifact. On high power, the neurons are remarkable for cell shrinkage, hypereosinophilic perykarya and loss of demarcation of the nuclear contours. The subjacent white matter is pale with vacuolization and poorly demarcated from the gray matter.
    • Comment: The observed distribution of the infarct corresponds to the left MCA vascular territory.
Differential diagnosis
  • Putrefaction / "Swiss cheese" artifact:
    • Foul smell
    • Multiple cavitations not limited to vascular territories and commonly in deep gray matter
    • Bacterial colonies without accompanying inflammation
  • Cerebral contusions:
    • History of trauma
      • Coup / countrecoup lesions are primarily located in the orbitofrontal cortices, frontal and temporal poles
    • Wedge shaped hemorrhagic lesions with the widest area oriented to the pial surface
    • Old contusions: brown, orange (i.e., hemosiderin) discoloration in the rim of the lesion, no superficial cortical remnant
  • Abscess:
    • Typically gray-white matter junction
    • Well defined borders often with fibrotic rim
    • Necrotic areas have abundant acute and chronic inflammation
    • Infectious organisms may be present
  • Infiltrating glioma:
    • Not in the distribution of a vascular territory
    • Ill defined lesions which expand and distort brain anatomy
    • Hypercellular glial lesion on histology without signs of neuronal ischemic changes
    • Mitotic activity and glomeruloid vascular proliferation common in high grade gliomas
  • Demyelination (e.g., multiple sclerosis):
    • Well demarcated foci of white matter discoloration
    • Classically seen around the ventricular system, though Luxol fast blue staining can show demyelination affecting white and gray matter
    • Not limited to a specific vascular territory
    • Microscopic features depend on the plaque's state of activity
      • Inactive: pallor, loss of oligodendrocytes, reactive astrocytes
      • Active: venules with mononuclear perivascular cuffing and relative preservation of axons within the lesion
Board review style question #1

Which morphologic feature is associated with acute irreversible neuronal ischemia?

  1. Cell swelling
  2. Hypereosinophilic cytoplasm and nuclear hyperchromasia
  3. Karyorrhexis
  4. Mineralization (ferrugination)
Board review style answer #1
B. Hypereosinophilic cytoplasm and nuclear hyperchromasia. The earliest morphological abnormalities observed by H&E are hypereosinophilic cytoplasm and loss of nuclear normal staining features. In the earliest phase of the infarct, the nuclei are hyperchromatic and pyknotic. In later stages, the nuclear chromatin disappears and the nuclei become eosinophilic, giving the impression that it merges with the surrounding eosinophilic cytoplasm. These changes can be observed also with hypoglycemia or excitotoxicity. Answer D is incorrect because mineralization is more associated with subacute to chronic changes in an infarct, rather than acute. Answer A is incorrect because while cell swelling can occur in the setting of acute ischemia, this change is not specific and when it occurs in ischemia it is potentially reversible. Answer C is incorrect because karyorrhexis is not associated with acute neuronal ischemia in adults, though it can be seen in ischemic injury in immature or premature fetal and neonatal acute infarcts.

Comment Here

Reference: Ischemic stroke / infarct
Board review style question #2
Which of the following are the most common locations for lacunar infarcts?

  1. Basal ganglia and pons
  2. Frontal lobe and hippocampus
  3. Medulla and cerebellum
  4. Spinal cord and mamillary bodies
Board review style answer #2
A. Basal ganglia and pons. Lacunar infarcts, defined by < 10 mm in length, are most commonly seen in the diencephalic regions (i.e., basal ganglia, thalamus) and in the brainstem, particularly in the pons and deep white matter (e.g., the internal capsule). These regions share in common that penetrating arteries supply the blood flow in these territories and hypertensive arteriopathy produces damage frequently in penetrating arteries and arterioles. Answers B, C and D are incorrect because although these sites may show lacunar infarcts, they are observed less frequently than lacunar infarcts in the basal ganglia and pons.

Comment Here

Reference: Ischemic stroke / infarct

Leukodystrophies
Definition / general
  • Leukodystrophy generally refers to a genetic disorder that affects white matter
  • Most often present in children, can occur in adulthood
  • Demyelination often progresses in occipital to frontal manner and there is often sparing of the U fibers (short association fibers, Wikipedia: Association Fiber)
  • Can result from a wide range of genetic defects involving formation, maintenance and breakdown of myelin
  • Most often classified based on etiology: peroxisomal, lysosomal and other
    • Peroxisomal: adrenoleukodystrophy and neonatal adrenoleukodystrophy
    • Lysosomal: Krabbe disease and metachromatic leukodystrophy
    • Other: Alexander disease, Canavan disease, vanishing white matter disease, others
Pathophysiology
  • Peroxisomal:
    • Adrenoleukodystrophy: X linked, single enzyme defect in ATP binding transporter, leads to reduced capacity to form coenzyme A derivative of very long chain fatty acids
    • Neonatal adrenoleukodystrophy: autosomal recessive, defective peroxisome assembly leads to decreased numbers of peroxisomes
  • Lysosomal:
    • Krabbe disease: autosomal recessive, galactocerebroside β-galactosidase deficiency
    • Metachromatic leukodystrophy: autosomal recessive (rarely autosomal dominant), aryl sulfatase A deficiency, rarely SAP-1
  • Other:
    • Alexander disease: sporadic, gain of function mutation of GFAP leads to Rosenthal fiber accumulation
    • Canavan disease: autosomal recessive, defective aspartoacylase activity
    • Vanishing white matter disease: autosomal recessive, mutation of genes involved in translation of factor EIF-2B
Clinical features
  • Varies depending on type and age at presentation
  • In infancy: often includes motor disabilities, macrocephaly, seizures or spasticity and developmental failure
  • In childhood: ataxia, vision changes, behavioral / educational issues
  • In adulthood: behavioral changes, psychosis, spasticity
Radiology description
Radiology images

Images hosted on other servers:
Missing Image

Marked loss of posterior white matter

Case reports
Treatment
Gross description
  • Coronal autopsy specimens often show unaffected gray matter, spared subcortical U fibers and either firm (adrenoleukodystrophy), chalky white matter (Krabbe, metachromatic leukodystrophy) or markedly softened white matter (vanishing white matter disease, Canavan disease)
Gross images

Images hosted on other servers:
Missing Image

Krabbe disease

Missing Image

Vanishing white matter disease

Microscopic (histologic) description
  • Peroxisomal:
    • Adrenoleukodystrophy: severe demyelination in cerebral white matter, optic nerves and internal capsule, yet sparing of U fibers
    • May see lesions of different ages: new lesions with large aggregates of macrophages with demyelination and perivascular lymphoid aggregates; older lesions with gliotic scar and no macrophage activity
    • Adrenal glands show enlarged "ballooned" eosinophilic cells with striated cytoplasm
  • Lysosomal:
    • Krabbe disease: pathognomonic PAS+ "globoid macrophages" (may be multinucleated giant cells), extensive myelin and oligodendrocyte loss, reactive astrocytic gliosis
    • Metachromatic leukodystrophy: accumulation of PAS and Luxol fast blue / LFB+ macrophages that show brown metachromasia with acidified cresyl violet, toluidine blue or thionine on frozen sections
    • Also extensive myelin and axonal loss in white matter and corticospinal tracts
    • Deposition of metachromatic sulfatides can also be seen in basal ganglia, numerous deep nuclei and peripheral nerves
  • Other:
    • Alexander disease: abundant Rosenthal fibers (especially perivascular, subpial and periventricular) with diffuse demyelination of white matter
      • Rosenthal-like fibers can be seen in cell bodies of astrocytes
    • Canavan disease: white matter vacuolation and demyelination predominantly at gray white junction; "spongiform leukodystrophy"
    • Vanishing white matter disease: significant cavitation of white matter with excessive oligodendrocytes in pericavity residual white matter
Microscopic (histologic) images

Images hosted on other servers:

Peroxisomal
Missing Image

X linked adreno-leukodystrophy: gliosis and inflammation


 Lysosomal:
Missing Image

Krabbe disease: globoid cells

Missing Image

Metachromatic leukodystrophy: loss of myelin

Missing Image

Metachromatic
leukodystrophy:
brown metachromasia
in peripheral nerve



Other:
Missing Image

Pelizaeus-Merzbacher disease: loss
of myelin

Missing Image

Pelizaeus-
Merzbacher
disease: cerebellar
degeneration

Missing Image

Alexander disease, Rosenthal
fibers

Missing Image

Canavan disease

Positive stains
  • Demyelination is highlighted by loss of Luxol fast blue (LFB), often (but not always) with spared axons demonstrated by neurofilament protein and accompanied by macrophages
  • Macrophages: CD68+, also positive for PAS in Krabbe and metachromatic leukodystrophy
  • Reactive gliosis highlighted by GFAP
Electron microscopy images

Images hosted on other servers:

Peroxisomal:
Missing Image

X linked adreno-leukodystrophy: trilamellar lipid products


 Lysosomal:
Missing Image

Metachromatic leukodystrophy: storage of sulfatides

Differential diagnosis

Meckel-Gruber syndrome
Definition / general
  • Meckel-Gruber syndrome (Meckel syndrome or MKS) is a rare, lethal ciliopathic genetic disorder characterized by abnormalities affecting several organ systems
  • Three classic symptoms are normally associated with MKS: occipital encephalocele, polycystic kidneys and polydactyly
  • Affected children or fetuses may also have abnormalities affecting the head and face, liver, lungs, heart and genitourinary tract
  • Meckel-Gruber syndrome is named for Johann F. Meckel and George B. Gruber; the first reports of the syndrome were published in 1822 by J. F. Meckel; G. B. Gruber also published reports in 1934 and gave it the name dysencephalia splanchnocystica
Essential features
    The three phenotypic features of the classical triad are:
    • Protrusion of a portion of the brain and its surrounding meninges through a defect in the back of the skull (occipital encephalocele)
    • Multiple cysts on the kidneys (cystic kidneys)
    • Extra fingers or toes (polydactyly)
Terminology
  • Meckel-Gruber syndrome, Meckel syndrome, Gruber syndrome, dysencephalia splanchnocystica, MES, MKS
Epidemiology
  • Prevalence is estimated at 1/50,000 births in Europe; worldwide prevalence is reported to be 1/13,250 to 1/140,000 live births; the live birth prevalence is significantly higher in the Finnish population (1/9,000); therefore this syndrome is suggested to be a Finnish heritage disease (J Pediatr Neurosci 2013;8:154)
  • No gender or ethnic predilection is reported
Pathophysiology
  • Failure of mesodermal induction has been suggested to cause Meckel-Gruber syndrome; the induction cascades of early morphogenesis involve numerous growth factors, homeobox genes and paired domain genes
  • It belongs to the ciliopathies, a category of diseases thought to be caused by dysfunction of cilia and flagella; polycystic liver and kidney disease, Bardet-Biedl syndrome, Alstrom syndrome and Joubert syndrome also belong to the same group (Annu Rev Genomics Hum Genet 2006;7:125)
Etiology
  • MKS is inherited as an autosomal recessive condition through thirteen genes: B9D1, B9D2, CC2D2A, CEP290, MKS1, RPGRIP1L, TCTN2, TCTN3, TMEM67, TMEM107, TMEM216, TMEM231 and TMEM237 (Eur J Hum Genet 2016;24)
  • It has a recurrence risk of 25%; genetic counseling should be provided to affected families
Clinical features
  • A study by Barisic et al, using data from 191 cases of MKS, as accessed through the European Surveillance of Congenital Anomalies (EUROCAT) network, found the prevalence of various characteristics of the disorder to be as follows (Eur J Hum Genet 2015;23:746):
    • Polycystic dysplastic kidneys (97.7%)
    • Polydactyly (87.3%)
    • Occipital encephalocele (83.8%)
    • Fibrotic / cystic changes of the liver (65.5%)
    • Other central nervous system (CNS) anomalies (51.4%): the features include hydrocephalus, anencephaly, holoprosencephaly, the absence of olfactory lobes as well as Dandy-Walker and Arnold-Chiari malformation
    • Orofacial clefts (31.8%): cleft lip and palate, microphthalmia and micrognathia may be observed
    • In addition cardiac malformations including atrial septal defect, aorta coarctation, patent arterial duct and valvular pulmonary stenosis may be present; genital ambiguity secondary to incomplete development of internal and external genitalia, and cryptorchidism in males are common; urothelial atresia and bone dysplasia are rare features; bowing or shortening of the limbs are also present
Diagnosis
  • Finding at least two of the three phenotypic features of the classical triad, in the presence of normal karyotype, makes the diagnosis solid; improvements in ultrasonography have enabled prenatal diagnosis as early as 10 weeks' gestation
  • Alpha-fetoprotein (AFP) level from either maternal blood or amniotic fluid may help detect encephalocele; AFP can be measured in amniotic fluid after 12 weeks' gestation and in maternal blood after 15 weeks' gestation
  • If anomalies are detected early in the first trimester, chorionic villus sampling (CVS) can be performed at 10-12 weeks' gestation or later in pregnancy if oligohydramnios does not permit amniocentesis; molecular genetic testing can be used to to assess prenatal diagnosis, linkage as well as carrier testing
  • The level of amniotic fluid may be significantly altered or remain normal, but a normal level of fluid should not be criteria for exclusion of diagnosis
Radiology description
  • Fetal ultrasonography can detect an occipital encephalocele, polydactyly and dysplastic kidneys in fetuses with Meckel-Gruber syndrome if oligohydramnios is not present
  • MRI is a valuable complement to ultrasonography in assessing fetal anomalies in the presence of severe oligohydramnios; the level of amniotic fluid may be significantly altered or remain normal
  • Color Doppler can assess lung perfusion in the last trimester, detect the presence of renal arteries in cases of oligohydramnios (suspected renal agenesis or hypoplasia), and assess flow in the umbilical arteries
Prognostic factors
  • Fetuses affected by MKS survive only a few days to a few weeks or die in utero; pulmonary hypoplasia secondary to oligohydramnios is the leading cause of death
  • Other complications are renal and liver failure
Treatment
  • No treatment is currently available; the outcome is always fatal
Gross description
  • Occipital encephalocele is characterized by extrusion of rhombic roof elements, cerebellar vermis, caudal third ventricle and distended fourth ventricle through a widened posterior fontanelle
  • Microscopic cysts cause the kidney to enlarge 10 to 20 times
  • Polydactyly may affect all four extremities and is typically postaxial or very rarely preaxial
Gross images

Images hosted on other servers:

Cystic renal dysplasia

Pulmonary hypoplasia

Microscopic (histologic) description
  • The primary renal abnormality appears to be failed interaction of the metanephric duct and renal blastema; the kidneys, therefore, show little corticomedullary differentiation, and the nephrons are severely deficient, causing enlargement of the kidneys; thin walled cysts appear throughout the parenchyma
  • Hepatic dysgenesis and liver fibrosis are identified only at postmortem examination; in Meckel-Gruber syndrome, the plates do not atrophy and prevent reorganization by the remaining biliary cells to form tubular ducts; the resultant fibrosis can be severe enough to occlude portal veins; bile canaliculi are smaller and less well developed and have inspissated bile within the abnormal ductules
Microscopic (histologic) images

Images hosted on other servers:

Cystic renal dysplasia

Differential diagnosis

Meningitis - bacterial
Definition / general
  • Acute inflammation involving the leptomeninges (particularly the arachnoid / subarachnoid space) secondary to the presence of bacteria within cerebrospinal fluid
Terminology
  • Synonyms include "acute meningitis," "pyogenic meningitis" and "purulent meningitis"
Epidemiology
  • As vaccines are developed and employed, the prevalence of acute bacterial meningitis has decreased and the epidemiology of inciting microorganisms has changed in proportion to successful vaccine deployment
  • Currently, the most prevalent microorganisms responsible for acute bacterial meningitis are group B Streptococcus (in infants less than 2 months of age) and Streptococcus pneumoniae in all other age groups, with the exception of 11 - 17 year olds, where Neisseria meningitidis still prevails
  • Haemophilus influenzae is still occasionally encountered in nonvaccinated individuals and Listeria monocytogenes comprises a few percent of cases (N Engl J Med 2011;364:2016, CMAJ 2012;184:1060, Public Health Rep 2010;125:129, Arch Dis Child 2012;97:993)
Sites
  • By definition, acute inflammation resides within the subarachnoid space
  • Inflammation of the dura mater ("pachymeningitis") is not included and is rather considered a separate entity as it occurs in distinctly different clinical circumstances and demonstrates distinct clinical features and disease associations
Pathophysiology
  • Critical advances in the understanding and treatment of acute bacterial meningitis follow the recognition that proteins within the bacterial cell wall are responsible for inciting the acute inflammatory response, even in the absence of viable bacteria
  • Although many molecules may be involved, lipopolysaccharide in the walls of gram negative organisms and techoic acid in the walls of gram positive microorganisms activate brain microglia, leading to a cascade of inflammatory changes resulting in cortical microvascular permeability with diffuse cerebral edema, resulting in increased intracranial pressure
  • Recognition of this inflammatory cascade has lead to the use of less bacteriolytic antibiotics as well as concomitant high dose corticosteroid administration during the acute phases of bacterial meningitis (J Clin Invest 2007;117:2004, Arq Neuropsiquiatr 2012;70:366)
Etiology
  • Organisms responsible for community acquired bacterial meningitis are discussed above
  • Bacterial meningitis following penetrating head trauma (either accidental or neurosurgical) are often caused by gram negative bacteria including Escherichia coli and Klebsiella pneumoniae
Clinical features
  • Classic clinical features include fever, headache, stiff neck and a variety of illicit reflexes secondary to meningeal irritation; with increased intracranial pressure, decreased cerebral perfusion may rapidly lead to loss of consciousness and death
  • Delayed therapy may also result in vascular inflammation with cerebral infarction (Cleve Clin J Med 2012;79:393)
Diagnosis
Laboratory
  • Comparison of normal cerebrospinal fluid cell count and chemistry with that generally encountered in patient with acute bacterial meningitis

    Condition Normal Bacterial meningitis
    WBC (/ul) < 5 > 500 (mostly PMNs)
    Protein (mg/dl) < 45 > 100
    Glucose (mg/dl) > 45 < 40
    CSF/serum glucose > 0.6 < 0.4

  • As cerebrospinal fluid glucose levels are dependent on circulating serum glucose levels, the CSF to serum glucose ratio is felt to be a more reliable parameter for the diagnosis of acute bacterial meningitis than absolute CSF glucose levels
  • Although the detection of acute meningitis is fast and reliable, molecular testing for specific microorganisms varies in both sensitivity and specificity
  • Therefore clinical vigilance must remain high during the treatment of this life threatening process (Arq Neuropsiquiatr 2013;71:693, Pediatrics 2010;126:62)
Radiology description
  • Acute bacterial meningitis usually presents with striking effacement of cerebral sulci and ventricular spaces secondary to diffuse cerebral edema
  • In addition, meningitic labyrinthitis on MRI predicts patients likely to develop postmeningitic hearing loss (Arch Otolaryngol Head Neck Surg 2011;137:441)
  • In patients who survive untreated, subsequent basal ganglia infarcts occur secondary to infectious vasculitis
Prognostic factors
  • Survival depends on prompt recognition of acute bacterial meningitis, followed by institution of appropriate antibiotic therapy
  • Early administration of high dose corticosteroid therapy reduces cerebral edema and improves survival
  • Approximately 15% of patients die from acute bacterial meningitis
  • Those surviving may develop post meningitic complications such as hydrocephalus and hearing loss (see "Radiology" above) (Mayo Clin Proc 2009;84:403, Pediatrics 2012;130:e8)
Case reports
Treatment
Gross images

Contributed by Mark Cohen, M.D.

Streptococcus - dilated cerebral veins associated with the thick purulent exudate within the leptomeninges

Tuberculous meningitis

Differential diagnosis
  • Clinical differential diagnosis of acute bacterial meningitis is broad
  • However, with typical imaging and cerebrospinal fluid cell counts and chemistries, the major differential diagnosis revolves around identification of the inciting organism
  • Poor initial response to antibiotics should raise suspicion of a nonclassical bacterial etiology, as well as the possibility of acute meningitis secondary to a nonbacterial organism, such as a fungus or protozoa

Methanol
Definition / general
  • Methanol is a clear, colorless alcohol that is commonly used in industrial products such as antifreeze and windshield wiper fluid; also perfumes, other solvents, commercial formaldehyde and illegally made alcohol
  • Toxicity occurs due to intentional overdose (as a substitute for ethanol) or accidental ingestion
Epidemiology
  • Approximately 1,000 - 2,000 cases of methanol poisoning are reported in the United States each year
Sites
  • Poisoning affects putamen and optic nerves
Clinical features
  • Headache, abdominal pain, nausea, vomiting, mental status changes
  • Loss of vision with hyperemia of the optic discs and a reduced papillary response to light
Laboratory
  • Elevated serum methanol levels
  • Metabolic acidosis: decreased total CO2 with increased anion gap
Radiology description
  • Bilateral necrosis of putamen with or without hemorrhage
  • Contrast enhancement of the retrobulbar segment of the optic nerves
Radiology images

Images hosted on other servers:
Missing Image

Hypodensity in bilateral putamen

Missing Image

Bilateral symmetrical hypodensities

Missing Image

Involves bilateral frontal lobes

Missing Image

Lesions show restriction of diffusion

Prognostic factors
  • Permanent visual impairment occurs in some survivors of methanol poisoning
Case reports
Treatment
  • Delays methanol metabolism until methanol is eliminated from body either naturally or via dialysis
  • Ethanol
  • Fomepizole: inhibitor of alcohol dehydrogenase
Gross description
  • Hemorrhagic necrosis of putamina
Gross images

Contributed by Kymberly A. Gyure, M.D.
Missing Image

Methanol

Microscopic (histologic) description
  • Degeneration of retinal ganglion cells
Differential diagnosis

Methotrexate toxicity
Definition / general
  • Chemotherapeutic agents, either alone or combined with radiation therapy, may lead to cognitive decline with an associated leukoencephalopathy
  • Most common offending agent is methotrexate, particularly when it is administered intrathecally
Terminology
  • Term disseminated necrotizing leukoencephalopathy (DNL) has been applied to the progressive and fatal form of neurologic disease induced by combined intrathecal methotrexate administration and radiation (Cancer 1975;35:291)
Epidemiology
  • Methotrexate is used predominantly to treat leukemia and lymphoma, including primary CNS lymphoma
  • Patients treated for childhood leukemia are at particular risk for developing methotrexate related leukoencephalopathy
Sites
  • Predominantly white matter
Pathophysiology
  • Methotrexate is a folic acid antagonist which inhibits DNA synthesis
  • It also indirectly inhibits the synthesis of methionine, which is necessary for the formation and maintenance of myelin, leading to vacuolar degeneration of white matter (Acta Neuropathol 1989;78:291)
  • Most individuals with severe neurologic complications secondary to methotrexate also have a history of radiation therapy and a synergistic effect of methotrexate and radiation is considered to be likely to lead to serious damage to white matter
Clinical features
  • Cognitive decline, decrease in IQ
Radiology description
  • Cerebral atrophy, decreased density of white matter on CT scans, diffuse periventricular hyperintensities on T2 weighted MR images and calcifications in the basal ganglia
Radiology images

Images hosted on other servers:
Missing Image

Toxic leukoencephal-opathy

Missing Image

Neurotoxicity

Case reports
Treatment
  • There is currently no effective treatment for methotrexate related leukoencephalopathy
Gross description
  • Discrete, gray brown foci of friable white matter with associated edema
Microscopic (histologic) description
  • Vacuolation or necrosis, axonal swellings (spheroids), macrophage infiltrates, gliosis
Microscopic (histologic) images

Contributed by Kymberly A. Gyure, M.D.
Missing Image

Methotrexate toxicity


Multiple sclerosis
Definition / general
  • Chronic, inflammatory demyelinating disease that may involve any part of the central nervous system
  • Lesions, known as plaques, are characterized by loss of myelin with relative axonal preservation, reactive gliosis and the presence of lymphocytes or macrophages
Essential features
  • Most common immune mediated demyelinating disorder of the central nervous system and most common cause of nontrauma related neurologic disability in young adults
  • Female predominant neurologic disorder consisting of episodic nervous system dysfunction and characteristic radiologic findings that demonstrate both dissemination in time (multiple episodes) and dissemination in space (multiple lesions seen on imaging)
Terminology
  • Multiple sclerosis
  • MS
ICD coding
  • ICD-10: G35 - multiple sclerosis
Epidemiology
  • Most frequent cause of nontrauma related permanent disability in young adults (Neurol Clin 2011;29:207)
  • Female predominance with F:M = 2.3:1
  • Mean age of onset: 28 - 31 years
  • Risk factors:
    • Genetics (HLA::DRB1)
    • Presence of coexisting autoimmune disease
    • Decreased sunlight exposure
    • Low vitamin D levels
    • Geographic location, with a higher latitude corresponding with a higher incidence of disease (Nature 2011;476:214)
  • More recently, it has been posited that there may be an association between EBV infection and the development of MS (Science 2022;375:296)
Sites
  • Can involve any site in the central nervous system (CNS), typically within the white matter but also involves the gray matter (Brain 2019;142:1858)
  • Most common sites for MS plaques:
    • Optic chiasm and tract (nearly always involved)
    • Periventricular white matter
    • Subcortical white matter
    • Deep gray nuclei
    • Periventricular areas of the brainstem, cerebellum and spinal cord
Pathophysiology
  • Since the cause of MS is unknown, a detailed pathophysiologic pathway has not yet been established (see Etiology)
Etiology
  • Cause of MS is unknown, though it is favored to be due to an autoimmune activation of lymphocytes (Arch Neurol 2004;61:1613)
Clinical features
  • Patients present with one or more distinct episodes of CNS dysfunction and characteristic MRI findings (dissemination in time and space) (Lancet Neurol 2018;17:162, Nat Rev Dis Primers 2018;4:43)
  • Relapsing remitting MS (RRMS)
    • Most common subtype of MS
    • Episodic neurologic deficits that may partially or fully resolve but are followed by additional relapses
  • Primary progressive MS (PPMS)
    • Nonepisodic progression of disease from the initial onset of symptoms
  • Secondary progressive MS (SPMS)
    • Typically follows RRMS, where the disease transitions from episodic to continued progression
  • Acute Marburg MS
    • Fulminant MS that is monophasic and rapidly progressive, usually leading to death within a year of symptom onset
    • Often seen in children and young adults
  • Tumefactive MS:
    • Uncommon subtype of acute MS that presents as a mass-like lesion with a radiologic differential diagnosis that includes demyelination, infection or neoplasm
    • Radiologically characterized by contrast enhancement often with unique open or incomplete ring
    • Surgical biopsy (with intraoperative frozen section consultation) is sometimes performed to help resolve the differential diagnosis
Diagnosis
Laboratory
  • Lumbar puncture is not required for the diagnosis of MS but can test for the presence of oligoclonal IgG bands in the cerebrospinal fluid (CSF), which is supportive of MS
    • E.g., if a patient is symptomatic with classic MRI findings showing dissemination in space but there is no known dissemination in time, oligoclonal bands can support a diagnosis of MS (Brain 2018;141:1075)
  • Majority of cells in the CSF differential count are lymphocytes (specifically T cells) and the protein levels should be within the reference range (Neurology 2004;63:1966)
Radiology description
Radiology images

Contributed by Jared T. Ahrendsen, M.D., Ph.D. and Pouya Jamshidi, M.D.
Dawson fingers

Dawson fingers

Inactive MS lesion Inactive MS lesion

Inactive MS lesion



Images hosted on other servers:

Plaques in spinal cord

Tumefactive MS

Prognostic factors
  • There are no consistently reliable prognostic factors for MS and outcomes for MS patients cannot be accurately predicted
  • Generally, patients with relapsing and remitting courses have a better prognosis than those with progressive forms of the disease (Brain 2006;129:584)
  • Rate of cervical cord atrophy, bowel / bladder symptoms at disease onset and incomplete recovery from initial attack are associated with a worse prognosis (Arch Neurol 2006;63:1686)
  • Pregnancy is thought to be protective, though disease can worsen in the initial postpartum period (Neurology 2017;89:563)
  • Extent of MRI abnormalities does not necessarily correlate with disease severity or prognosis (Neurology 2006;66:1384)
Case reports
Treatment
  • Acute exacerbation / episode (Mult Scler 2020;26:1352458520924595)
    • High dose glucocorticoids are the mainstay of treatment for an acute exacerbation
    • Plasma exchange can be considered in patients with poor response
  • Relapsing remitting MS (Mult Scler 2020;26:1352458520924595, Neurology 2018;90:777)
    • Monoclonal antibodies (natalizumab, alemtuzumab, ocrelizumab, rituximab) are generally first line therapy
    • Oral therapies including fumarates and S1P receptor modulators (siponimod, fingolimod) can be used in patients who do not wish to have or are unable to tolerate infusions / injectables
    • Interferons and glatiramer acetate
  • Secondary progressive MS
    • Same treatment as for relapsing remitting MS, though generally ineffective
  • Primary progressive MS
    • Ocrelizumab is the only FDA approved drug that has been shown to slow the progression of disease in PPMS (N Engl J Med 2017;376:209)
Gross description
  • Chronic plaques tend to be rounded, tan-gray and variably sized with a sharp demarcation from the surrounding brain tissue
  • More recent lesions will be tan-pink or tan-yellow with less sharply defined borders
  • In the spinal cord, plaques will have a fanned out appearance with sharp borders and can potentially involve multiple adjacent signaling pathways (Pract Neurol 2016;16:279)
Gross images

Contributed by Rachel A. Multz, M.D. and Jared T. Ahrendsen, M.D., Ph.D.
Subcortical and periventricular plaques

Subcortical and periventricular plaques

Subcortical plaques

Subcortical plaques

Periventricular plaques

Frozen section description
  • Typically only performed in cases of acute tumefactive MS, where there is radiologic suspicion for neoplasm or infection
  • Findings are similar for any type of demyelinating disease:
    • Sheets of foamy macrophages in a background of reactive gliosis
    • Can be misdiagnosed as high grade glioma, especially since reactive atypia and mitotic can activity seen (Creutzfeldt cells)
    • Cell populations will be more heterogeneous than neoplastic processes
    • On permanent sections, there will be a sharp border between the area of interest and background brain tissue
  • Reference: Acta Neuropathol 2017;133:13
Frozen section images

Contributed by Jared T. Ahrendsen, M.D., Ph.D.
Frozen section of demyelination lesion

Frozen section of demyelination lesion

Microscopic (histologic) description
  • Active MS plaque (Brain Pathol 2005;15:217):
    • Prominent perivascular lymphoid infiltrates consisting predominantly of CD8+ T lymphocytes
    • Parenchymal and perivascular macrophages, some with visible myelin globules on Luxol fast blue / periodic acid-Schiff stain (LFB / PAS)
      • More specific for active demyelination than lymphocytes
    • Preferential loss of myelin with relative axonal preservation and the formation of axonal spheroids (swellings)
    • Reactive astrocytosis
    • In the periphery of the plaque will see remyelination with thinner myelin sheaths than background axons
  • Chronic / inactive MS plaque (Brain Pathol 2005;15:217):
    • Relatively acellular lesions with near complete loss of myelin and sharp borders
    • Should not see inflammation (or if present, rare lymphocytes) and will only see oligodendrocytes at the margin
    • Reactive gliosis
  • Baló concentric sclerosis (Neurol Sci 2004;25:S319):
    • Rare histologic subtype of MS with concentric and alternating rings of demyelination and preserved myelin
    • Areas of demyelination will show features similar to those of an active MS plaque
  • Acute multiple sclerosis, Marburg type (Mult Scler 2015;21:485)
    • Multiple, poorly defined plaques
    • All plaques are active, with numerous macrophages, reactive astrogliosis and perivascular lymphocyte cuffs
    • Sometimes edema or necrosis may be seen
Microscopic (histologic) images

Contributed by Rachel A. Multz, M.D. and Jared T. Ahrendsen, M.D., Ph.D.
Chronic demyelination plaque Chronic demyelination plaque

Chronic demyelination plaque

Partial remyelination

Partial remyelination


Active MS lesion Active MS lesion

Active MS lesion

Active MS lesion Active MS lesion

Active MS lesion

Cytology images

Contributed by Jared T. Ahrendsen, M.D., Ph.D.
Smear preparation of demyelination lesion

Smear preparation of demyelination lesion

Positive stains
  • Luxol fast blue (LFB):
    • In active plaques, will highlight normal background myelin with complete loss of staining in the area of demyelination
      • Border between demyelination and normal appearing white matter will be distinct but not as sharp as that of an inactive plaque
    • In areas where remyelination has occurred, myelin will be present but less dense than the uninvolved white matter (shadow plaque)
    • Highlights myelin globules within macrophages in areas of active demyelination
  • CD68 or CD163 will highlight macrophages in areas of active demyelination
  • Neurofilament or a silver stain will highlight relative axonal preservation and the presence of occasional axonal spheroids
  • GFAP highlights reactive astrogliosis
  • Ki67 may be elevated in active plaques
  • CD3 will highlight perivascular T lymphocytes but is not necessary for diagnosis
  • Reference: Acta Neuropathol 2017;133:13
Negative stains
Electron microscopy description
  • No longer used in routine clinical practice
  • Active lesions will show lymphocytes entering the endothelium of blood vessels, axonal spheroids filled with mitochondria and neurofilaments, a vesicular network of degenerated myelin surrounding axons and myelin globules within macrophages
  • Chronic lesions will show demyelinated axons or remyelinated axons with thinner myelin sheaths (N Engl J Med 2006;354:942)
Sample pathology report
  • Brain lesion, left parietal lobe, biopsy:
    • Reactive brain tissue with numerous macrophages and evidence of demyelination (see comment)
    • Comment: The findings are suggestive of a demyelinating process. Given the radiologic impression of a mass-like lesion in the brain, tumefactive multiple sclerosis should be considered as a diagnostic consideration.
Differential diagnosis
  • Cerebral infarction:
    • Sheets of macrophages with engulfed myelin and evidence of demyelination (but primarily distributed along vascular territories)
    • Loss of axons (as opposed to demyelination, in which axons are relatively spared)
    • Other features of hypoxic ischemic injury are generally present
  • Neuromyelitis optica (NMO):
    • Formerly considered a variant of multiple sclerosis
    • Predominantly involves the optic tract and spinal cord, sometimes extensively involving the gray matter
    • Unique pathogenesis with antiaquaporin 4 antibodies
    • Neutrophils present in areas of demyelination, which are absent in MS
    • Prominent axonal loss and destruction of astrocytes
  • Acute disseminated encephalomyelitis (ADEM):
    • Marked T cell perivascular inflammation with local demyelination, however, will lack large plaque formation
    • Often with history of recent viral illness or vaccination
    • Self limited with rapid recovery and low probability of lasting neurologic sequelae
  • Progressive multifocal leukoencephalopathy (PML):
    • Clinical history of immunosuppression
    • Multiple foci of demyelination with sheets of macrophages and minimal lymphoid infiltrates
    • Unique plum colored viral inclusions in oligodendrocytes and bizarre appearing astrocytes
    • Demonstration of JC viral (PCR assay on cerebrospinal fluid; SV40 immunostain on tissue sections)
    • Note that MS patients treated with certain immunomodulatory agents (especially natalizumab) are at increased risk of developing PML (Front Neurol 2020;11:579438)
Board review style question #1

The pathologic features seen in this axial section of the spinal cord (H&E with Luxol fast blue [LFB]) taken at autopsy are most consistent with which of the following conditions?

  1. Amyotrophic lateral sclerosis
  2. Foix-Alajouanine syndrome
  3. Multiple sclerosis
  4. Subacute combined degeneration
  5. Tabes dorsalis
Board review style answer #1
C. Multiple sclerosis (MS). As shown in the image, there is asymmetric demyelination in random areas of the spinal cord in MS. Answer A is incorrect because amyotrophic lateral sclerosis shows symmetric degeneration of lateral corticospinal tracts. Answer B is incorrect because Foix-Alajouanine syndrome demonstrates myelopathy caused by dural arteriovenous malformation located in the spinal cord. Answer D is incorrect because subacute combined degeneration demonstrates relatively symmetric degeneration of both the lateral corticospinal tracts and the dorsal columns. Answer E is incorrect because tabes dorsalis shows symmetric degeneration of dorsal columns and dorsal nerve roots.

Comment Here

Reference: Multiple sclerosis
Board review style question #2

A 48 year old woman with multiple sclerosis treated with natalizumab develops numerous FLAIR signal abnormalities in the right centrum semiovale and left occipital lobe white matter. The patient deteriorated clinically and died 2 weeks after presentation. Brain autopsy was performed, revealing the microscopic features shown in the image above in areas of FLAIR signal abnormality. What is the most likely diagnosis?

  1. Active multiple sclerosis plaque
  2. Acute disseminated encephalomyeltis
  3. Infiltrating astrocytoma
  4. Progressive multifocal leukoenceophalopathy
  5. Toxoplasma encephalitis
Board review style answer #2
D. Progressive multifocal leukoencephalopathy (PML). The presence of macrophages, reactive gliosis, bizarre appearing astrocytes and plum colored oligodendroglial inclusions, along with the characteristic history described in the vignette, is most suggestive of PML. Active multiple sclerosis lesion is incorrect: active MS lesions are characterized by numerous macrophages and reactive gliosis; however, viral inclusions would not be seen in oligodendrocytes. Acute disseminated encephalomyelitis is incorrect: this is a monophasic demyelinating illness, distinct from multiple sclerosis and classically associated with recent viral infection or immunization; viral inclusions would not be seen in oligodendrocytes. Infiltrating astrocytoma is incorrect: while the atypical glial cells shown in the image might raise a concern for an infiltrating astrocytoma, the clinical history and imaging findings are not suggestive of a neoplastic process. Toxoplasma encephalitis is incorrect: Toxoplasma infection in the brain is characterized by necrotizing abscesses and the presence of parasites within pseudocysts (bradyzoites) or free parasites (tachyzoites).

Comment Here

Reference: Multiple sclerosis

Multiple system atrophy
Definition / general
  • Multiple system atrophy (MSA) is a sporadic, neurodegenerative disease with core clinical features of parkinsonism, autonomic dysfunction and cerebellar ataxia
  • Pathologically, MSA is characterized by glial cytoplasmic inclusions (GCIs) that are predominantly within oligodendroglia and composed of alpha-synuclein
Essential features
  • Clinically characterized by a triad of autonomic dysfunction, levodopa resistant parkinsonism and cerebellar ataxia
  • Pathologically defined by widespread frequent alpha-synuclein glial cytoplasmic inclusions (GCIs) and striatonigral or olivopontocerebellar degeneration
Terminology
  • Shy-Drager syndrome (autonomic dysfunction)
  • Olivopontocerebellar atrophy (OPCA)
  • Striatonigral degeneration
ICD coding
  • ICD-10:
    • G90.3 - multisystem degeneration of the autonomic nervous system
    • G23.2 - striatonigral degeneration
  • ICD-11: 8D87.0 - multiple system atrophy
Epidemiology
Sites
  • Central and peripheral nervous system
    • Cerebellum
    • Putamen
    • Pons
    • Substantia nigra
    • Spinal cord (intermediolateral column neurons)
  • Musculoskeletal
    • Atrophy of the posterior cricoarytenoid muscles
Pathophysiology
  • Unknown
  • Hypotheses of alpha-synuclein accumulation in oligodendroglia include SNCA gene reactivation in oligodendroglia and neuron oligodendroglia protein transfer (Acta Neuropathol Commun 2019;7:113)
Etiology
Clinical features
  • 2 clinical subtypes: parkinsonism predominant (MSA-P) and cerebellar ataxia predominant (MSA-C)
Diagnosis
  • There are 4 diagnostic categories of MSA: neuropathologically established, clinically established, clinically probable and possible prodromal (Mov Disord 2022;37:1131)
  • Neuropathologic diagnosis can only be made by postmortem examination
    • Widespread alpha-synuclein positive GCIs in association with neurodegenerative changes in nigrostriatal or olivopontocerebellar structures
  • Clinically established diagnosis requires a sporadic, progressive, adult onset disease with
    • Autonomic dysfunction
    • Either cerebellar ataxia syndrome or levodopa unresponsive parkinsonism
    • 2 supportive clinical features (features include rapid disease progression, postural instability, levodopa induced craniocervical dystonia, severe dysphagia, Babinski sign, postural deformities, stridor and pathologic laughter / crying)
    • 1 MRI marker (markers include atrophy of cerebellum, middle cerebellar peduncle, pons or putamen; hot cross bun sign; increased diffusivity of putamen or middle cerebellar peduncle)
  • Clinically probable diagnosis requires a sporadic, progressive, adult onset disease with
    • At least 2 of the following: parkinsonism, cerebellar ataxia or autonomic dysfunction
    • 1 supportive clinical feature (same as listed above)
    • Possible prodromal diagnosis requires a sporadic, progressive, adult onset disease with
      • At least 1 clinical nonmotor entry criteria (REM sleep behavior disorder, urogenital failure or neurogenic orthostatic hypotension)
      • At least 1 clinical motor entry criteria (subtle parkinsonian signs, subtle cerebellar signs)
Radiology description
  • Atrophy of putamen and signal decrease on iron sensitive sequences
  • Atrophy of cerebellum, pons and middle cerebellar peduncle
  • Hot cross bun sign (T2 hyperintense signal in axial MRI of pons; thought to be due to gliosis of pontocerebellar fibers)
  • Increased diffusivity of putamen and middle cerebral peduncle (J Mov Disord 2018;11:107)
Radiology images

Images hosted on other servers:

Bilateral putaminal atrophy

Hot cross bun sign

Hot cross bun sign and cerebellopontine atrophy

MRI findings

Prognostic factors
Case reports
  • 31 year old man carrying LRRK2 G2019S mutation and presenting with REM behavior disorder who was diagnosed with neuropathologically confirmed multiple system atrophy at autopsy (Mov Disord 2019;34:1080)
  • 66 year old woman with 4 year history of MSA and sudden death (Intern Med 2019;58:1643)
  • 70 year old man with parkinsonism and autonomic dysfunction (Acta Neuropathol 2019;137:167)
  • 6 patients (37 - 66 years old) presented with severe urinary retention, bowel and sexual dysfunction and abnormal anal sphincter electromyelography (EMG) (J Neurol 2020;267:659)
Treatment
  • No disease modifying or neuroprotective therapies are available
  • Treatment is aimed at symptomatic relief, with physical therapy and occupational therapy as mainstay
  • Autonomic dysfunction is treated similar to other neurogenic autonomic dysfunction
  • Parkinsonism symptoms are generally poorly responsive to levodopa but ~33% of patients will have transient clinical benefit (Mov Disord 2007;22:2141)
Clinical images

Images hosted on other servers:
Clinical signs of MSA-C

Clinical signs of MSA-C

Gross description
  • Atrophy of the cerebellum, middle cerebellar peduncles and pons
  • Pallor of the substantia nigra and locus coeruleus
  • Atrophy and gray-brown discoloration of the putamen (Brain Pathol 1999;9:721)
Gross images

Contributed by Emile Pinarbasi, M.D., Ph.D.

Cerebellar atrophy

Discoloration of the putamen

Depigmentation of substantia nigra

Pontine atrophy

Microscopic (histologic) description
  • Diagnostic criteria (Neuropathol Appl Neurobiol 2007;33:615)
    • Widespread abundant CNS alpha-synuclein positive GCIs
    • Neurodegenerative changes (atrophy, gliosis) in striatonigral or olivopontocerebellar structures
  • GCIs are in oligodendrocytes (in gray and white matter) and can be flame-like, triangular or sickle shaped
  • GCIs are composed of alpha-synuclein
  • GCIs can also be seen with Gallyas and other silver stains and are immunoreactive for ubiquitin and alpha B crystallin
  • Other alpha-synuclein inclusions, including intraneuronal inclusions (nuclear and cytoplasmic) and neuropil threads can also be seen, particularly in the pons and putamen but are not required for diagnosis
  • Tau positive granular glial inclusions can be seen, primarily in the cerebellum and putamen, particularly in cases of long disease duration (J Neurol Sci 2020;416:117010)
Microscopic (histologic) images

Contributed by Emile Pinarbasi, M.D., Ph.D.

Cerebellar atrophy with prominent white matter loss

Lipofuscin accumulation in putamen

Alpha-synuclein positive GCIs

Flame and sickle shaped GCIs


Alpha-synuclein GCIs and neuropil threads

Gliosis in pontine crossing fibers

GCIs in pontine crossing fibers

Filamentous intraneuronal inclusions

Positive stains
Negative stains
Electron microscopy description
  • GCIs are composed of a loose meshwork of randomly arranged, loosely packed filaments with cross sectional diameters of 15 - 30 nm
Electron microscopy images

Images hosted on other servers:

Glial cytoplasmic inclusions

Sample pathology report
  • Brain, 1,410 grams, autopsy:
    • Multiple system atrophy (see comment)
    • Comment: This case demonstrates classic gross and microscopic findings of multiple system atrophy (MSA) with pronounced atrophy of the pons and cerebellum, depigmentation of the substantia nigra and locus coeruleus and widespread alpha-synuclein glial cytoplasmic inclusions. MSA is a rare progressive neurodegenerative disease of unknown etiology and there are no effective disease modifying or neuroprotective therapies.
Differential diagnosis
  • Lewy body pathology (Parkinson disease and Lewy body dementia):
    • No alpha-synuclein GCIs
    • No cerebellar or pontine atrophy
    • Depigmentation of substantia nigra and locus coeruleus
    • Round to amorphous intraneuronal alpha-synuclein inclusions (Lewy bodies)
  • Inherited sporadic spinocerebellar ataxias:
    • No alpha-synuclein GCIs
    • No depigmentation of substantia nigra and locus coeruleus
    • Cerebellar atrophy
  • Corticobasal degeneration:
    • No alpha-synuclein GCIs
    • No cerebellar pontine atrophy
    • Prominent cortical and thalamic atrophy
    • Depigmentation of substantia nigra and locus coeruleus
    • Tau positive inclusions: astrocytic plaques and coiled bodies
  • Progressive supranuclear palsy:
    • No alpha-synuclein GCIs
    • No cerebellar atrophy
    • Prominent atrophy of frontal cortex, midbrain and pons
    • Tau positive inclusions: tufted astrocytes and coiled bodies
Board review style question #1


The images above are taken from the postmortem brain examination of a 72 year old man with 6 years of progressive ataxia, prominent autonomic dysfunction, bradykinesia and pill rolling tremor. Microscopic examination shows widespread oligodendroglial inclusions. These inclusions are positive for which of the following?

  1. Alpha-synuclein
  2. Amyloid beta
  3. Phospho-tau
  4. Phospho-TDP-43
Board review style answer #1
A. Alpha-synuclein

The gross image shows pronounced cerebellar and pontine atrophy and the microscopic image shows frequent, widespread oligodendroglial inclusions. These features are diagnostic for multiple system atrophy (MSA). This correlates with the clinical presentation of parkinsonism, ataxia and autonomic dysfunction. The oligodendroglial inclusions in this sporadic, fatal neurodegenerative disease are composed of alpha-synuclein.

Answer B is incorrect because amyloid beta forms extracellular plaques in Alzheimer disease neuropathologic change (ADNC) and deposits in vessel walls in cerebral amyloid angiopathy (CAA). Amyloid beta does not form oligodendroglial inclusions and B amyloid inclusions are not a feature of MSA. Answer C is incorrect because phospho-tau white matter inclusions can be seen in primary tauopathies, most notably corticobasal degeneration (CBD), characterized by teeming white matter disease; however, gross CBD will not have atrophy of cerebellum and pons. Additionally, the inclusions fill processes of astrocytes, giving a characteristic bushy / spiked rather than a flame shaped appearance.

Answer D is incorrect because phospho-TDP-43 inclusions are found in frontotemporal lobar disease and amyotrophic lateral sclerosis. Frontotemporal lobar disease presents with dementia and is characterized by pronounced frontal and temporal atrophy. Amyotrophic lateral sclerosis classically presents with upper and lower motor symptoms and grossly shows atrophy of ventral nerve roots. Neither of these would be expected to show gross atrophy of the cerebellum and pons. Additionally, while phospho-TDP-43 inclusions are infrequently seen in oligodendroglia, they are far more frequent in neurons and astrocytes.

Comment Here

Reference: Multiple system atrophy
Board review style question #2


A 62 year old woman with a 5 year history of parkinsonism, ataxia and orthostatic hypotension dies of respiratory failure. Postmortem brain examination reveals pronounced cerebellar atrophy and widespread alpha-synuclein positive oligodendroglial inclusions. Which of the following regarding this entity is true?

  1. Amyloid plaques are a characteristic microscopic finding
  2. Gross features include depigmentation of the substantia nigra
  3. Prominent cortical atrophy is a common feature
  4. The diagnostic alpha-synuclein inclusions shown above are also known as Lewy bodies
Board review style answer #2
B. Gross features include depigmentation of the substantia nigra

The combination of cerebellar atrophy and frequent widespread oligodendroglial alpha-synuclein inclusions is diagnostic of multiple system atrophy. This correlates with the clinical presentation of parkinsonism, ataxia and autonomic dysfunction. One of the common gross findings in multiple system atrophy is depigmentation of the substantia nigra.

The diagnostic alpha-synuclein inclusions in multiple system atrophy are known as glial cytoplasmic inclusions (GCIs). Answer D is incorrect because Lewy bodies are round alpha-synuclein positive intraneuronal inclusions seen in Lewy body pathology (Parkinson disease and Lewy body dementia). Answer C is incorrect because cortical atrophy is not a gross feature of multiple system atrophy. Answer A is incorrect because amyloid plaques are a histologic finding in Alzheimer disease and are not seen in multiple system atrophy.

Comment Here

Reference: Multiple system atrophy

Neural tube closure defects
Clinical images

Contributed by Mark R. Wick, M.D.

Sacral

Microscopic (histologic) images

Contributed by Mark R. Wick, M.D.

Skin

Skin, EMA

Various images


Other acute viral infections (pending)
Arboviral encephalitis
[Pending]
Poliomyelitis
[Pending]

Paraneoplastic syndromes
Definition / general
  • Nervous system dysfunction associated with cancer but without direct tumor invasion, infection or vascular complications of neural tissue
  • May precede clinical recognition of tumors
  • Occurs in < 1% of cancer patients; usually small cell carcinoma of lung, breast cancer or ovarian cancer
  • Often due to autoantibodies
  • May respond to treatment of the underlying tumors
Cerebellar degeneration
Definition / general:
  • Destruction of Purkinje cells (antibody mediated) with perivascular lymphocytes
  • Causes progressive bilateral leg and arm ataxia, dysarthria, variable vertigo and diplopia
  • Associated with breast, gynecologic, Hodgkin lymphoma and small cell carcinoma of lung
  • Often associated with anti-Yo antibody in serum or CSF
  • References: Brain 2003;126:1409
Lambert-Eaton myasthenic syndrome
Definition / general:
  • Destruction of neuromuscular junction (presynaptic terminals) by small cell lung cancer; due to IgG antibody
Limbic encephalitis
Definition / general:
  • Associated with subacute dementia - perivascular inflammatory cuffs, microglial nodules, neuronal loss, gliosis of temporal lobe and resembles infectious process
  • Also in brainstem; most common with small cell lung cancer
Myelopathies
Definition / general:
  • Necrotizing myelopathy due to leukemia, lymphoma and lung carcinoma
  • Myelitis due to anti-Hu antibody associated with small cell lung carcinoma
Opsoclonus / myoclonus (spontaneous chaotic eye movements)
Definition / general:
  • Affects brain stem but site unknown
  • Due to breast and small cell lung cancer; also neuroblastoma
  • May be associated with anti-Ri autoantibody
Peripheral neuropathy
Definition / general:
  • Most common paraneoplastic syndrome
  • May be associated with anti-Hu antibody in lung cancer patients
Primary lateral sclerosis
Definition / general:
  • Rarely caused by breast cancer
  • 50% develop lower motor neuron signs eventually
Retinal degeneration
Definition / general:
  • Destruction of photoreceptors
  • Due to small cell carcinoma and gynecologic tumors
Stiff man syndrome
Definition / general:
  • Destruction of spinal interneurons by amphiphysin antibody
  • Usually breast cancer, also lung cancer and thymoma
Subacute sensory neuropathy
Definition / general:
Subacute motor neuronopathy
Definition / general:

Case reports:

Differential diagnosis:

Parkinson's disease (pending)
Table of Contents
Definition / general
Definition / general
[Pending]

Pontine and extrapontine myelinolysis
Definition / general
  • Central pontine and extrapontine myelinolysis are complications of treatment of marked hyponatremia
Terminology
  • Also termed osmotic demyelination syndrome
Epidemiology
  • Occurs most commonly in setting of chronic alcoholism (~40% of cases)
  • Hyperosmolar state and other electrolyte imbalances are common
  • Other associated conditions are hepatic dysfunction / liver transplantation, malnutrition
Sites
  • Most commonly central pons
  • Extrapontine sites are cerebellum and lateral geniculate, regions of extensive gray white matter apposition
Pathophysiology / etiology
  • Associated with rapid correction of hyponatremia, due to disruption of blood brain barrier with edema and myelinolysis
Clinical features
  • Abrupt onset of encephalopathy following recovery from hyponatremia
  • Followed by dysarthria, dysphagia, quadriparesis, possible "locked in" syndrome, neurobehavioral deficits
Diagnosis
  • Typical imaging findings in an appropriate clinical scenario
Radiology description
  • "Trident" or "bat wing" shaped area of restricted diffusion in central pons
  • Symmetric T2 weighted signal abnormalities in cases with extrapontine involvement
Prognostic factors
  • Outcome (death, residual disability or complete recovery) is not predicted by clinical features or extent of radiologic abnormalities
Case reports
Treatment
  • Prevention is most important
  • Once it occurs, corticosteroids and supportive care
Gross description
  • Triangular, T shaped or diamond shaped area of discoloration which may be centrally cavitated in basis pontis
Gross images

Images hosted on other servers:
Missing Image

Small lesion

Microscopic (histologic) description
  • Myelin / oligodendrocyte loss with relative preservation of neurons / axons
  • Macrophages may be present but no significant lymphocytic inflammatory infiltrate
Microscopic (histologic) images

Images hosted on other servers:
Missing Image

Myelin stain

Differential diagnosis
  • Other causes of demyelination

Primary amebic meningoencephalitis
Definition / general
  • Primary amebic meningoencephalitis (PAM) is a rare, waterborne central nervous system (CNS) infection caused by the free living amoeba Naegleria fowleri
Essential features
  • Waterborne CNS infection caused by N. fowleri, with rapid onset and a high mortality rate
  • Characteristic organism has a single nucleus, vacuolated cytoplasm and is highlighted by Wright-Giemsa stain on cerebrospinal fluid (CSF) smears or on histology H&E stained slides
Terminology
  • Naegleriasis
ICD coding
  • ICD-10: B60.2 - naegleriasis
Epidemiology
Sites
  • Naegleria penetrates the olfactory mucosa and migrates into the central nervous system through the cribriform plate
  • In the brain, the organism multiplies and causes hemorrhagic necrosis of brain parenchyma and secondary involvement of the meninges (Microbiology (Reading) 2012;158:791)
Pathophysiology
  • N. fowleri occurs in 3 forms
    • Inactive cyst form: 7 - 12 μm spherical, incredibly resistant structure that can survive in low temperatures of winter
    • Transitory flagellate form: 10 - 16 μm pear shaped with 2 flagella, thrives at 27 - 37 °C; this is not the reproductively active or pathogenic form
    • Trophozoite form: ~22 μm long and 7 µm wide, long, slender, reproductively active form that can cause infection; trophozoites grow at 35 - 46 °C
  • Trophozoites are in the infective stage, which invades the central nervous system after nasal inoculation with amoebas, disrupting the olfactory mucosa
  • 2 primary mechanisms are noted for brain damage
Etiology
Diagrams / tables

Images hosted on other servers:
Infective mechanism of N. fowleri

Infective mechanism of N. fowleri

Clinical features
  • Disease usually develops within 1 - 7 days after exposure (J Assoc Physicians India 2015;63:69)
  • Clinical symptoms can be divided into early and late (J Pediatric Infect Dis Soc 2015;4:e68)
    • Early: flu-like symptoms only
      • Headache, nausea, vomiting, fever, fatigue and earache
    • Late: CNS involvement
      • Lethargy, nuchal rigidity, anorexia, irritation, confusion, photophobia, seizures, cranial nerve abnormalities, blurred vision and coma
Diagnosis
Laboratory
  • Can be diagnosed using microscopic methods, CSF culture, amoebic antigen detection and direct detection of parasite DNA by RT PCR
  • CBC shows leukocytosis, predominantly neutrophils
  • Increased opening pressure for CSF with increased red blood cell count
  • Leukocytosis in CSF (neutrophil predominant); increased proteins and decreased glucose in CSF
  • CSF reveals many neutrophils along with pseudopod forming, actively motile N. fowleri amoeba trophozoites, highlighted by Wright-Giemsa or trichrome special stains
  • N. fowleri can be cultivated in nonnutrient / low nutrient agar plate or in a Nelson growth medium
  • RT PCR can help determine exact genus and species (ITS1 oligonucleotide sequence of Naegleria spp.) (J Water Health 2023;21:972)
  • References: Biomolecules 2021;11:1320, J Water Health 2023;21:972, J Pediatric Infect Dis Soc 2015;4:e68, Am J Trop Med Hyg 2017;97:1600
Radiology description
Radiology images

Images hosted on other servers:
CT scan findings

CT scan

MRI findings

MRI

Case reports
Treatment
  • As the disease is rare and rapidly fatal, there are no clinical trials to determine optimal treatment regimens
  • Generally, amphotericin B is recommended both intravenously and intrathecally
  • Considering high disease mortality, a combination of drugs is usually used
  • Amphotericin in combination with fluconazole, azithromycin, miltefosine and posaconazole has been noted to be effective
  • Recently it has been demonstrated that amphotericin B and nystatin conjugated with silver nanoparticles are very effective in disease models
  • Administration of dexamethasone reduces intracranial pressure, cerebral edema and brain damage caused by hyperinflammation
  • References: Exp Parasitol 2018;187:1, StatPearls: Amebic Meningoencephalitis [Accessed 14 September 2023], Biomolecules 2021;11:1320, ACS Chem Neurosci 2016;7:1178
Gross description
  • Done on autopsy
  • Brain shows extensive areas of hemorrhage and necrosis
  • Cortex of brain is edematous
  • Leptomeningeal purulent exudate may be noted
  • Reference: AJNR Am J Neuroradiol 2006;27:1217
Frozen section description
  • Frozen section not performed as PAM usually presents clinically with features of an infection
Microscopic (histologic) description
  • Brain tissue shows massive, acute on chronic inflammatory cell infiltrate comprised of neutrophils, macrophages and a few eosinophils
  • Necrotizing vasculitis, thrombosis, fibrinoid necrosis and a large number of round, large organisms, 15 - 25 μm, with vacuolated cytoplasm and single nuclei
  • Leptomeningeal exudate is noted
  • Direct wet mount preparation of CSF also reveals characteristic highly motile amoebic organisms and cysts
  • Organism trophozoite form has a single nucleus with prominent nucleoli and exhibits vacuolated cytoplasm
  • References: AJNR Am J Neuroradiol 2006;27:1217, Emerg Infect Dis 2004;10:1835, BMC Infect Dis 2017;17:532
Microscopic (histologic) images

Contributed by Mohammad Khurram Minhas, M.B.B.S.
Prominent nucleoli

Prominent nucleoli

Acute necrotizing inflammation

Acute necrotizing inflammation

Necrotizing vasculitis

Necrotizing vasculitis

Obvious cytoplasmic vacuolations

Obvious cytoplasmic vacuolations

Cytology description
  • In CSF examination, N. fowleri trophozoites are large, round to pear shaped cells with a prominent, dark nucleus and scattered vacuoles (ACS Chem Neurosci 2016;7:1178)
Cytology images

Images hosted on other servers:
Trophozoites on CSF cytological examination

Trophozoites on CSF cytological examination

Immunofluorescence description
Negative stains
  • Gram stain (should be avoided on laboratory testing as it can kill the amoeba)
Electron microscopy description
  • Characteristic amoeba nuclei have a large nucleolus and distinctive mitochondria with coarse, granular cristae (J Med Microbiol 1979;12:363)
Electron microscopy description

Images hosted on other servers:
Trophozoite under electron microscope

Trophozoite

Trophozoite with phagocytic activity

Trophozoite with phagocytic activity

Videos

Lab detection of N. fowleri

Sample pathology report
  • Brain space occupying lesion, biopsy:
    • Primary amebic meningoencephalitis (naegleriasis) (see comment)
    • Comment: Histological examination of the sections reveal brain parenchyma predominantly exhibiting reactive gliosis along with dense inflammatory cell infiltrate comprised of neutrophils, lymphocytes and histiocytes. The intervening vascular channels show evidence of necrotizing vasculitis; a few scattered organisms are noted with centrally placed nuclei, a single prominent nucleolus and vacuolated cytoplasm. These organisms are highlighted on Wright-Giemsa stain and are positive for Nfa1 antibody on immunohistochemistry. The findings are suggestive of primary amebic meningoencephalitis (naegleriasis).
Differential diagnosis
  • Other amebic infections:
    • Balamuthia and Acanthamoeba brain infections can mimic PAM caused by N. fowleri; however, these infections are chronic and subacute in nature
    • Both of these infections elicit a granulomatous response, which is absent in PAM (Mod Pathol 2007;20:1230)
  • Brain abscess:
    • Contains inflammatory cells along with macrophages
    • Lacks N. fowleri trophozoites and Nfa1 is negative by immunohistochemistry
  • Other bacterial, viral and protozoal causes of meningoencephalitis:
Board review style question #1

A 16 year old boy presented with acute onset of severe headache, emesis and fever after he went to a swimming camp at a lake. He was admitted into the emergency room. Cerebrospinal fluid (CSF) chemical analysis revealed leukocytosis, increased protein and decreased glucose. Microscopic examination of the CSF revealed predominance of neutrophils and scattered organisms with single nuclei, vacuolated cytoplasm and prominent pseudopodia. These organisms were highlighted on Wright-Giemsa stain. What is the most likely diagnosis?

  1. Acanthamoeba encephalitis
  2. Cytomegalovirus (CMV) encephalitis
  3. Herpes encephalitis
  4. Primary amebic meningoencephalitis
Board review style answer #1
D. Primary amebic meningoencephalitis (PAM). PAM is a waterborne disease caused by Naegleria fowleri and has rapid onset. The characteristic organism has a single nucleus, vacuolated cytoplasm and is highlighted on the Wright-Giemsa special stain. Answer A is incorrect because Acanthamoeba encephalitis has subacute onset of symptoms and is not associated with infected water. Answers B and C are incorrect because both CMV and herpes encephalitis cause acute viral encephalitis. The CSF findings are not suggestive of viral encephalitis and the organism morphology is also not typical of CMV or herpes infected cells.

Comment Here

Reference: Primary amebic meningoencephalitis

Primary angiitis of CNS (pending)
Table of Contents
Definition / general
Definition / general
[Pending]

Progressive multifocal leukoencephalopathy (PML) and other JC virus infections
Definition / general
  • Until recently, central nervous system infections with JC virus presented as a progressive, multifocal, demyelinating / degenerative white matter process within patients with chronic lymphoproliferative disorders
  • Even today, progressive multifocal leukoencephalopathy and JC virus infection are often thought as being one and the same process
  • However, in recent years, major changes in the understanding of these infections include:
    1. White matter disease may be nonprogressive, unifocal and inflammatory
    2. Neuronal infection is seen commonly in patients with white matter disease and may also be seen in isolation
    3. Reversal of the underlying predisposing immunosuppressed state (which is still the only effective therapy) may be accompanied by paradoxical worsening of the central nervous system disease, referred to as PML-IRIS (immune reconstitution inflammatory syndrome)
Terminology
  • Progressive multifocal leukoencephalopathy (PML), JC virus granule cell neuronopathy (JCV GCN), JC virus encephalopathy (JCVE), progressive multifocal leukoencephalopathy - immune reconstitution inflammatory syndrome (PML-IRIS), JC virus granule cell neuronopathy - immune reconstitution inflammatory syndrome (GCN-IRIS)
Epidemiology
  • Initially described as a rare complication of B cell lymphoproliferative disorders, JC virus infections affect ~5% of AIDS patients (although infections in patients treated with combined antiretroviral therapy are milder than those seen prior to implementation of these regimens)
  • Patients with autoimmune diseases receiving monoclonal antibody therapies, especially patients with multiple sclerosis being treated with natalizumab (~400 patients reported to date, MAbs 2009;1:583)
Sites
  • Progressive multifocal leukoencephalopathy may affect the white matter anywhere within the central nervous system and has a distinct predilection for the cortical / subcortical interface
  • JC virus granule cell neuronopathy selectively infects the granule cells of the cerebellum, while JC virus encephalopathy selectively infects cerebrocortical neurons
Pathophysiology
  • Once the JC virus is reactivated (see below), it gains access to the central nervous system, where it lytically infects oligodendroglial cells, nonlytically infects astrocytes and may infect neurons either in combination with glial cells or by themselves (Discov Med 2011;12:495, PLoS Pathog 2012;8:e1003014)
Etiology
  • Although primary infections with JC virus may lead to development of central nervous system disease, most cases are felt to be secondary to reactivation of dormant virus residing within the kidney, bone marrow or brain
  • ~50% of population is serologically positive for JC virus, which is usually acquired asymptomatically in childhood
  • Although often thought of as a disease of profoundly immunosuppressed individuals, JC virus central nervous system infection may occur in patients with minimal or even occult immunosuppression
  • Some investigators believe that the fundamental reactivating process is B cell dysregulation, which is in turn usually but not always a result of loss of T cell surveillance
Clinical features
  • Due to the wide range of nervous system structures which may be involved, the clinical manifestations may be protean (resembling Proteus in having a varied nature)
  • However, PML is characterized by behavioral and cognitive abnormalities along with gait abnormalities, motor weakness, visual field deficits, speech and language deficits and incoordination
  • JC virus granule cell neuronopathy presents with cerebellar dysfunction, including ataxia and dysmetria
  • JC virus encephalopathy presents with global cerebrocortical dysfunction accompanied by seizures (Cleve Clin J Med 2011;78:S8)
Diagnosis
  • Diagnosis of progressive multifocal leukoencephalopathy is made using a combination of clinical features, imaging findings and either CSF PCR for JC virus or demonstration of viral infection on brain biopsy
  • Newer ultrasensitive techniques for CSF PCR have demonstrated greater than 95% sensitivity although this may be lower in patients with more modest immunosuppression
    • In such cases, a brain biopsy may be needed to establish a definitive diagnosis although many cases may be treated as probable PML using only clinical and imaging features (Neurology 2013;80:1430)
Laboratory
  • Most HIV+ patients with PML have CD4 lymphocyte counts less than 200 cells per cubic millimeter but ~10% have CD4 counts greater than 200/mm3
  • CSF all counts are usually less than 20 per cubic millimeter
  • CSF PCR for JC virus is the most useful laboratory diagnostic test but is not 100% sensitive (see above, J Clin Virol 2008;43:247, J Med Microbiol 2009;58:253)
Radiology description
  • CT scans show hypointense lesions within the affected white matter of patients with PML
  • MRI demonstrates hyperintense lesions on T2 and FLAIR weighted images
  • Faint, typically peripheral located contrast enhancement may be seen in ~5% of cases although this increases to 15% in patients with HIV associated PML and 40%% in patients with natalizumab associated PML
    • Latter is often monofocal, with frontal lobe lesions predominating
  • PML-IRIS, regardless of the initial predisposing condition, is characterized by prominent enhancement secondary to the reconstituted inflammatory response (Neurology 2012;79:1041)
Radiology images

Contributed by Mark Cohen, M.D.

White matter hyperintensity



Images hosted on other servers:

Fluid attenuated inversion recovery image

FLAIR sequence

Prognosis and treatment
  • No specific effective therapy for JC virus - survival depends entirely on the ability to reverse the immunosuppressed state
  • Overall, 1/3 of patients die, 1/3 have severe neurological deficits, 1/3 recover with minimal to moderate neurological deficits (Neurology 2009;73:1551, J Virol 2011;85:7256)
  • Paradoxical worsening of symptoms (referred to as the immune reconstitution inflammatory syndrome) appears to be controllable (paradoxically) with corticosteroid therapy (high dose, short term)
Case reports
  • 49 year old woman with multiple sclerosis (MS) and lesion suspicious of progressive multifocal leukoencephalopathy (PML) (Neurology 2011;77:2010)
  • 61 year old man with treated follicular lymphoma and cerebellar mass (Case of the Week #265)
  • 74 year old man with psoriasis who developed PML after fumaric acid treatment (N Engl J Med 2013;368:1657)

    PML-IRIS:
  • 23 year old man with simultaneous PML-IRIS after discontinuation of natalizumab (Neurology 2012;78:1390)
  • 38 year old woman with PML-IRIS and treated with brentuximab (Neurology 2012;79:2075)
  • 49 year old woman with maraviroc and JC virus associated immune reconstitution inflammatory syndrome (N Engl J Med 2014;370:486)

    PML, possibly in combination with JC virus granule cell neuronopathy:
  • 83 year old woman with progressive hemiataxia, tremor and infratentorial lesions (Neurology 2011;77:e7)
Gross images

Contributed by Mark Cohen, M.D.

Degenerative change within the subcortical white matter

Microscopic (histologic) images

Contributed by Mark Cohen, M.D.

Extention into the overlying cerebral cortex

Perivascular inflammation

Cowdry B-like viral inclusions

Viral DNA

SV40



Images hosted on other servers:

Luxol fast blue staining

H&E, CD68, nuclear p53

Positive stains
  • In situ hybridization demonstrating JC viral DNA indicates viral infection
  • All commercially available antibodies raised against SV40 antigen cross react with JC virus and confirm the active stage of the virus cycle
  • Some antibodies are not sensitive enough to be used on paraffin embedded formalin fixed sections so care must be used in the evaluation of these preparations
Electron microscopy images

Images hosted on other servers:

Frontal brain biopsy

Differential diagnosis
  • On biopsy, principal differential diagnostic considerations include noninfectious demyelination and theoretically glioma (as a result of misinterpretation of enlarged atypical viral nuclei)
  • Attention to the preserved nuclear to cytoplasmic ratio should prevent the latter, while careful scrutiny for viral inclusions should prevent the former

Progressive supranuclear palsy (pending)
Table of Contents
Definition / general
Definition / general
[Pending]

Rabies
Definition / general
  • Zoonotic disease caused by rabies virus, which is usually transmitted to humans by animal bites
Essential features
  • Rabies is an almost uniformly fatal neurologic illness caused by rabies virus which is typically transmitted by a bite from a rabid animal
  • Two main clinical forms, encephalitic and paralytic
  • Characteristic pathologic features include Negri bodies, which are intracytoplasmic, eosinophilic inclusions found in neurons of the hippocampus and cerebellum
Epidemiology
  • ~60,000 human rabies deaths per year worldwide
  • More common in developing countries where rabies is endemic in dogs
  • Rabies in bats is an important risk factor in North America
Sites
  • Hippocampi and cerebellum
Pathophysiology
  • Rabies virus is secreted in the saliva of infected animals and is inoculated into tissue at the bite site
  • Virus then spreads via the peripheral nerves to the brainstem or spinal cord and disseminates throughout the central nervous system along neuroanatomical connections and selectively injures neuronal processes (axons and dendrites) without producing neuronal death
Etiology
  • Caused by rabies virus, a negative strand RNA virus in the family Rhabdoviridae
Clinical features
  • Nonspecific, flu-like prodrome
  • Paresthesia, pain or itching at the site of the bite
  • Encephalitic ("furious") form (80% of cases): brain involvement predominates; acute neurologic illness characterized by hyperactivity, disorientation, bizarre behavior, hallucinations and autonomic dysfunction
  • Hydrophobia (pharyngeal / laryngeal spasm when attempting to drink) is characteristic but present in only 50% - 80% of patients
  • Paralytic form: spinal cord and peripheral nerve / nerve root involvement predominates; less common variant with weakness in the bitten limb followed by progression to quadriparesis with facial weakness
Laboratory
  • CSF pleocytosis in 60% of cases
  • Diagnosis can be confirmed by detecting anti rabies virus antibodies in serum or cerebrospinal fluid and rabies virus antigen or RNA in tissue (nuchal skin biopsy) or fluids
Radiology description
  • Normal or increased signal of gray matter structures on T2 weighted images
Prognostic factors
  • Almost always fatal in nonimmunized patients
Case reports
Treatment
  • Post exposure prophylaxis: wound cleansing and administration of rabies vaccine and rabies immune globulin
Gross description
  • Brain and spinal cord are grossly normal in most cases
Microscopic (histologic) description
  • Negri and lyssa bodies: round to oval, eosinophilic intracytoplasmic inclusions in the pyramidal cells of the hippocampus and cerebellar Purkinje cells
  • Perivascular chronic inflammation, microglial nodules and neuronophagia
Microscopic (histologic) images

Contributed by Kymberly A. Gyure, M.D.

Rabies

Differential diagnosis
  • Bat acquired cases are more likely to be misdiagnosed due to the lack a bite history
  • Guillain-Barré syndrome

Radiation injury
Definition / general
  • Radiation therapy is commonly used as adjuvant therapy for primary and secondary malignant CNS neoplasms
  • Includes autonomic, cognitive and functional deficits
  • Spectrum of toxic CNS effects is classified based on time of appearance of symptoms
  • Histological features range from no obvious lesions to overt brain necrosis
Essential features
  • Radiation injury mainly affects white matter
  • Typically classified based on time of appearance of symptoms
  • Late focal radiation injury: differential diagnosis includes recurrent / residual tumor
  • Acute, early delayed and late (diffuse) radiation injury: clinical and radiological diagnosis
Terminology
  • Radiation induced brain injury
    • CNS lesions secondary to radiation injury are typically classified based on time of appearance of symptoms (see Diagrams / tables) (Radiat Res 2000;153:357)
      • Acute radiation induced brain injury: days to weeks
      • Early delayed radiation induced brain injury: weeks to 3 - 6 months
      • Late radiation induced brain injury (focal and diffuse): > 3 - 6 months
  • Treatment induced necrosis of brain (in neurooncology setting): also known as late radiation induced brain injury, focal type; includes chemoradiation or only radiation effects
  • Necrotizing leukoencephalopathy: diffuse white matter injury following chemotherapy with or without radiation (AJNR Am J Neuroradiol 1991;12:45)
  • Radiation induced brain cavernomas (Acta Biomed 2019;90:77)
  • Radiation induced brain aneurysms (World Neurosurg 2020;135:339)
  • Additional terminologies
    • Pseudoprogression - transient, weeks to < 6 months
    • Progressive disease - recurrent / residual disease status post therapy
    • Pseudoresponse - frequently observed in high grade glioma patients treated with angiogenesis inhibitors (e.g.. bevacizumab) (Semin Neurol 2017;37:589)
ICD coding
  • ICD-10: G93.89 - other specified disorders of brain
    • Not well characterized
    • Applicable to postradiation encephalopathy
Epidemiology
Sites
Pathophysiology
  • Acute radiation induced brain injury: disruption of blood brain barrier, edema and increased intracranial pressure (focal)
  • Early delayed radiation induced brain injury: transient demyelination
  • Late radiation induced brain injury (focal and diffuse): microvascular injury resulting in tissue hypoxia, demyelination and gliosis, white matter necrosis
  • Experimental hypothesis
    • Vascular hypothesis (late radiation injury)
      • Radiation induced vascular changes: vessel wall thickening, vessel dilation and endothelial cell nuclear enlargement (dose dependent); capillary rarefaction
    • Parenchymal / glial hypothesis
    • Dynamic interaction between the 2 hypotheses
  • Pseudoprogression
  • Reference: Front Oncol 2012;2:73
Etiology
Diagrams / tables

Images hosted on other servers:

Timeline of cancer treatment related effects

Clinical features
  • Latency: 0 - 32 years (mean: 6 years)
  • Clinical features are heterogeneous and difficult to predict
  • Acute radiation induced brain injury: headache, fatigue and drowsiness
    • Transient worsening of symptoms
  • Early delayed radiation induced brain injury: generalized weakness, somnolence, attention deficits and short term memory loss
    • Usually transient
  • Late radiation induced brain injury: reflects damage to both hippocampal and nonhippocampal dependent domains (irreversible / progressive / fatal)
    • Focal radiation injury: focal neurologic deficits and focal increase in intracranial pressure
      • Indistinguishable from symptoms due to a mass of any kind
      • Principal clinical differential diagnosis is recurrent / residual tumor (progressive disease)
    • Diffuse radiation injury (cognitive impairment): decreased verbal memory, spatial memory, attention, problem solving ability and executional functions
      • Can also have depression, anxiety and somatization
  • Advanced stages: dementia, stupor state, hallucinations and delusion
  • Pseudoprogression
  • References: IntechOpen: Radiation Induced Brain Injury After Radiotherapy for Brain Tumor [Accessed 7 April 2023], Front Oncol 2012;2:73, Neuro Oncol 2019;21:1118
Diagnosis
  • Acute, early delayed and late (diffuse) radiation injury: clinical and radiological diagnosis
  • Late (focal) radiation injury (diagnostic ambiguity)
    • Clinically and radiographically overlap with recurrent / residual disease
    • Histopathologic diagnosis depends on pathologist's experience and subjective impression
    • Radiation necrosis versus recurrent / residual tumor
      • Requires multidisciplinary approach (neuropathologist, neuroradiologist, radiation oncologist, neurosurgeon and neurooncologist) (J Neurooncol 2013;113:485)
    • Reference: Neuro Oncol 2019;21:1118
Radiology description
  • Late (focal) radiation injury
    • CT: focal hypodensity, contrast enhancement, mass effect
    • MRI: focal decreased signal on T1WI; focal hyperintensity on T2WI, contrast enhancement, mass effect
  • Late (diffuse) radiation injury
    • CT: diffuse white matter hypodensity, contrast enhancement, mass effect
    • MRI: diffuse decreased white matter signal on T1WI; diffuse white matter hyperintensity on T2WI
  • Pseudoprogression
    • Represents a unique, transient, predominantly radiographic phenomenon encountered in patients with high grade glioma status post chemoradiation
    • MRI - increase in abnormal nodular enhancement on T1 post contrast; increase in T2/FLAIR signal hyperintensities (suggestive of cerebral edema) (Semin Neurol 2017;37:589)
  • Reference: AJNR Am J Neuroradiol 1991;12:45
Radiology images

Images hosted on other servers:

Focal radiation necrosis

Differential diagnoses

Hemorrhagic radiation injury

Diffuse white matter injury

Diffuse white matter change (severe)

Concurrent focal and diffuse white matter injury


Diffuse necrotizing leukoencephalopathy

Missing Image Missing Image

Radiation necrosis of pons

Missing Image

Abnormal enhancement in left occipital lobe

Missing Image

Irregular enhancement around surgical cavity

Missing Image

Pseudoprogression

Prognostic factors
  • Radiation necrosis and pseudoprogression without active tumor is associated with better prognosis (J Neurooncol 2013;113:485)
  • Active malignant tumor portends poor prognosis (percentage treatment response has limited prognostic value) (J Neurooncol 2019;141:421)
  • Increased Ki67 proliferation index is associated with worse prognosis
  • See Etiology
Case reports
  • 23 year old woman with remote history of low grade brainstem glioma, with delayed radiation induced stroke (Pediatr Neurol 2019;98:87)
  • 37 year old man with radiation retinopathy after external beam irradiation for nasopharyngeal carcinoma (Pract Radiat Oncol 2018;8:366)
  • 52 year old man with cystic radiation encephalopathy after radiotherapy treatment for nasopharyngeal carcinoma (BMC Neurol 2021;21:59)
  • 59 year old man's autopsy revealing late delayed radiation injury following radiotherapy treatment at age 14 (Neuropathology 2019;39:106)
Treatment
  • Symptomatic treatment
  • Oral corticosteroids for symptomatic cases
  • Resection with frozen section diagnosis
  • No known medical therapy for cognitive impairment (Front Oncol 2012;2:73)
  • Hippocampal avoidance strategies
    • Stereotactic conformal radiotherapy
    • Intensity modulated radiotherapy
    • Proton beam therapy
  • Potential therapies under investigation (IntechOpen: Radiation Induced Brain Injury After Radiotherapy for Brain Tumor [Accessed 7 April 2023], Neurooncol Adv 2020;2:vdaa057)
    • Antiplatelet and anticoagulation
    • Reactive oxygen species (ROS) scavengers
    • Improving microcirculation: butylphthalide
    • Neurogenesis: neural stem cell therapy
    • Renin angiotensin system inhibitors
    • Anti-VEGF antibody
    • Hyperbaric oxygen treatment
    • Exercise
    • Small molecule compounds targeting p53 isoform Δ133p53α
    • Peroxisomal proliferator activated receptor agonists
    • Memantine
    • Lithium
Gross description
  • Small (usually < 0.5 cm) brain / lesional tissue piece(s), white-tan, with or without hemorrhage / necrosis
Frozen section description
  • Gross: see Gross description
  • Squash / smears: see Cytology description
  • Microscopy
    • Challenging to differentiate between postradiation effects (radiation induced atypia) and brain tumor (residual / recurrent)
    • Experience improves accuracy
    • Seek second opinion when unsure
    • See Microscopic (histologic) description - focal late radiation injury; and see Differential diagnosis - focal late radiation induced brain injury (treatment induced necrosis) versus recurrent / residual tumor
Frozen section images

Contributed by Palgun Nisarga, M.D.
Negative for tumor (frozen) Negative for tumor (frozen)

Negative for tumor

Recurrent / residual tumor (frozen)

Recurrent / residual tumor

Microscopic (histologic) description
  • Focal late radiation injury
    • Hyalinization and fibrinoid necrosis of small arteries and arterioles (characteristic finding) and narrow lumens with endothelial proliferation
    • White matter changes range from minimal changes to rarefaction of myelin and reactive gliosis to coagulative necrosis
    • With or without perivascular mononuclear infiltrate
    • Radiation induced cytologic atypia
  • Diffuse late radiation injury
    • Histopathologic diagnosis not indicated
    • White matter changes similar to focal late radiation injury
  • Pseudoprogression
    • No established histopathological classification criteria for treatment necrosis or pseudoprogression (Neuro Oncol 2019;21:1118)
  • Progressive disease
    • Tumor cellularity depends on the treatment (radiation) response
  • In practice, the most common histologic finding is radiation necrosis with active tumor (J Neurooncol 2019;141:347)
  • References: AJNR Am J Neuroradiol 1991;12:45, IntechOpen: Radiation Induced Brain Injury After Radiotherapy for Brain Tumor [Accessed 7 April 2023]
Microscopic (histologic) images

Contributed by Palgun Nisarga, M.D.
Negative for tumor (FFPE) Negative for tumor (FFPE)

Negative for tumor

Negative for tumor (FFPE)

Negative for tumor

Recurrent / residual tumor (FFPE)

Recurrent / residual tumor

Cytology description
  • Reactive astrocytes with or without mononuclear cells in a necrotic background
  • Residual / recurrent tumor cells may be present
Cytology images

Contributed by Palgun Nisarga, M.D.
Squash preparation (frozen) Squash preparation (frozen)

Squash preparation (frozen)

Squash preparation (frozen) Squash preparation (frozen)

Squash preparation (frozen)

Sample pathology report
  • Brain, right temporal, resection:
    • Changes consistent with radiation / chemotherapy effect
    • No evidence of tumor

    • Brain, left frontal, resection:
      • Recurrent / residual glioblastoma with changes consistent with radiation / chemotherapy effect
Differential diagnosis
Board review style question #1
Intraoperative histopathologic evaluation is recommended in which of the following types of brain radiation injury?

  1. Acute radiation induced brain injury
  2. Early delayed radiation induced brain injury
  3. Early delayed and late focal radiation induced brain injury
  4. Late diffuse radiation induced brain injury
  5. Late focal radiation induced brain injury
Board review style answer #1
E. Late focal radiation induced brain injury. Late focal radiation injury can be confused with residual / recurrent tumor based on the imaging findings. Answers A - D are incorrect because they have imaging findings different from that of a tumor.

Comment Here

Reference: Radiation injury
Board review style question #2

What is the characteristic histopathologic finding in focal (late) radiation induced brain injury?

  1. Coagulative necrosis with reactive gliosis
  2. Hyalinization and fibrinoid necrosis of small sized arteries
  3. Narrow vascular lumen with endothelial proliferation
  4. Perivascular mononuclear infiltrate
  5. Rarefaction of myelin
Board review style answer #2
B. Hyalinization and fibrinoid necrosis of small sized arteries. Answers A, C, D and E are incorrect because these findings are nonspecific for radiation injury and can be seen in other conditions.

Comment Here

Reference: Radiation injury

Subacute combined degeneration
Definition / general
  • Acquired myelopathy caused by vitamin B12 (cobalamin) deficiency
Sites
  • Posterior and lateral columns of spinal cord
Pathophysiology
  • Adenosylcobalamin and methylcobalamin are metabolites of cobalamin; they are coenzymes in reactions which (a) convert methylmalonyl CoA to succinyl CoA and (b) transfer methyl groups to homocysteine to form methionine, respectively
  • Accumulation of methylmalonyl CoA causes a decrease in normal myelin synthesis and incorporation of abnormal fatty acids into neuronal lipids
Etiology
  • Cobalamin deficiency: vegetarian diet
  • Impaired absorption of cobalamin intrinsic factor (IF) complex: pernicious anemia / atrophic gastritis, partial gastrectomy, fish tapeworm infection, Imerslund-Gräsbeck syndrome (genetic defect of cobalamin IF complex receptor), Crohn disease
  • Subacute combined degeneration can be triggered by exposure to nitrous oxide in B12 deficient patients
Clinical features
  • Progressive sensory abnormalities, ascending paresthesias, weakness, ataxia, loss of sphincter control and gait impairment
Diagnosis
  • Schilling test: radiolabeled cyanocobalamin is administered orally and its excretion is measured in the urine
  • Increased serum methylmalonic acid and homocysteine concentrations
  • Decreased serum cobalamin concentration
  • Macrocytic / megaloblastic anemia
Radiology description
  • Symmetric, increased T2 signal confined to posterior and lateral columns of the cervical and thoracic spinal cord
Radiology images

Images hosted on other servers:
Missing Image

Cervical spinal cord MRI

Missing Image

Brain MRI

Prognostic factors
  • Less severe disease at time of diagnosis, including absence of sensory dermatomal deficits, Romberg sign and Babinski sign, is associated with a higher rate of resolution following cobalamin replacement therapy
Case reports
Treatment
  • Intramuscular cobalamin injections
Gross description
  • Shrunken spinal cord with discoloration of the posterior and lateral columns
Microscopic (histologic) description
  • Bilaterally symmetric myelin loss and degeneration in the posterior and lateral columns of the spinal cord
  • Dense gliosis in longstanding cases
Microscopic (histologic) images

Contributed by Kymberly A. Gyure, M.D.

Subacute combined degeneration

Peripheral smear images

Images hosted on other servers:
Missing Image

Bone marrow aspiration

Differential diagnosis
  • Tabes dorsalis: does not affect lateral columns of spinal cord
  • Vacuolar myelopathy of acquired immunodeficiency syndrome: thought to be related to abnormal vitamin B12 metabolism but serum B12 levels are typically normal

Subacute sclerosing panencephalitis (SSPE) (pending)

Syphilis (pending)

Taenia solium (neurocysticercosis)

Toxoplasmosis

Trypanosomes (pending)

Tuberculosis
Definition / general
  • An infectious / inflammatory process within the central nervous system (CNS) that can involve the meninges, brain parenchyma or both, and is typically composed of areas of cheese-like (caseous) necrosis, lymphocytes, macrophages and multinucleated giant cells reacting to the presence of Mycobacterium tuberculosis
  • Less commonly, the inflammatory infiltrates can contain a significant number of neutrophils, particularly in acute cases
Essential features
  • Leptomeningeal or localized parenchymal infections that occur via hematogenous dissemination of inhaled M. tuberculosis
    • This can occur at the time of initial infection or later in the course of more chronic, established infections
  • Infection occurs when M. tuberculosis is inhaled and interacts with pulmonary alveolar macrophages
  • Low level bacteremia is present during the early stages of infection, during which time seeding of the leptomeninges or brain parenchyma can occur
    • Bacteremia can persist in patients with progressive disease or recur in patients with reactivation of latent infection
  • Radiographic imaging can provide important clues to the correct diagnosis
  • Classical features of leptomeningeal disease include prominent basilar leptomeningeal enhancement, sometimes associated with infarcts in deep gray and white matter of the cerebral hemispheres
  • Parenchymal lesions (tuberculomas or less commonly, tuberculous abscesses) present as solitary or multiple mass lesions, most commonly in the cerebral hemispheres in adults or in the cerebellum in children, often associated with ring enhancement
  • Identification of M. tuberculosis bacilli in cerebrospinal fluid or in biopsies of parenchymal lesions via acid fast stains or culture or the identification of M. tuberculosis related DNA sequences by PCR or similar nucleic acid amplification techniques is required for a definitive diagnosis
Terminology
  • Diagnostic terms include tuberculous meningitis, meningeal tuberculosis and tuberculomas
ICD coding
  • ICD-10:
    • A17 - tuberculosis of the nervous system
      • A17.0 - tuberculous meningitis
      • A17.1 - meningeal tuberculoma
      • A17.8 - other tuberculosis of the nervous system
        • A17.81 - tuberculoma of brain and spinal cord
        • A17.82 - tuberculous meningoencephalitis
        • A17.83 - tuberculous neuritis
      • A17.9 - tuberculosis of the nervous system, unspecified
Epidemiology
  • Tuberculosis (TB) remains a leading cause of morbidity and mortality, with an estimated 10.6 million cases noted worldwide in 2021 and an estimated 1.6 million deaths during the same year (WHO: Global Tuberculosis Report 2022 [Accessed 6 April 2023])
  • The number of newly diagnosed TB cases increased by 3.6% between 2020 and 2021, reversing a steady decline in new cases noted over the preceding 2 decades
  • CNS tuberculosis is estimated to occur in ~1% of all TB cases
  • Children and HIV positive individuals account for a disproportionate number of cases of CNS TB
Sites
  • Clinically apparent infections can develop within the leptomeninges (TB meningitis) or as localized, space occupying lesions within the parenchyma of the brain or spinal cord (tuberculomas or less commonly, tuberculous abscesses)
    • Tuberculomas may also occur in the leptomeninges, dura and extradural spaces
  • TB meningitis and tuberculomas may coexist
  • TB spondylitis (Pott disease) involving the thoracic or lumbosacral vertebrae, although not strictly a CNS infection, can also cause significant neurological disease when the compromised vertebrae collapse and lead to mechanical injury of the spinal cord (J Spinal Cord Med 2011;34:440)
Pathophysiology
  • Infection with M. tuberculosis occurs when individuals inhale airborne droplets that contain viable bacilli
    • Droplets measuring < 5 μm in diameter reach pulmonary alveoli, where bacilli come into contact with alveolar macrophages
  • In the presence of alveolar surfactant protein A, mycobacterial surface proteins known as pathogen associated molecular patterns bind to receptors expressed on alveolar macrophages, including CD44, Fc, mannose, immunoglobulin, complement and Toll-like receptors (Front Cell Infect Microbiol 2021;10:618414, Semin Immunol 2014;26:471, Microbes Infect 2001;3:37)
  • Binding of the mycobacteria to these receptors facilitates phagocytosis of the bacilli and fusion of engulfed microorganisms with lysosomes (Front Cell Infect Microbiol 2021;10:618414)
    • These interactions also trigger the generation of multiple chemokines and cytokines that activate, in turn, a host T helper cell mediated immune response and the formation of granulomas with varying degrees of caseating necrosis
  • During the initial phase of the host response, a low level bacteremia occurs before the mycobacteria are completely contained within macrophages and dissemination of the organisms to other oxygen rich sites, including the CNS, can occur (Clin Microbiol Rev 2008;21:243)
  • Host reaction to mycobacterial antigens is complex, containing elements that promote inactivation and killing of the bacilli, as well as additional elements that paradoxically allow M. tuberculosis to evade the host immune response (Semin Immunol 2014;26:471)
  • Although most mycobacteria are eventually well contained in immunocompetent hosts, a small number of organisms may remain viable even while sequestered in macrophages, resulting in continuous remodeling of granuloma (Front Cell Infect Microbiol 2021;10:618414)
  • Reactivation of latent infection, with release of microorganisms sequestered within macrophages, can occur whenever the host's normal immune response is compromised, as in instances of malnutrition, HIV or iatrogenic immunosuppression (Front Cell Infect Microbiol 2021;10:618414)
  • As in the case of bacteremia occurring in the early stages of M. tuberculosis infection, hematogenous dissemination of organisms can also occur during such periods of reactivation
  • Circulating cytokines, particularly TNFa, appear to play a role in increasing blood - brain barrier permeability and allowing mycobacteria to gain access to the CNS, where organisms may find their way to the meninges, brain parenchyma or spinal cord (Front Cell Infect Microbiol 2021;10:618414, Clin Microbiol Rev 2008;21:243)
  • Microglia appear to be the major target for mycobacteria that gain access to the CNS (Clin Microbiol Rev 2004;17:942, Clin Microbiol Rev 2008;21:243)
    • Within the CNS M. tuberculosis interacts with surface receptors on resident microglia; among these, the CD14 appears to play a particularly important role in the ingestion of mycobacteria by microglial cells (Infect Immun 1995;63:1598)
  • M. tuberculosis infected microglia release a host of cytokines and chemokines, including TNFa, IL6, IL1b and CXCL10; IL1 and IL10 production, in contrast, is inhibited, particularly with more virulent strains of M. tuberculosis (Clin Microbiol Rev 2008;21:243, Neurochem Int 2004;44:381)
  • Interactions between M. tuberculosis and resident microglia trigger a sequence of cellular events to those seen in the infection of pulmonary alveolar macrophages, resulting in the formation of single or multiple granulomas termed Rich foci, named after the work of Dr. Arnold Rich at The Johns Hopkins Hospital (Bull Johns Hopkins Hosp 1933;52:5)
  • Depending upon their location, rupture of Rich foci release mycobacteria into the subarachnoid space or CNS parenchyma, resulting in the development of meningitis or localized parenchymal infections, respectively (J Neurol Sci 2021;421:117278)
Etiology
  • Although a number of different mycobacterial species can cause human infection, virtually all cases of CNS TB are caused by Mycobacterium tuberculosis, a nonmotile, strictly aerobic bacillus
  • Cell wall of M. tuberculosis is rich in lipids, including mycolic acid, cord factor and wax D, which confer a significant degree of resistance to lysosomal bactericidal activity within macrophages (J Neurol Sci 2021;421:117278)
  • Characterized by a slow generation time in vivo and in vitro, with positive cultures often taking 4 - 6 weeks to develop (J Neurol Sci 2021;421:117278)
Clinical features
Diagnosis
  • Diagnosis of CNS TB is dependent on a combination of clinical findings (see above), radiographic imaging studies and laboratory evaluation of cerebrospinal fluid (CSF) or CNS biopsies
  • Evidence of tuberculous infection elsewhere (e.g., lung) provides a very important clue to the possibility of CNS TB in patients with neurological abnormalities
Laboratory
  • Includes assessment of CSF protein and glucose levels, CSF cytology and identification of M. tuberculosis in CSF or tissue biopsies via acid fast stains, isolation of M. tuberculosis in culture and identification of bacterial nucleic acids and antigens and measurement of adenosine deaminase and tuberculostearic acid in CSF
  • CSF profiles in TB meningitis typically include a lymphocytic pleocytosis, elevated protein levels and decreased glucose levels (Clin Microbiol Rev 2008;21:243, Curr Opin Neurol 1994;7:191, Clin Neuroradiol 2019;29:3)
    • Protein levels may range from 0.5 to 3 g/L, while CSF glucose levels are usually < 45 mg/dL (or below 40 - 60% of serum glucose levels) (Clin Neuroradiol 2019;29:3)
    • While a lymphocytic pleocytosis is most commonly seen, in a minority of patients, a neutrophilic pleocytosis may be present, either early in the course of the disease or shortly after initiation of antituberculous antibiotics (Clin Microbiol Rev 2008;21:243, Acta Cytol 1982;26:678)
    • CSF profiles in parenchymal tuberculomas are nonspecific in the absence of concomitant leptomeningeal disease
    • Cytological evaluation of CSF in suspected CNS TB cases is imperative, not only to characterize any inflammatory cells but to exclude leptomeningeal carcinomatosis, the latter being an important differential diagnostic consideration in suspected TB meningitis
  • Morphological identification of M. tuberculosis in CSF or in smears or histological sections of tissue biopsies using the Ziehl-Neelson or Kinyoun acid fast stain (AFB) continues to play a key role in the diagnosis of CNS TB (Clin Microbiol Rev 2008;21:243)
    • Centrifugation of large volumes (10 - 20 mL) of CSF with AFB staining of the resultant sediment has been shown to improve the sensitivity of detection of M. tuberculosis in CSF samples to 91% (J Clin Pathol 1953;6:241)
  • Isolation of M. tuberculosis from cultures of CSF or tissues remains the gold standard in the diagnosis of CNS TB; isolation of M. tuberculosis in culture has the additional benefit of enabling laboratories to test for drug sensitivity, an important consideration with the emergence of drug resistant strains of the organism (Clin Microbiol Rev 2008;21:243)
    • Because M. tuberculosis grows slowly in vitro, treatment decisions must usually be made before culture results are available (J Neurol Sci 2021;421:117278)
  • Molecular diagnosis of CNS TB, specifically the detection of M. tuberculosis associated nucleic acids by a variety of nucleic acid amplification (NAA) tests has supplemented traditional staining and culture in the diagnosis of CNS TB and offers the special advantage of yielding positive results more rapidly than AFB staining or cultures (J Neurol Sci 2021;421:117278)
    • Reported sensitivities of commercially available NAA assays range from 56 - 100%, with diagnostic specificities ranging from 96 - 100% (Clin Microbiol Rev 2008;21:243)
    • NAA assays continue to evolve, with newer forms offering greater sensitivity and ease of use than earlier generation tests; although a positive NAA assay provides useful diagnostic information, as with any laboratory test, a negative assay does not exclude the possibility of TB
  • Detection of M. tuberculosis associated antibodies and antigens in CSF is used in some laboratories, with reported sensitivities ranging from 52 - 94% and specificities ranging from 96 - 100% (Clin Microbiol Rev 2008;21:243)
  • Measurement of adenosine deaminase and tuberculostearic acid in CSF have also been used in the evaluation of suspected CNS TB, albeit much less widely than traditional morphology, culture and NAA assays (Clin Microbiol Rev 2008;21:243)
Radiology description
  • Imaging of the brain (computerized tomography [CT] and magnetic resonance imaging [MRI]) is extremely important in the evaluation of patients with suspected TB meningitis or tuberculoma(s)
  • Radiographic imaging in cases of suspected CNS TB should include evaluation of the entire neuroaxis
  • Contrast enhanced MRI is generally superior to CT in terms of both specificity and sensitivity in the diagnosis of CNS TB (Clin Neuroradiol 2019;29:3)
  • Important radiographic features of TB meningitis area basal meningeal enhancement, hydrocephalus and cerebral infarcts (particularly in the basal ganglia, anterior thalamus and internal capsules) (Clin Neuroradiol 2019;29:3, Radiol Res Pract 2015;2015:202806)
  • Parenchymal tuberculomas may be solitary or multiple and may be associated with meningitis
    • Radiographic appearance varies with the age of the lesion, the presence or absence of caseous necrosis (see histology below) and the presence or absence of calcification
  • CT features of tuberculomas include areas of low density associated with peripheral ring enhancement and edema; calcification may be seen in older lesions, producing a characteristic target sign (Radiol Res Pract 2015;2015:202806)
  • MRI scans of caseating tuberculomas reveal areas of iso or hypointensity surrounded by a hyperintense rim in nonenhanced T1 weighted images; areas of peripheral ring enhancement are commonly seen after administration of gadolinium (Clin Imaging 2020;68:263)
  • MRI scans of noncaseating tuberculomas reveal areas of hypointensity in noncontrast enhanced T1 weighted images, with a pattern of homogenous enhancement following the administration of gadolinium (Clin Neuroradiol 2019;29:3)
  • Tuberculous abscesses may be solitary or multiple and have radiographic appearances quite similar to those seen in pyogenic bacterial abscesses, namely central hypo or isointensity associated with ring enhancement and regional edema
Radiology images

Contributed by Benjamin Greenberg, M.D., James Luby, M.D. and Dennis K. Burns, M.D.
CT, TB meningitis CT, TB meningitis

CT, TB meningitis

MRI, tuberculoma MRI, tuberculoma

MRI, tuberculoma

MRI, tuberculous abscess

MRI, tuberculous abscess

Prognostic factors
  • In cases of TB meningitis, the clinical stage of disease at the time of presentation, as defined by the British Medical Research Council (MRC) criteria, is the single most important predictor of mortality and long term neurologic sequelae (Lancet 1948;1:582, Cochrane Database Syst Rev 2016;4:CD002244, Clin Microbiol Rev 2008;21:243)
  • Mortality in patients with TB meningitis with MRC stage III disease (clouding of sensorium, focal neurological deficits, seizures, involuntary movements) ranges from 50 to 70%, while the outlook in patients with MRC stage I disease is considerably better, with a reported mortality of 18% (Am J Trop Med Hyg 1998;58:28, Clin Microbiol Rev 2008;21:243)
  • Additional indictors of a poor prognosis in TB meningitis include (Postgrad Med J 1999;75:133):
    • Extremes of age (i.e., infants and elderly patients)
    • Malnutrition
    • Chronic alcoholism
    • Immunosuppression
    • Hydrocephalus
    • Markedly elevated CSF protein
    • Very low CSF glucose levels
  • In patients with tuberculoma, the presence of coma and evidence of miliary disease on chest Xray has been associated with a poor prognosis (J Pak Med Assoc 2004;54:83)
Case reports
  • 2 year old previously healthy girl with a fatal case of TB meningitis (Pediatr Rep 2022;14:175)
  • 19 year old woman with TB meningitis presented with progressive motor weakness in the absence of sensory abnormalities (Cureus 2017;9:e1222)
  • 28 year old previously healthy woman with TB meningitis presented with intermittent fever, generalized limb weakness, dysphagia and visual hallucinations (Malays J Med Sci 2017;24:119)
  • 38 year old woman who developed tuberculoma during the course of TB meningitis (Przegl Epidemiol 2013;67:23)
  • 69 year old man with symptomatic tuberculoma (IDCases 2019;17:e00582)
Treatment
  • Prompt treatment of CNS TB, particularly TB meningitis, is critical (J Neurol Sci 2021;421:117278)
  • Most cases of CNS TB can be treated medically, although surgery may be indicated in certain patients (see below)
  • Medical therapy includes (Clin Microbiol Rev 2008;21:243, J Neurol Sci 2021;421:117278)
    • Intensive phase of 4 antituberculous antibiotics administered over a 2 month period
    • Continuation phase of 2 antituberculous antibiotics administered over an additional 7 to 10 months
    • Glucocorticoids
  • Immune reconstitution inflammatory syndrome (IRIS) has been reported in ~33% of patients receiving antituberculous therapy and may be associated with significant clinical deterioration (J Neurol Sci 2021;421:117278)
  • Emergence of multidrug resistant strains of M. tuberculosis may complicate medical treatment
  • Neurosurgical intervention may be indicated in patients with hydrocephalus, in patients with tuberculomas with elevated intracranial pressure and in patients with spinal cord compression (Afr Health Sci 2011;11:116)
    • Surgical treatment of tuberculomas has become much less common with the advent of effective medical therapy
Gross description
Gross images

Contributed by Dennis K. Burns, M.D.
Tuberculous meningitis, ventral surface of brain

Tuberculous meningitis, ventral surface of brain

Tuberculous meningitis, coronal section

Tuberculous meningitis, coronal section

Tuberculoma, cerebral hemispheres

Tuberculoma, cerebral hemispheres

Frozen section description
  • Frozen sections on suspected tuberculomas should be performed with careful attention to barrier protection and minimalization of aerosols, owing to the risk of contamination of cryostats and other equipment in the pathology laboratory and the risk of exposing laboratory workers to M. tuberculosis
  • In particular, the use of pressurized coolants should be scrupulously avoided in the preparation of frozen sections in cases of suspected tuberculoma
  • Frozen sections of tuberculomas have been reported to demonstrate areas of necrosis, epithelioid macrophages and giant cells (NMC Case Rep J 2014;1:33)
  • Specimens of tuberculomas submitted for frozen section are often quite small and may lack specific diagnostic features
Frozen section images

Contributed by Dennis K. Burns, M.D.
Tuberculoma

Tuberculoma

Microscopic (histologic) description
  • Histological hallmark of M. tuberculosis infection is granulomatous inflammation, characterized by the presence of epithelioid macrophages and multinucleated giant cells that accumulate in response to the poorly digestible, waxy, mycolipid rich wall of the organism
  • Areas of caseous necrosis typically develop in areas of granulomatous inflammation over time, reflecting the development of cell mediated immunity and delayed hypersensitivity to M. tuberculosis associated antigens (Antimicrob Agents Chemother 2003;47:833)
  • Inflammatory reaction in the subarachnoid space in cases of TB meningitis is associated with the development of proliferative changes in the intima of regional vessels, particularly in penetrating arteries originating from the distal internal carotid arteries, middle cerebral arteries and other components of the circle of Willis (Clin Neuroradiol 2019;29:3)
  • Progressive intimal arterial proliferation leads to ischemic damage in the territories supplied by the affected vessels
Microscopic (histologic) images

Contributed by Dennis K. Burns, M.D.
Paraffin section, tuberculous meningitis

Paraffin section, tuberculous meningitis

Paraffin section, tuberculoma

Paraffin section, tuberculoma

Paraffin section, tuberculous abscess Paraffin section, tuberculous abscess

Paraffin section, tuberculous abscess

Cytology description
  • Cytological studies in cases of CNS TB are most commonly undertaken in the evaluation of CSF samples
  • As noted previously (see Laboratory above), cytological evaluation of CSF for the presence of malignant cells should always be undertaken to exclude the possibility of leptomeningeal carcinomatosis, an important differential diagnostic consideration in suspected TB meningitis
  • As in the case of frozen sections, the preparation of smears from fresh, potentially infectious tissue obtained intraoperatively in suspected CNS TB cases is discouraged and if undertaken, should always be done with the same precautions used in preparing frozen sections
  • Although little has been published about the cytological features of tuberculomas, reports of fine needle aspirations of tuberculomas in other sites have noted the presence of granulomas with epithelioid macrophages (Acta Cytol 1999;43:191)
Positive stains
Sample pathology report
  • Brain, left parietal lobe, resection:
    • Necrotizing granulomatous inflammation with acid fast bacilli, compatible with tuberculoma

  • Brain, autopsy:
    • Granulomatous leptomeningitis with caseous necrosis, proliferative vasculopathy and acid fast bacilli, compatible with tuberculous meningitis
Differential diagnosis
  • A large number of infectious and noninfectious entities may mimic TB meningitis and intraparenchymal tuberculomas (Clin Neuroradiol 2019;29:3)
  • Differential diagnoses in suspected TB meningitis include
    • Bacterial (pyogenic) meningitis:
      • CSF characterized by neutrophilic pleocytosis
    • Fungal meningitis:
      • CSF profile may be similar to that seen in TB meningitis
      • Diagnosis dependent upon identification of specific pathogen
    • Nontuberculous mycobacterial meningitis:
      • Generally associated with immunosuppression, CD4 counts of < 50 cells/μl
    • Neurobrucellosis:
      • Identification of anti-Brucella antibodies in CSF is both sensitive and specific
    • Neurosarcoidosis:
      • May be associated with cranial nerve VII dysfunction, pituitary involvement
      • Documentation of elevated ACE or IL2 receptor levels may be of diagnostic value
    • Neurosyphillis:
      • Identification of antitreponemal antibodies or treponemal DNA in CSF is of diagnostic value
    • Viral meningitis / meningoencephalitis:
      • Mononuclear pleocytosis, normal glucose
      • Definitive diagnosis based on identification of specific antiviral antibodies on viral nucleic acid in CSF
    • Leptomeningeal carcinomatosis:
      • Malignant cells detected in CSF
      • CSF profile may include elevated protein and very low glucose levels
  • Differential diagnoses in suspected tuberculoma(s) and tuberculous abscess include
Board review style question #1
A 53 year old man presents for evaluation of intermittent fever and a persistent headache. Neurological examination reveals evidence of decreased visual acuity and bilateral lateral gaze paralysis. An MRI scan reveals communicating hydrocephalus and evidence of meningeal enhancement, most pronounced in the basilar leptomeninges. Which one of the following is the most likely diagnosis in this patient?

  1. Acute bacterial meningitis
  2. Cryptococcal meningitis
  3. Leptomeningeal carcinomatosis
  4. Tuberculous meningitis
  5. Viral meningoencephalitis
Board review style answer #1
D. Tuberculous meningitis. The combination of a cranial nerve VI (abducens) palsy, optic pathway damage, basilar leptomeningeal enhancement and communicating hydrocephalus is highly suggestive of tuberculous meningitis. Identification of M. tuberculosis via microscopy or culture, identification of M. tuberculosis associated nucleic acid via nucleic acid amplification techniques or identification of M. tuberculosis associated antigens in cerebrospinal fluid is necessary for a definitive diagnosis.

Comment Here

Reference: Tuberculosis
Board review style question #2
Which one of the following proteins plays the most important role in the ingestion of M. tuberculosis by CNS microglial cells?

  1. CD4
  2. CD14
  3. CD68
  4. IL1
  5. IL10
Board review style answer #2
B. CD14. The CD14 receptor has been identified as a critical surface receptor that facilitates ingestion of M. tuberculosis by microglia. Answers A and C are incorrect because while CD68 immunoreactivity is typically seen in CNS granulomas in a variety of different conditions, including TB, neither the CD68 nor the CD4 receptor have been shown to play a specific role in the binding to or ingestion of M. tuberculosis by microglia. Answers D and E are incorrect because IL1 and IL10 production are decreased in M. tuberculosis infected microglia.

Comment Here

Reference: Tuberculosis
Board review style question #3
Which one of the following has been associated with a poor prognosis in patients with TB meningitis?

  1. British MRC stage I disease
  2. High CSF glucose levels
  3. Hydrocephalus
  4. Low CSF protein levels
  5. Middle age
Board review style answer #3
C. Hydrocephalus. Of the factors listed, only the presence of hydrocephalus has been associated with a poor prognosis in TB meningitis. The remaining answers are incorrect because other conditions associated with a poor prognosis include disease in either the very young or the very elderly, a very low CSF glucose, a very high CSF protein and severe clinical disease (MRC stage III disease). Malnutrition, immunosuppression and chronic alcoholism have also been associated with a poor prognosis.

Comment Here

Reference: Tuberculosis
Board review style question #4

A 5 year old child was found unresponsive at home. A family member stated that the child had had intermittent fevers and irritability prior to becoming unresponsive. An elderly uncle who lived with the family had also been ill, with a history of fever, coughing and weight loss. Emergency medical personnel were unable to detect a pulse and began cardiopulmonary resuscitation. The child was taken to a nearby emergency, where advanced life support measures were unsuccessful. The case was referred to the medical examiner's office, where an autopsy was performed. A coronal section of the patient's brain is shown in the image above. What is the most likely cause of the lesions in the basal ganglia indicated by the arrows?

  1. Contiguous spread of leptomeningeal infection
  2. Embolic occlusion of vessels in the circle of Willis
  3. Hematogenous dissemination of microorganisms
  4. Obstruction of the flow of cerebrospinal fluid
  5. Vasculitis of lenticulostriate arteries
Board review style answer #4
E. Vasculitis of lenticulostriate arteries. The lesions in the basal ganglia are recent infarcts caused by injury to penetrating lenticulostriate arteries originating in the region of the circle of Willis. The tuberculous exudate, particularly conspicuous at the base of the brain in this patient, is characteristically associated with inflammation (vasculitis) and proliferative changes in regional vessels, with resultant compromise of arterial lumens and infarcts in territories supplied by these vessels.

Comment Here

Reference: Tuberculosis

Vascular dementia (pending)
Table of Contents
Definition / general
Definition / general
[Pending]

Wernicke-Korsakoff syndrome
Definition / general
  • Acute and chronic neuropsychiatric condition secondary to thiamine (vitamin B1) deficiency
  • Wernicke encephalopathy (WE): neuropsychiatric syndrome resulting from thiamine (vitamin B1) deficiency; short lived and severe condition
  • Korsakoff syndrome (KS): a disproportionate impairment in memory relative to other features of cognition and secondary to thiamine deficiency
    • Usually follows or accompanies Wernicke encephalopathy
    • Chronic, long lasting and debilitating condition
Essential features
  • Inability to utilize glucose due to deficiency of thiamine, a cofactor for 3 enzymatic processes
  • Mammillary bodies and periventricular (third and fourth) regions are usually affected
  • Neuropathological changes of Wernicke-Korsakoff syndrome (WKS) depend on age, stage and severity of the disease
Terminology
  • Wernicke disease
  • Wernicke-Korsakoff syndrome
  • Wernicke encephalopathy
  • Korsakoff psychosis
  • Alcohol amnestic disorder
ICD coding
  • ICD-10:
    • E51 - thiamine deficiency
    • E51.2 - Wernicke encephalopathy
      • E51.2 - brain degeneration (cortical) (progressive) in beriberi
      • E51.2 - vitamin B1 encephalopathy (acute)
      • E51.2 - Wernicke disease or syndrome
      • E51.2 - Wernicke polioencephalitis, superior hemorrhagic
    • F10.26 - alcohol dependence with alcohol induced persisting amnestic disorder
      • F10.26 - Wernicke-Korsakoff syndrome or psychosis (alcohol dependence)
    • F10.96 - alcohol use, unspecified with alcohol induced persisting amnestic disorder
      • F10.96 - Wernicke-Korsakoff syndrome or psychosis (alcohol use)
    • F04 - amnestic disorder due to known physiological condition
      • F04 - Wernicke-Korsakoff syndrome or psychosis (nonalcoholic)
    • Korsakoff syndrome induced by other psychoactive substances (F13.26, F13.96, F19.16, F19.26, F19.96)
Epidemiology
Sites
Pathophysiology
  • Thiamine is a cofactor for enzymes involved in energy production and synthesis of biomolecules (see Diagrams / tables)
  • Brain areas with high energy requirement or high thiamine turnover show characteristic lesions
    • Mitochondrial dysfunction → decreased ATP → cell death (necrosis / apoptosis)
    • Decreased glutathione → increased free radicals → oxidative stress
    • Disturbances in carbohydrate metabolism leads to loss of osmotic gradients, resulting in intracellular fluid accumulation (cytotoxic edema) and eventual blood brain barrier breakdown (vasogenic edema)
  • References: Alcohol Res Health 2003;27:134, Int J Biochem Cell Biol 2009;41:717
Etiology
  • Wernicke encephalopathy: thiamine deficiency with or without magnesium deficiency (Lancet 1999;353:1768)
  • Korsakoff syndrome: failure to treat Wernicke encephalopathy
Diagrams / tables

Contributed by Palgun Nisarga, M.D.
Pathophysiology of thiamine deficiency

Pathophysiology of thiamine deficiency



Images hosted on other servers:
Metabolic pathways related to thiamine

Metabolic pathways related to thiamine

Clinical features
  • Wernicke encephalopathy:
    • Triad (seen in 16%); 19% of the patients have none of these symptoms
      • Mental status abnormality (confusion) (34 - 82%)
      • Ocular abnormalities (nystagmus or ophthalmoplegia) (15 - 29%)
      • Ataxia (23 - 25%)
    • Other symptoms: polyneuropathy (11%), hypotension, epileptic seizures, progressive hearing loss (Neuropsychol Rev 2012;22:170)
  • Korsakoff syndrome: profound anterograde amnesia and temporally graded retrograde amnesia with confabulation
  • Clinical course: reversible stages of Wernicke encephalopathy → failure to treat → Korsakoff syndrome or death
Diagnosis
  • Mainly based on clinical and imaging findings
Laboratory
  • Measurement of thiamine or its metabolites in blood / erythrocytes
  • Red blood cell transketolase activity
  • Reference: Jpn J Radiol 2020;38:809
Radiology description
  • Acute Wernicke encephalopathy: bilateral, symmetric T2 and fluid attenuated inversion recovery (FLAIR) signal hyperintensities (figures 1 and 2)
  • Chronic Wernicke-Korsakoff syndrome: structural volume deficits in thalamus and mammillary bodies (figure 3)
  • Reference: Alcohol Alcohol 2009;44:155
Radiology images

Images hosted on other servers:
Acute Wernicke encephalopathy

Acute Wernicke encephalopathy

FLAIR images of acute Wernicke encephalopathy

FLAIR images of acute Wernicke encephalopathy

Chronic Wernicke-Korsakoff syndrome

Chronic Wernicke-Korsakoff syndrome

Prognostic factors
Case reports
  • 28 year old woman, 45 days after childbirth, with recurrent vomiting during pregnancy and medical past of esophagectomy and partial gastrectomy, presented with Wernicke encephalopathy and catatonia (Neurology 2020;94:2378)
  • 54 year old woman with extensive cognitive problems following acute Wernicke encephalopathy (Front Psychiatry 2014;5:59)
  • 56 year old woman with schizophrenia developed Wernicke encephalopathy (Front Psychiatry 2021;12:657649)
Treatment
  • Parenteral thiamine administration, given before or with intravenous glucose
Gross description
  • Changes of Wernicke-Korsakoff syndrome depend on age, stage and severity of the disease
    • Wernicke encephalopathy and Wernicke-Korsakoff syndrome are not distinguishable by gross or microscopic pathology
  • Acute Wernicke encephalopathy:
    • Petechial hemorrhages (and rarely necrosis) involving mammillary bodies and bilateral subcortical regions of periventricular (third and fourth) areas in affected structures
    • 25% have normal appearing mammillary bodies
  • Chronic Wernicke encephalopathy:
    • Atrophy / shrinkage and brown-tan discoloration (hemosiderin) of mammillary bodies
    • Unaffected band of tissue immediately adjacent to the ventricular system
    • Widened third ventricle
    • Atrophy of anterior / superior part of cerebellar vermis (lobes I - IV)
  • References: Gray: Escourolle & Poirier's Manual of Basic Neuropathology, 5th Edition, 2013, Love: Greenfield's Neuropathology, 9th Edition, 2015
Gross images

Contributed by Kymberly A. Gyure, M.D.
Mammillary bodies - acute Wernicke encephalopathy

Mammillary bodies - acute Wernicke encephalopathy

Mammillary bodies - chronic Wernicke encephalopathy

Mammillary bodies - chronic Wernicke encephalopathy

Gross thalamus - acute Wernicke encephalopathy

Thalamus - acute Wernicke encephalopathy

Microscopic (histologic) description
  • Changes of Wernicke-Korsakoff syndrome depend on age, stage and severity of the disease
    • Superimposed cases: acute Wernicke encephalopathy on chronic Wernicke encephalopathy
  • Acute Wernicke encephalopathy:
    • Edema
    • Vascular changes:
      • Hypertrophy or hyperplasia of endothelial cells (all vessel types)
      • Fibrinoid degeneration / necrosis (arterioles)
      • Earliest change is seen (1 - 2 days)
      • Perivascular spaces may contain lipid laden macrophages
    • Hemorrhagic areas reveal extravasated red blood cells and few hemosiderin laden macrophages
      • Ball microhemorrhages
    • Reactive gliosis (follows vascular changes)
    • Neurons are usually spared
    • Loss of myelin
    • Absence of inflammatory reaction
  • Chronic Wernicke encephalopathy:
    • Loss of native tissue (atrophy)
    • Gliosis / astrocytosis
    • Hemosiderin laden macrophages
    • Mild neuronal loss to neuronal sparing
    • Cerebellum
      • Cortical degeneration of anterior / superior cerebellar vermis
      • Patchy loss of Purkinje cells (decrease in 20 - 30% of mean cellular density) with Bergmann gliosis
      • Thinning of molecular layer, atrophy / shrinkage
      • Preserved granule cells
    • Superimposed cases: acute Wernicke encephalopathy on chronic Wernicke encephalopathy
  • Korsakoff psychosis:
    • Findings similar to chronic Wernicke encephalopathy
    • Mammillary bodies and medial dorsal thalamus are affected
  • References: Gray: Escourolle & Poirier's Manual of Basic Neuropathology, 5th Edition, 2013, Love: Greenfield's Neuropathology, 9th Edition, 2015
Microscopic (histologic) images

Contributed by Kymberly A. Gyure, M.D.
Acute Wernicke encephalopathy

Acute Wernicke encephalopathy

Cerebellar vermis - chronic Wernicke encephalopathy

Cerebellar vermis - chronic Wernicke encephalopathy

Sample pathology report
  • Brain, autopsy:
    • History of thiamine deficiency with associated encephalopathy
    • Mammillary body atrophy with relative preservation of neurons and gliosis consistent with Wernicke encephalopathy
Differential diagnosis
  • Alcohol / drug intoxication:
    • Alcohol abuse is one of the etiologies for Wernicke-Korsakoff syndrome
    • Hyperactive delirium could be secondary to possible alcohol withdrawal symptoms
    • Symmetric, hyperintense signal in the basal ganglia, especially the globus pallidus, on T1 weighted MR images
    • Alcohol related brain injury - neuronal loss in superior frontal cortex, hypothalamus (supraoptic / paraventricular nuclei); cerebellum - superior vermis molecular layer atrophy, Purkinje cell loss, Bergmann gliosis
    • Hepatic encephalopathy - affects globus pallidus (commonly), cerebellar dentate, diencephalon, brain stem; Alzheimer type II astrocytosis
  • Bilateral paramedian thalamic infarcts:
    • Infractions to the artery of Percheron (with or without midbrain involvement)
    • Confusion, hypersomnolence, dysarthria, amnesia and ocular movement disorders
    • Noninvasive imaging usually enough for diagnosis (CT, MRI, CT angiogram, MRA angiogram, PET and SPECT)
  • Hypoxic ischemic damage / stroke:
    • Noninvasive imaging usually enough for diagnosis (CT, MRI, CT angiogram, MRA angiogram, PET and SPECT)
    • Circumscribed areas of pallor with red dead neurons (12 - 48 hours), macrophage infiltrate + cavitation of cortex (layers 2 - 6) + loss of axons (3 - 30 days) and circumscribed organized area of cavitation + scattered macrophages + residual glial tissue (months - years)
  • Seizure disorders:
    • Wernicke-Korsakoff syndrome can be associated with seizures
    • Detailed personal and family history is essential
    • Electroencephalography (EEG) is a biomarker for epilepsy
    • Temporal lobe epilepsy - hippocampal sclerosis (classical hippocampal sclerosis [40%] to end folium sclerosis [25%]) (Noebels: Jasper's Basic Mechanisms of the Epilepsies, 4th Edition, 2012)
    • Tumors - ganglioglioma, dysembryoplastic neuroepithelial tumors, anglocentric glioma, gangliocytoma, etc.
    • Focal cortical dysplasia or cortical malformations
    • Others: glial scar, vascular malformations, encephalitis (N Engl J Med 2017;377:1648)
  • Concussion:
    • Traumatically induced transient disturbance of brain function
    • Diagnosis remains an exclusively clinical diagnosis based on history and exam findings
    • Imaging modalities can be employed to rule out other injuries
    • Neurodegeneration reported in mouse model (J Neurotrauma 1994;11:657)
  • Viral (herpes) encephalitis:
    • Preceded by prodromal flu-like illness (including fever and headache) and manifests as superimposed behavioral changes, seizures, focal neurological signs and cognitive impairment
    • MRI shows asymmetric high signal within the medial temporal lobes, insular cortex and orbitofrontal and cingulate cortex
    • Perivascular lymphocytic cuffing, microglial nodules, neuronophagia, demyelination, ependymitis, gliosis
    • Viral inclusions are rarely seen
      • Intranuclear inclusions (Cowdry type A: herpes simplex, oligodendrocytic JC virus)
      • Cytoplasmic inclusions (Negri bodies; rabies)
    • Pathogen specific immunohistochemistry
  • Neurodegenerative / demyelinating disease:
    • Diagnosis remains an exclusively clinical diagnosis based on history, exam, laboratory and ancillary test findings
    • Most reliable method to detect mild cognitive impairment is neuropsychological testing
    • Includes diagnostic criteria to diagnose demyelinating disorders
    • Multiple sclerosis - myelin loss + relative axonal sparing, macrophage infiltrate (activity), chronic inactive / active plaques
    • Acute demyelinating encephalomyelitis - perivenular cuffs of myelin loss, perivascular macrophages
    • Acute hemorrhagic leukoencephalitis - fibrinoid necrosis of postcapillary venules + hemorrhage (ring and ball), neutrophilic debris and fibrin exudates + ischemic changes
    • Tumefactive demyelinating lesion - myelin loss + relative axonal sparing; macrophages, reactive astrocytosis, mitoses and variable perivascular / parenchymal lymphocytes
  • Neoplasia:
    • Clinical history, imaging and tissue biopsy is helpful to differentiate
Board review style question #1
Which of the following pathological features is seen in acute Wernicke encephalopathy?

  1. Acute inflammation
  2. Alzheimer type II astrocytes
  3. Endothelial hyperplasia
  4. Geographic necrosis
  5. Granuloma
Board review style question #1
C. Endothelial hyperplasia

Comment Here

Reference: Wernicke-Korsakoff syndrome
Board review style question #2

Which of the following is the most common site to be involved in Wernicke-Korsakoff syndrome?

  1. Cerebellar vermis
  2. Corpora quadrigemina and tectum
  3. Dorsal medial nucleus of thalamus
  4. Mammillary body
  5. Medial pulvinar
Board review style answer #2
D. Mammillary body

Comment Here

Reference: Wernicke-Korsakoff syndrome

Whipple disease (pending)

X linked bulbospinal neuronopathy (Kennedy disease)
Definition / general
  • X linked recessive disorder that predominantly affects men, usually between the ages of 20 and 50, causing slowly progressive weakness of the proximal limbs, lower motor neuron and facial (bulbar) muscles as well as facial (perioral) fasciculation, loss / reduction of reflexes, diabetes mellitus and gynecomastia
  • Often associated with gynecomastia due to an expansion of a repeat of the trinucleotide CAG encoding glutamine in the androgen receptor gene (J Mol Neurosci 2016;58:313, Neurol Clin 2015;33:847)
  • Commonly associated with infertility and primary sensory neuronopathy
    • Neuronopathy: polyneuropathy involving destruction of the cell bodies of neurons
  • Patients with X linked bulbospinal neuronopathy generally live a normal life span; milder form of the disease has been reported in elderly men (Neurology 2008;70:1967)
Essential features
  • Rare neurodegenerative disorder characterized by slowly progressive weakness of the proximal limbs, lower motor neuron and facial (bulbar) muscles as well as facial (perioral) fasciculation, loss / reduction of reflexes, diabetes mellitus and gynecomastia (Neurol Clin 2015;33:847)
  • X linked recessive disorder that predominantly affects men
    • Expansion of tandem CAG repeat region in first exon of androgen receptor gene on proximal part of long arm of X chromosome (Xq11-12) (Nature 1991;352:77)
  • Characteristic clinical features and genetic testing can help distinguish X linked bulbospinal neuronopathy from amyotrophic lateral sclerosis (ALS)
  • Patients with X linked bulbospinal neuronopathy generally live a normal life span, despite not having treatment options that halt the progression of the disease
Terminology
  • X linked bulbospinal neuronopathy
  • Spinobulbar muscular atrophy
  • Kennedy disease
  • Spinal and bulbar muscular atrophy
  • Spinal bulbar muscular atrophy
  • X linked spinal and bulbar muscular atrophy
  • X linked spinal bulbar muscular atrophy
ICD coding
  • ICD-10:
    • G12.1 - other inherited spinal muscular atrophy
    • G12.2 - motor neuron disease
    • G12.9 - spinal muscular atrophy, unspecified
Epidemiology
  • Rare disorder with distinct populations and varying prevalence
  • Affected men typically develop weakness in third and fourth decades of life, plus androgen insensitivity with reduced fertility and gynecomastia (Neurol Clin 2015;33:847)
    • Age range: 18 - 64 years
  • General impression is that X linked bulbospinal neuronopathy, due to mild symptoms exhibited by some patients, may be underdiagnosed (Arch Neurol 2002;59:1921)
Sites
  • Proximal limb muscles
  • Bulbar muscles
  • Sensory nerve
Pathophysiology
  • X linked recessive disorder
  • Expansion of tandem CAG repeat region in first exon of androgen receptor (AR) gene on proximal part of long arm of X chromosome (Xq11-12)
    • There is variable phenotypic expression between and within families, which is not clearly related to the size of the CAG expansion
    • Repeat lengths of 38 - 62 CAGs have been reported in patients, versus 11 - 32 CAGs in normal individuals (Muscle Nerve 1995;18:1378)
  • CAG repeat is expressed as an expanded polyglutamine tract in the androgen receptor; studies indicate that androgen dependent gain of function by the receptor results in toxicity of the mutant protein
  • In addition to the toxic effects of the AR, loss of normal receptor function also contributes to the androgen insensitivity aspects of the disease phenotype (Neuron 2014;82:251)
  • Translocation of the mutant AR into the nucleus also seems necessary for toxicity
    • Deletion of the nuclear localization signal prevents toxicity in a mouse model
  • Presence of nuclear inclusion bodies is a pathological hallmark of the polyglutamine disease
    • In X linked bulbospinal neuronopathy, nuclear inclusions contain mutated truncated androgen receptor and are most commonly seen within motor neurons in the brainstem and spinal cord (Ann Neurol 1998;44:249)
  • There is evidence for mitochondrial dysfunction in X linked bulbospinal neuronopathy, which is likely a result of complex interactions of elongated polyglutamine (polyQ) AR with mitochondrial protein and nuclear / DNA, resulting in oxidative stress and ultimately activation of the mitochondrial caspase pathway or decreased mitochondrial membrane potential (Hum Mol Genet 2009;18:27)
Etiology
  • Trinucleotide CAG repeat expansion in the gene encoding the androgen receptor located on the X chromosome (Xq11-12)
Diagrams / tables

Images hosted on other servers:
Impacts of spinal and bulbar muscular atrophy

Impacts of spinal and bulbar muscular atrophy

Clinical features
  • Facial, hypoglossal and spinal cord motor neuron degeneration with neurogenic wasting of corresponding skeletal muscles (particularly tongue)
    • Third, fourth and sixth (oculomotor, trochlear and abducens) cranial nerves are spared
  • Slowly progressive lower motor neuron weakness of facial, bulbar and proximal limb muscles with fasciculations (Neurol Clin 2015;33:847)
    • Progression of weakness is slow, with ~2% decrease in muscle strength by quantitative muscle testing per year
  • Tremor and cramping are also common
  • Often associated with gynecomastia, infertility and primary sensory neuronopathy
  • Family history may be absent in as many as 26 - 60% of patients (Arch Neurol 2002;59:1921)
Diagnosis
  • Electrophysiology
  • Laboratory (serum levels)
  • Imaging
    • Magnetic resonance imaging (MRI)
    • Proton magnetic resonance spectroscopy
  • Molecular diagnosis (Rev Neurol (Paris) 2017;173:326)
Laboratory
  • Laboratory (serum)
    • Elevated serum creatine kinase
    • Normal or elevated serum testosterone levels
    • Elevated serum myoglobin may help differentiate from amyotrophic lateral sclerosis (Int J Neurosci 2021;131:1209)
  • Electrophysiology
  • Genetic test
    • DNA analysis of peripheral blood detection of CAG triplet repeat expansion in the AR gene (Neurology 1997;49:568)
Radiology description
  • Magnetic resonance imaging (MRI)
  • Proton magnetic resonance spectroscopy
    • Significant reduction in the ratio of N-acetyl-aspartate to phosphocreatine in the motor cortex (J Neurol Sci 2009;277:71)
Prognostic factors
  • Slowly progressive disease but life expectancy does not seem to be significantly affected (Neurology 2008;70:1967)
  • Study of 223 cases of X linked bulbospinal neuronopathy shows that those patients with longer CAG repeat size displayed an earlier onset of disability in activities of daily living but the rate of subsequent declines was independent of the triplet repeat size (Brain 2006;129:1446)
  • Urinary levels of the oxidative stress marker 8 hydroxydeoxyguanosine (8-OHdG) correlates with severity of motor dysfunction (Muscle Nerve 2012;46:692)
  • Development of insulin resistance is a clinical feature of X linked bulbospinal neuronopathy and the degree of this resistance has been reported to correlate with disease severity (J Neurol 2017;264:839)
Case reports
Treatment
  • There is currently no effective therapy to prevent progression of the disease
  • Management is focused on preventing complications and improving mobility and function
  • Clinical trials have focused on reducing AR ligand
Clinical images

Images hosted on other servers:
Tongue atrophy

Tongue atrophy

Asymmetric facial muscle atrophy

Asymmetric facial muscle atrophy

Gross description
Microscopic (histologic) description
  • Depletion of lower motor neurons through spinal cord segments and brainstem motor nuclei
  • Muscle histopathology is atypical for a motor neuron disease
    • Cytoplasmic inclusions are lacking and both neurogenic and myogenic alterations are seen, with neurogenic alterations being more common
    • Commonly described morphological changes include groups of small atrophic myofibers with pyknotic nuclei and hypertrophic fibers, sometimes with central nuclei (Clin Neuropathol 2015;34:199)
  • Nuclear inclusions may be found in spinal neurons and brainstem motor neurons
  • Nuclear inclusions can also be found in nonneural tissues, including skin, dermis, kidney, heart and testis (Ellison: Neuropathology - A Reference Text of CNS Pathology, 3rd Edition, 2013)
  • Ultrastructurally, most nuclei contain clusters of dense heterochromatin (Clin Neuropathol 2015;34:199)
Microscopic (histologic) images

Images hosted on other servers:
Muscle biopsy

Muscle biopsy

Muscle biopsy

Pathology of Kennedy disease

Positive stains
Electron microscopy description
Videos

Hand tremor, tongue fasciculation

Sample pathology report
  • Peripheral blood:
    • Positive for CAG repeat expansion in AR (46 repeats) (see comment)
    • Comment: Genetic testing on peripheral blood identified 46 CAG repeats in the androgen receptor gene, which would support a clinical impression of X linked bulbospinal neuronopathy (Kennedy disease).
Differential diagnosis
  • Amyotrophic lateral sclerosis (ALS):
    • Clinically, characterized by upper and lower motor neuron signs / symptoms: gross atrophy of anterior spinal nerve roots, neuronal loss and gliosis affecting anterior horn of spinal cord and primary motor cortex, pallor of corticospinal tracts, Bunina bodies (cystatin C positive) and skein-like inclusions (TDP-43 and ubiquitin positive) in motor neurons
    • No CAG repeat expansion in the androgen receptor gene
    • Familial cases show C9orf72 expansion and SOD1 mutation, which are the 2 most common inherited genetic alterations
  • Hereditary spastic paraplegia:
    • Different clinical findings
      • Primarily affects upper motor neurons with slow degeneration
      • Slow degeneration causes the muscles to not receive the correct messages, causing progressive spasticity
    • No CAG repeat expansion in the androgen receptor gene
    • 80+ genetic mutations cause various types of hereditary spastic paraplegia but some patients have no known mutation
  • Myasthenia gravis:
    • Different clinical findings
      • Symptoms typically occur or are exacerbated after exertion, better in morning and worse at night
      • Positive nerve electrical stimulation
      • Often thymic hyperplasia and hypertrophy by CT
      • Normal tendon reflexes
    • No CAG repeat expansion in the androgen receptor gene
    • Treatment with cholinesterase inhibitors is effective
  • Secondary causes of motor neuron disease (Ellison: Neuropathology - A Reference Text of CNS Pathology, 3rd Edition, 2013):
Board review style question #1
A middle aged adopted man with progressive proximal weakness and perioral fasciculation is suspected to suffer from X linked bulbospinal neuronopathy. Peripheral blood is drawn. What is the expected genetic finding?

  1. C9orf72 mutation
  2. Expansion of tandem CAG repeat
  3. Homozygous deletion of exon 7 in the SMN1 gene
  4. LGMDR1 mutation
  5. SOD1 mutation
Board review style answer #1
B. Expansion of tandem CAG repeat. X linked bulbospinal neuronopathy (Kennedy disease) is a rare, neurodegenerative disorder resulting from the expansion of tandem CAG repeat region in first exon of androgen receptor gene on proximal part of long arm of X chromosome. Answers A, C, D and E are incorrect because these mutations are associated with other disorders. C9orf72 expansion and SOD1 are associated with ALS. Homozygous deletion of exon 7 in the SMN1 gene and SOD1 mutations are associated with spinal muscular atrophy. SMN1 gene is associated with spinal muscular atrophy (Neuro Asia 2022;27;955). LGMDR1 mutation is associated with limb girdle muscular dystrophy (J Neuromuscul Dis 2021;8:125).

Comment Here

Reference: X linked bulbospinal neuronopathy
Board review style question #2
Which of the following features is characteristic of Kennedy disease (spinal and bulbar muscular atrophy)?

  1. It has similar clinical features as amyotrophic lateral sclerosis (ALS)
  2. It is a neurodegenerative disease of upper motor neurons
  3. It is due to expansion of tandem CAG repeat region in the androgen receptor gene
  4. It primarily affects women
Board review style answer #2
C. It is due to expansion of tandem CAG repeat region in the androgen receptor gene. Answer A is incorrect because the clinical features of Kennedy disease differ from amyotrophic lateral sclerosis. Answer B is incorrect because Kennedy disease is a lower neuron degenerative disease. Answer D is incorrect because Kennedy disease primarily affects men.

Comment Here

Reference: X linked bulbospinal neuronopathy
Back to top
Recent CNS nontumor Pathology books

Barbato: 2016

Bault : 2015

Braak: 2015

Gray: 2018

Kleinschmidt-DeMasters: 2022

Kovacs: 2015

Love: 2015

Perry: 2017

Rub: 2015



Find related Pathology books: muscle and peripheral nerve nontumor, neuropathology
Image 01 Image 02